+ All Categories
Home > Documents > SOCIETATEA DE ŞTIINŢE MATEMATICE DIN - ISJ … matematica 2015... · Web viewpentru punctaj maxim...

SOCIETATEA DE ŞTIINŢE MATEMATICE DIN - ISJ … matematica 2015... · Web viewpentru punctaj maxim...

Date post: 16-Mar-2018
Category:
Upload: phungcong
View: 250 times
Download: 8 times
Share this document with a friend
159
SOCIETATEA DE ŞTIINŢE MATEMATICE DIN ROMÂNIA FILIALA MARAMUREŞ INSPECTORATUL ŞCOLAR JUDEŢEAN MARAMUREŞ ANUAR 2015
Transcript
Page 1: SOCIETATEA DE ŞTIINŢE MATEMATICE DIN - ISJ … matematica 2015... · Web viewpentru punctaj maxim la sectiunea liceu Zelina Mihai, clasa a X-a, C.N. „V. Lucaciu”, prof. Boroica

SOCIETATEA DE ŞTIINŢE MATEMATICE DIN ROMÂNIA

FILIALA MARAMUREŞ

INSPECTORATUL ȘCOLAR JUDEȚEAN MARAMUREȘ

ANUAR 2015

BAIA MARE2015

Page 2: SOCIETATEA DE ŞTIINŢE MATEMATICE DIN - ISJ … matematica 2015... · Web viewpentru punctaj maxim la sectiunea liceu Zelina Mihai, clasa a X-a, C.N. „V. Lucaciu”, prof. Boroica

Coordonatori:

prof. univ. dr. Berinde Vasile, preşedinte, Filiala Maramureş a S.S.M.R.prof. Maiorescu Gheorghe, inspector şcolar pentru matematică, I.S.J. Maramureş

Colectivul de elaborare:

prof. Fărcaş Natalia, C.N. „V. Lucaciu” Baia Mareprof. Sfara Gheorghe, C.N. „V. Lucaciu” Baia Mareprof. Breitkopf Marieta, Şc.Gim. „N. Stănescu” Baia Mareprof. Mușuroaia Nicolae, C.N. „Gh. Șincai” Baia Mareprof. Bretan Andrei, Șc.Gim. „N. Iorga” Baia Mareprof. Cosma Simona, Șc.Gim. Nr.18 Baia Mare

Corectura:

prof. Ienuţaş Vasile, Șc.Gim. „G. Coșbuc” Baia Mareprof. Șerba Lucia, C.T. „A. Saligny” Baia Mare

Culegere şi tehnoredactare:

prof. Fărcaş Natalia, C.N. „V. Lucaciu” Baia Mareprof. Breitkopf Marieta, Şc.Gim. „N. Stănescu” Baia Mareprof. Cosma Simona, Șc.Gim. Nr.18 Baia Mareprof. Bobb Laura, Șc.Gim. Nr.18 Baia Mare

2

Page 3: SOCIETATEA DE ŞTIINŢE MATEMATICE DIN - ISJ … matematica 2015... · Web viewpentru punctaj maxim la sectiunea liceu Zelina Mihai, clasa a X-a, C.N. „V. Lucaciu”, prof. Boroica

REZULTATE OBȚINUTE LA CONCURSURILE DE MATEMATICĂ

TABĂRA JUDEŢEANĂ DE EXCELENŢĂ LA MATEMATICĂ1-5 septembrie 2014, Vaser -Valea Babii

În perioada 1.09.2014 - 05.09.2014, din iniţiativa ISJ Maramureş inspector de specialitate Gheorghe Maiorescu şi cu sprijinul CJ Maramureş preşedinte Zamfir Ciceu, s-a desfăşurat Tabăra Judeţeană de Excelenţă, Matematică Vaser – Valea Babii. Au participat un număr de 22 de elevi din localităţile Baia Mare şi Vişeu de Jos, de la C.N. „Gh. Şincai”, C.N. „V. Lucaciu”, Şc.Gim. „N. Iorga”. Timpul, peisajul, gazdele au fost deosebite în această perioadă a anului, atmosfera fiind una de studiu foarte serios din toate punctele de vedere.

Aducem mulțumiri conducerii Liceului Teoretic „Bogdan Vodă” Vișeu de Sus, director Olar Paul Eusebiu, director adjunct Pop Vesel Floare, pentru implicarea deosebită în desfășurarea taberei și domnului Coman Vasile, director S.C.R.G. Holz Company Vișeu de Sus, care prin sponsorizare a mijlocit transportul elevilor pe ruta Vișeu de Sus-Valea Babii și retur.

Au susţinut cursuri profesorii: Muşuroaia Nicolae, Boroica Gheorghe, Bojor Florin, Pop Adrian – de la C.N. „Gh. Şincai” Baia Mare; Bretan Andrei şi Zah Ştefan - de la Şc.Gim. „N. Iorga” Baia Mare; Pop-Vesel Floare – de la Lic.Teor. Vişeu de Sus; Tomoiagă Ioan - Şc.Gim. Vişeu de Jos. Au fost prezenți în calitate de asistent medical d-na Papp Adriana, iar ca director de tabără d-l prof. Bretan Andrei.

Premiile oferite elevilor participanţi au fost următoarele:Clasa a VI-a: Talpoş Carina, Zaharia Oana de la Şc.Gim. „N. Iorga”; Maxim Sonia, Herzal Radu de la C.N. „Gh. Şincai” – premiul I; Pop Raul de la Şc.Gim. Vişeu de Jos – premiul II; Clasa a VII-a: Pop Călin de la Şc.Gim. „N. Iorga”, Becsi Paul C.N. „Gh. Şincai” – premiul I; Boroica Adrian, Andreicuţ Teofil de la C.N. „Gh. Şincai” – premiul II; Francioli Daria de la C.N. „V. Lucaciu” – premiul III; Clasa a VIII-a: Mercea Ioana de la C.N. „Gh. Şincai”, Petz Alin de la C.N. „V. Lucaciu” – premiul I; Clasa a IX-a: Lucaciu Sergiu, Bojor Barbu, Pop Vlad de la C.N. „Gh. Şincai” – premiul I; Tămâian Andrei, Mărieş Maria de la C.N. „Gh. Şincai” – premiul II; Clasa a XI-a: Cotan Paul, Ţînţar Oana de la C.N. „Gh. Şincai” – premiul I; Butnar Adrian de la C.N. „Gh. Şincai” – premiul II; Clasa a XII-a: Ciurte Tudor, Bud Cristian de la C.N. „Gh. Şincai” – premiul I.

OLIMPIADA DE MATEMATICĂ – etapa județeană14.03.2015

Locul de desfăşurare: Pentru clasele IV-VII - Şcoala Gimnazială „N. Iorga” Baia Mare Pentru clasele VIII-XII - Colegiul Naţional „Gheorghe Şincai” Baia Mare

3

Page 4: SOCIETATEA DE ŞTIINŢE MATEMATICE DIN - ISJ … matematica 2015... · Web viewpentru punctaj maxim la sectiunea liceu Zelina Mihai, clasa a X-a, C.N. „V. Lucaciu”, prof. Boroica

Concursul de Matematică Aplicată „Adolf Haimovici” - Colegiul Naţional „Gheorghe Şincai” Baia Mare Comisia de organizare și desfășurare a olimpiadei de matematică:Preşedinte: prof. Ban Vasile, inspector şcolar general adjunctPreşedinţi executivi: prof. Maiorescu Gheorghe, inspector şcolar pentru matematică; prof. Fornvald Natalia, inspector şcolar pentru învățământ primar.Vicepreşedinţi: prof. Heuberger Cristian, C.N. „Gheorghe Șincai” Baia Mare; prof. Boloș Mihai, Șc.Gim. „N. Iorga” Baia MareSecretari: prof. Bojor Florin, C.N. „Gh. Șincai” Baia Mare; prof. Podină Camelia, Lic. Teor. „E. Racoviță” Baia Mare; prof. Peter Zita, Șc.Gim. „N. Iorga” Baia MareMembri: prof. Berci Ioan, C.N. „D. Vodă” Sighetu Marmaţiei; prof. Boga Ovidiu L. T. „Gr. Moisil” Târgu Lăpuş; prof. Boroica Gabriela, C.N. „V. Lucaciu” Baia Mare; prof. Breitkopf Marieta, Șc.Gim. „N. Stănescu” Baia Mare; prof. Bretan Andrei, Șc.Gim. „N. Iorga” Baia Mare; prof. Codrea Lucia, Șc.Gim. „N. Iorga” Baia Mare; prof. Cosma Simona, Șc.Gim. Nr.18 Baia Mare; prof. Covaciu Viorica, Șc.Gim. „N. Iorga” Baia Mare; prof. Gherasin Gheorghe, Lic.Teor. „Regele Ferdinand” Sighetu Marmaţiei; prof. Heuberger Dana, C.N. „Gh. Şincai” Baia Mare; prof. Ienuțaș Vasile Șc.Gim. „G. Coşbuc” Baia Mare; prof. Lapsanszki Edith, Șc.Gim. „N. Iorga” Baia Mare; prof. Lopată Angela, Șc.Gim. Gârdani; prof. Mastan Eliza, Lic.Teor. „N. Laszlo” Baia Mare; prof. Mic Vasile, Șc.Gim. „G. Coşbuc” Sighetu Marmaţiei; prof. Mihali Marinela, Liceul Borșa; prof. Muşuroia Nicolae, C.N. „Gh. Şincai” Baia Mare; prof. Plici Marina, C.N. „Gh. Şincai” Baia Mare; prof. Sfara Gheorghe, C.N. „V. Lucaciu” Baia Mare; prof. Stark Andreea, Șc. Gim. „N. Iorga” Baia Mare; prof. Ștețco Mariana, Lic.Pedagogic „Regele Ferdinand” Sighetu Marmaţiei; prof. Temian Gavrilă, C.E. „N. Titulescu” Baia Mare; prof. Tomoiagă Ioan, Șc.Gim. Vişeu de Jos; prof. Ujlaki Zita, Șc.Gim. „N. Iorga” Baia Mare; prof. Urda Maria, Șc.Gim. Nr. 1 MoiseiLista premianţilorClasa a IV-a:Premiul I: Mărcuş Alexandru (Şc.Gim. „N. Iorga”, înv. Tarţa Diana/ Petrean Ramona), Tuş Traian (Şc.Gim. „N.Stănescu”, înv. Pop Luminiţa), Costin Oana (Şc.Gim. „N.Stănescu”, înv. Pop Luminiţa), Muntean Tudor (Şc.Gim. „N. Iorga”, înv. Covaciu Viorica), Maghiari Iulia (Şc.Gim. „D. Cantemir”, înv. Coroian Emilia), Crişan Samuel (Şc.Gim. „D. Cantemir”, înv. Bîrle Delia), Paul Ioana (Şc.Gim. „A. Iancu”, înv. Berlinger Maria), Fechete Şerban (Şc.Gim. „N.Stănescu”, înv. Pop Luminiţa), Şpan Mihai (Şc.Gim. „N. Iorga”, înv. Mărieş Zorica), Tiut Cristian (Şc.Gim. „G. Coşbuc”, înv. Dragoş Maria), Marchiş Marcus (Şc.Gim. „D. Cantemir”, înv. Bîrle Delia), Pop Andra (Şc.Gim. „D. Cantemir”, înv. Pop Irma), Rodila Andrei (Lic.Ped. „Regele Ferdind” Sighet, înv. Orban Dolly), Tibil Oana (Şc.Gim. „G. Coşbuc”, înv. Dragoş Maria), Trandafirescu Alexa (Şc.Gim. „D. Cantemir”, înv. Coroian Emilia), Miclăuş Carmen (Şc.Gim. „D. Cantemir” , înv. Coroian Emilia), Ariciu Andra (Şc.Gim. „D. Cantemir” , înv. Pop Irma), Abrudan Sara (Şc.Gim. „S. Bărnuţiu”, înv. Micle Maria), Ianoş Andreea (Şc.Gim. „N.Stănescu”, înv. Pop Luminiţa), Sângeorzan Luca (Şc.Gim. „Al. Ioan Cuza”, înv. Barbur Maria Cicica), Tanasă Denis Viorel (Şc.Gim. nr.9 Borşa , înv. Tomoiagă

4

Page 5: SOCIETATEA DE ŞTIINŢE MATEMATICE DIN - ISJ … matematica 2015... · Web viewpentru punctaj maxim la sectiunea liceu Zelina Mihai, clasa a X-a, C.N. „V. Lucaciu”, prof. Boroica

Iuliana), Colopelnic Daria ( Lic.Ped. „Regele Ferdind” Sighet, înv. Orban Dolly); Premiul II: Hanţig Lorena Maria (Şc.Gim. nr.9 Borşa , înv. Timiş Cornelia), Rogozsan Robert (Şc.Gim. „I. L. Caragiale”, înv. Roman Cristina), Vascul Andrei (Şc.Gim. Baia Sprie, înv. Chereji Anamaria), Grumaz Tudor (Şc.Gim. „A. Iancu”, înv. Berlinger Maria), Felleg Kristina (Şc.Gim. „N.Stănescu”, înv. Tyekar Ana-Maria), Buda Larisa Ioana (Şc.Gim. „I. L. Caragiale”, înv. Roman Cristina), Şimon Adrian Vasile (Şc.Gim. Vişeu de Jos, înv. Pop Anişoara), Dolca Diana (Şc.Gim. „L. Rebreanu” Dragomireşti, înv. Ofrim Floare), Dunca Ioniţa (Şc.Gim. „L. Rebreanu” Dragomireşti, înv. Bodea Maria), Hanos Izabela (Şc.Gim. „A. Iancu”,înv. Berlinger Maria), Indrecan Călin (Şc.Gim. „D. Cantemir”, înv. Pop Irma), Moisi Samuel (Lic.Ped. „Regele Ferdind” Sighet, înv. Grigor Dorina), Onea Iulian (Şc.Gim. „N. Iorga”, înv. Mărieş Zorica), Herte-Turda Alexandra (Şc.Gim. „D. Cantemir”, înv. Coroian Emilia), Pascu Dacian (Liceul de Arte, înv. Chiorean M.), Pop Rahela (Şc.Gim. „D. Cantemir”, înv. Bîrle Delia), Drob Constantin (Şc.Gim. „M. Eminescu” Săliştea de Sus, înv. Vancea Florina Teodora), Şerban Raul (Şc.Gim. „N. Iorga”,înv. Tarţa Diana/Petrean Ramona), Tomoiaga Antonia (Şc.Gim. „G. Coşbuc” Sighet, înv. Şerban Margareta), Săsăran Aurelian Noris (Şc.Gim. „I. L. Caragiale”, înv. Roman Cristina), Pascu Ilie Iulian (Şc.Gim. Vişeu de Jos, înv. Pop Anişoara), Stremtan Daria Alessia (Şc.Gim. nr.7 Vişeu, înv. Paşcu Elvira), Pop Darius (Şc.Gim. „G. Coşbuc”, înv. Dragoş Maria), Buia Theodor (Şc.Gim. „D. Cantemir”, înv. Bîrle Delia), Arduszădan Darius (Şc.Gim. „N.Stănescu”, înv. Tyekar Ana-Maria), Barbul- Zetea Luca (L.T.A. ,,Alexiu Berinde” Seini, înv. Sabo Doina), Nodiş Teodora (Şc.Gim. „O. Goga”, înv. Dorca Onorica), Şimonca Giulia (Şc.Gim. „D. Cantemir”, înv. Coroian Emilia), Ghiţ Marian (Şc.Gim. „Al. Ioan Cuza”, înv. Barbur Maria Cicica), Ghişe Daria (Şc.Gim. „V. Lucaciu” Şişeşti, înv. Rus Elvira), Şimon Gabriel Adrian (Şc.Gim. Vişeu de Jos, înv. Pop Anişoara), Cordea Dania Mălina (Şc.Gim. „V. Alecsandri”, înv. Achim Simona); Premiul III: Oana Cristiana (L.T. „Gr. C. Moisil” Tg. Lăpuş, înv. Tuns Valerie), Coroian-Mariş Ioana (Şc.Gim. „G. Coşbuc” Sighet, înv. Siman Doina), Giurgiu Alex (Şc.Gim. „N. Iorga”, înv. Mărieş Zorica), Varga Renata (Şc.Gim. „G. Coşbuc”, înv. Dragoş Maria), Kovacs Patrick (Şc.Gim. „A. Iancu”, înv. Coza Ioana), Andron Darius (Şc.Gim. „D. Cantemir”, înv. Coroian Emilia), Bonte Daiana (Şc.Gim. „D. Cantemir”, înv. Bîrle Delia), Dumitru Marian (Şc.Gim. „L. Blaga”, înv.Zah Maria), Botiş Alexandru (Şc.Gim. „N.Stănescu”, înv. Tyekar Ana-Maria), Ghiras Tudor (Şc.Gim. „N.Stănescu”, înv. Pop Luminiţa), Ocian Oana (Şc.Gim. „D. Cantemir”, înv. Pop Irma), Tunyogi Kevin (Şc.Gim. „N.Stănescu”, înv. Paşcaniuc Zita), Bota Alexandra (Şc.Gim. „Dr.Ilie Lazar” Giulesti, înv. Mois Ioana), Pop Ana (Şc.Gim. „V. Babeş”, înv. Hotea Victoria), Boitor Ariana (Şc.Gim. „Al. Ivasiuc”, înv. Filip Maria), Gheţe Ruxandra (Şc.Gim. „A. Iancu”, înv. Coza Ioana), Pausan Eleia (Şc.Gim. „G. Coşbuc”, înv. Chiorean Adela), Coman David (Şc.Gim. „D. Cantemir”, înv. Bîrle Delia), Prună Ştefana (Şc.Gim. „N. Iorga”, înv. Tarţa Diana/Petrean Ramona), Petrovan Alexandra (Şc.Gim. „G. Coşbuc” Sighet, înv. Şerban Margareta), Neamțu Luca (L.T.A. ,,Alexiu Berinde” Seini, înv. Sabo Doina), Borodi Denisa (Lic.Ped. „Regele Ferdind” Sighet, înv. Grigor Dorina), Diaconescu Răzvan (Şc.Gim.

5

Page 6: SOCIETATEA DE ŞTIINŢE MATEMATICE DIN - ISJ … matematica 2015... · Web viewpentru punctaj maxim la sectiunea liceu Zelina Mihai, clasa a X-a, C.N. „V. Lucaciu”, prof. Boroica

„L. Blaga”, înv. Bârsan Elisabeta), Paşca David (Şc.Gim. „L. Blaga”, înv. Zah Maria), Cadar Mihnea (Şc.Gim. „L. Blaga”, înv. Zah Maria), Naghi Alexia (L.T.A ,,Alexiu Berinde” Seini, înv. Sabo Doina), Tătaru Andrei (Şc.Gim. „N.Stănescu”, înv. Pop Luminiţa), Marinca Bogdan (Şc.Gim. „N. Iorga”, înv. Pop Carmen), Coman Andrada Ioana (Şc.Gim. Vişeu de Jos, înv. Pop Anişoara), Raţ Carmina (Şc.Gim. „N. Stănescu”, înv. Paşcaniuc Zita), David Cristian (Şc.Gim. „N. Stănescu”, înv. Paşcaniuc Zita), Pop Larisa (Şc.Gim. „D. Cantemir”, înv. Coroian Emilia), Bărbuş Vlad (L.T.A. ,,Alexiu Berinde” Seini, înv. Sabo Doina), Opriş Iuliana Andreea (Şc.Gim. „I. L. Caragiale”, înv. Roman Cristina), Gheorghiu Camelia (Şc.Gim. „I. L. Caragiale”, înv. Roman Cristina), Szmuck Patrick (Şc.Gim. Nr.18, înv. Porumb Dorina), Biriş Marc (Şc.Gim. „N.Stănescu”, înv. Pop Luminiţa), Mureşan Ana Maria (Lic.Teor. „I. Buteanu” Şomcuta Mare, înv. Pârvan Rozalia), Ilban Alessandro (Şc.Gim. „L. Rebreanu” Dragomireşti, înv. Ofrim Floare), Giurgi Bogdan (Şc.Gim. „G. Coşbuc” Sighet , înv. Mih C./Bolchis A.), Gherlan Marius (Şc.Gim. „G. Coşbuc” Sighet, înv. Mih C./Bolchis A.), Nagy Alin (Şc.Gim. „N. Iorga”, înv. Mărieş Zorica), Perta Tudor (Şc.Gim. „G. Coşbuc”, înv. Dragoş Maria), Vasiut Alexandra (Şc.Gim. „D. Cantemir”, înv. Coroian Emilia), Pop Daria (Liceul de Arte, înv. Chiorean M.), Stainer Patricia (Şc.Gim. „G. Coşbuc”, înv. Chiorean Adela), Petruţ Alexandra (Şc.Gim. „D. Cantemir”, înv. Bîrle Delia), Cormoş Cătălin (Şc.Gim. „A. Iancu”, înv. Berlinger Maria), Gaie Darius (Şc.Gim. „Al. Ioan Cuza”, înv. Barbur Maria Cicica), Mureşan Alexia (Şc.Gim. „N.Stănescu”, înv. Pop Luminiţa), Rus Tudor (Şc.Gim. „N. Iorga”, înv. Tarţa Diana/Petrean Ramona), Horga Alex (Şc.Gim. „Al. Ivasiuc”, înv. Puşcaş Teodora), Alb Alexia (Lic.Teor. „I. Buteanu” Şomcuta Mare, înv. Pârvan Rozalia), Băbuş Ştefan Andrei (L.T. „Gr.C. Moisil” Tg. Lăpuş , înv. Panait Anca), Puşcaş Darius (Şc.Gim. „N. Iorga”, înv. Mărieş Zorica), Muntean Larisa (Şc.Gim. „N. Iorga”, înv. Mărieş Zorica), Katona Daria (Şc.Gim. „N. Iorga”, înv. Mărieş Zorica), Petrar Andreea (Lic.Ped. „Regele Ferdind” Sighet, înv. Grigor Dorina), Ponoran Patricia (Șc.Gim. „G. Coşbuc”, înv. Nechita Rodica), Țol Erik Claudiu (Şc.Gim. „A. Iancu”, înv. Coza Ioana), Petric Vasile (Şc.Gim. „L. Blaga”, înv. Zah Maria), Pascu Crina Cristina (Şc.Gim. Vişeu de Jos, înv. Pop Ioan), Man Rareş (Şc.Gim. „N. Iorga”, înv. Pop Carmen), David Marian (Şc.Gim. „N. Iorga”, înv. Pop Carmen), Szebeni Amalia (Şc.Gim. „G. Coşbuc” Sighet, înv. Nemes Ileana), Giesswein Patrik (Şc.Gim. „N. Iorga”, înv. Mărieş Zorica), Talpoş Andra (Şc.Gim. „G. Coşbuc”, înv. Dragos Maria), Demeter Andrei (Şc.Gim. „G. Coşbuc” , înv. Dragoş Maria), Tar Ivett Dora (Şc.Gim. „O. Goga”, înv. Dorca Onorica), Pop Denisa (Şc.Gim. „D. Cantemir”, înv. Coroian Emilia), Popescu Andrei (Şc.Gim. „G. Coşbuc”, înv. Chiorean Adela) .Clasa a V-a:Premiul I: Zlampareţ George (C.N. „Gh. Şincai”, prof. Muşuroia Nicolae), Robu Raluca (C.N. „V. Lucaciu” , prof. Bretan Andrei), Gulin Tudor (C.N. „Gh. Şincai”, prof. Muşuroia Nicolae), Dragoş Andrea Rita (C.N. „V. Lucaciu”, prof. Covaciu Traian), Lazea Darius (C.N. „Gh. Şincai”, prof. Bojor Meda), Pop Andreea Maria (C.N. „V. Lucaciu”, prof. Bretan Andrei), Suciu Bogdan (Şc. Gim. „D. Cantemir”, prof. Hossu Călin), Pop Ştrenpel Alexandru (C.N. „Gh. Şincai”, prof. Muşuroia Nicolae), Danci Patricia (C.N. „V. Lucaciu”, prof. Bretan Andrei), Filip

6

Page 7: SOCIETATEA DE ŞTIINŢE MATEMATICE DIN - ISJ … matematica 2015... · Web viewpentru punctaj maxim la sectiunea liceu Zelina Mihai, clasa a X-a, C.N. „V. Lucaciu”, prof. Boroica

Radu ( C.N. „V. Lucaciu”, prof. Bretan Andrei), Ştirbu Silvia Maria (C.N. „V. Lucaciu”, prof. Bretan Andrei ), Vădean Cătălin (C.N. „Gh. Şincai”, prof. Bojor Meda); Premiul II: Borcuti Oana (C.N. „Gh. Şincai”, prof. Muşuroia Nicolae), Darolţi Bogdan (Şc.Gim. „N. Iorga”, prof. Boloş Mihai), Mecea Corina (Şc.Gim. „G. Coşbuc”, prof. Ienuţaş Vasile), Munteanu Vlad Marian (C.N. „V. Lucaciu”, prof. Bretan Andrei), Suciu Petru (Liceul de Arte , prof. Bunu Iulian), Tămaşan Tudor Andrei (C.N. „V. Lucaciu”, prof. Bretan Andrei), Grad Laurenţiu (Şc.Gim. Nr.9 Borşa, prof. Popescu Ana), Covrig Larisa (C.N. „D. Vodă” Sighet , prof. Bedeoan Loredana), Iliuţă Filip (C.N. „Gh. Şincai”, prof. Muşuroia Nicolae), Leo Mihai Ioan (Lic.Teor. „P. Rareş” Tg. Lăpuş, prof. Găvrilaş Emilia), Leordean Raul ( C.N. „V. Lucaciu”, prof. Bretan Andrei), Pop Ioana-Alexandra (Şc.Gim. „D. Cantemir”, prof. Szerasz Maria), Turda Maria (Şc.Gim. „G. Coşbuc”, prof. Zah Ştefan), Mujdar Milan (C.N. „D. Vodă” Sighet , prof. Bedeoan Loredana), Bragaru Maria (Liceul de Arte , prof. Bunu Iulian), Cozmuţa Tudor (Şc.Gim. „N. Iorga”, prof. Boloş Mihai), Mesaroş Radu (C.N. „V. Lucaciu”, prof. Bretan Andrei); Premiul III: Andreicuţ Elena (C.N. „Gh. Şincai”, prof. Bojor Meda), Plavoşin Vlad (C.N. „V. Lucaciu”, prof. Bretan Andrei), Şimon Iarina (C.N. „V. Lucaciu”, prof. Bretan Andrei), Ţîţoc Ioana Alexandra (Lic.Teor. „P. Rareş” Tg. Lăpuş, prof. Găvrilaş Emilia), Hotea Mihai (C.N. „D. Vodă” Sighet , prof. Bedeoan Loredana), Moldovan Mihai (C.N. „Gh. Şincai”, prof. Muşuroia Nicolae), Vince Denisa (C.N. „V. Lucaciu”, prof. Bretan Andrei), Vrînceanu Flaviu (C.N. „Gh. Şincai”, prof. Bojor Meda), Korponay Albert (Lic.Teor. „N. Laszlo”, prof. Mastan Eliza), Gardu Ionuţ (Şc.Gim. „L. Blaga”, prof. Nagy Anamaria), Hosu Iulia (C.N. „D. Vodă” Sighet , prof. Bedeoan Loredana), Iştovan Irina (Şc.Gim. Recea, prof. Mureşan Corina), Pop Cristina Maria (Şc.Gim. „Al. I. Cuza”, prof. Caltea Amalia), Şeu David (Şc.Gim. „G. Coşbuc”, prof. Ienuţaş Vasile), Medeşan Tudor (C.N. „Gh. Şincai”, prof. Bojor Meda), Vâță Alexandra (C.N.„Gh. Şincai”, prof. Bojor Meda), Zubaşcu Adina (Şc.Gim. „L. Rebreanu” Dragomireşti, prof. Zubaşcu Daniela), Brenner Vanessa (Şc.Gim. „G. Coşbuc” Sighet , prof. Zetea Bogdan), Lupu George (Liceul Borşa, prof. Mihali Marinela), Paven Andreea Ioana (Lic.Teor. „P. Rareş” Tg. Lăpuş , prof. Găvrilaş Emilia), Petrescu Oana (C.N. „Gh. Şincai”, prof. Bojor Meda), Suru Lia (C.N. „Gh. Şincai”, prof. Bojor Meda), Trif Cristina (C.N. „V. Lucaciu”, prof. Covaciu Traian), Verdeş Vlad (C.N. „Gh. Şincai”, prof. Bojor Meda).Clasa a VI-a: Premiul I: Ciceu Denis (Şc.Gim. „N. Iorga”, prof. Bretan Andrei), Zaharie Oana (Şc.Gim. „N. Iorga”, prof. Bretan Andrei), Talpoş Carina (Şc.Gim. „N. Iorga”, prof. Bretan Andrei), Turda Raul (C.N. „Gh. Şincai”, prof. Boroica Gheorghe), Pop Raul (Şc.Gim. Vişeu de Jos , prof. Tomoiagă Ioan), Mariş Cătălin (Liceul Borşa, prof. Mihali Marinela), Treista Ioana (C.N. „D. Vodă” Sighet, prof. Bedeoan Loredana), Lazăr Laurenţiu (C.N. „D. Vodă” Sighet, prof. Bedeoan Loredana), Săsăran Tania (C.N. „V. Lucaciu”, prof. Sabău Ştefan); Premiul II: Doia Bogdan-Mihai (Şc.Gim. „D. Cantemir”, prof. Szerasz Maria), Riglea Teodora (C.N. „Gh. Şincai”, prof. Heuberger Dana), Maxim Sonia (C.N. „Gh. Şincai”, prof. Boroica Gheorghe), Mihalca Grigore (Şc.Gim. Vişeu de Jos , prof. Tomoiagă Ioan), Andreica

7

Page 8: SOCIETATEA DE ŞTIINŢE MATEMATICE DIN - ISJ … matematica 2015... · Web viewpentru punctaj maxim la sectiunea liceu Zelina Mihai, clasa a X-a, C.N. „V. Lucaciu”, prof. Boroica

Paola (Şc.Gim. nr.7 Vişeu , prof. Stan Sînziana), Cionte Sergiu (C.N. „Gh. Şincai”, prof. Boroica Gheorghe), Vanciu Daria (Şc.Gim. „N. Iorga”, prof. Bretan Andrei), Ardelean Alexandru (Şc.Gim. „N. Iorga”, prof. Boloş Mihai), Dănilă Alexandru (Şc.Gim. „N. Iorga”, prof. Bretan Andrei), Ghişe Teodora (Liceul de Arte , prof. Bunu Iulian), Herţeg Ioana (C.N. „V. Lucaciu”, prof. Sabău Ştefan), Stan Teodora (C.N. „V. Lucaciu, prof. Sabău Ştefan); Premiul III: Scoropan Cristian (Şc.Gim. „N. Iorga , prof. Bretan Andrei), Afrăsinei Cătălin (C.N. „Gh. Şincai” , prof. Heuberger Dana), Ilieş Bogdan (Şc.Gim. „G. Coşbuc” Sighet , prof. Zetea Bogdan), Lendel Andreea (C.N. „Gh. Şincai”, prof. Boroica Gheorghe), Paşca Teodora (L.T. Vişeu, prof. Paşca Raluca), Urda Denisa (Liceul de Arte , prof. Bunu Iulian), Biriş Iasmina (C.N. „Gh. Şincai”, prof. Boroica Gheorghe), Herzal Radu (C.N. „Gh. Şincai”, prof. Heuberger Dana), Mercea Cezar (Şc.Gim. „N. Iorga”, prof. Bretan Andrei), Corneştean Andreea (Lic.Ped. „Regele Ferdinand” Sighet , prof. Puţ Liliana), Feier Andrei (Şc.Gim. „Dr.Ilie Lazar” Giuleşti, prof. Rednic Gheorghe), Ivanciuc Dragoş Vasile (C.N. „D. Vodă” Sighet, prof. Bedeoan Loredana), Onea Andrei (Şc.Gim. „L. Blaga”, prof. Keller Otto), Rad Vlăduţ (Şc.Gim. „G. Coşbuc” Sighet , prof. Zetea Bogdan), Şuba Giulia (Şc.Gim. „I. L. Caragiale”, prof. Ionescu Sorin), Biriş Erik (C.N. „Gh. Şincai”, prof. Heuberger Dana), Dragoş Monica (Şc.Gim. „N. Iorga”, prof. Boloş Mihai), Filip Georgiana (Şc.Gim. „Al. Ivasiuc”), Giuroiu Tudor (C.N. „Gh. Şincai”, prof. Boroica Gheorghe), Marina-Fiţ Paul (C.N. „D. Vodă” Sighet , prof. Bedeoan Loredana), Tătaru Mihai (C.N. „Gh. Şincai”, prof. Heuberger Dana), Ţicală Andreea Irina (Şc.Gim. „M. Eminescu” Săliştea de Sus , prof. Vlad Marie).Clasa a VII-a: Premiul I: Robu Vlad (Şc.Gim. „N. Iorga”, prof. Bretan Andrei), Pop Călin (Şc.Gim. „N. Iorga”, prof. Bretan Andrei), Boroica Adrian (C.N. „Gh. Şincai”, prof. Bojor Florin), Becsi Paul (C.N. „Gh. Şincai”, prof. Bojor Florin); Premiul II: Andreicuţ Teofil (C.N.”Gh. Şincai”, prof. Bojor Florin), Moldovan Nicolae (Şc.Gim. „G. Coşbuc”, prof. Pop Cosmin), Ilieş Iulia (C.N.”Gh. Şincai”, prof. Bojor Florin), Onea Vlad (Şc.Gim. „N. Iorga”, prof. Bretan Andrei), Francioli Daria (C.N. „V. Lucaciu”, prof. Sfara Gheorghe), Ţiplea Ştefan (C.N. „D. Vodă” Sighet , prof. Giurgi Vasile), Gaborean Vasile (C.N. „D. Vodă” Sighet , prof. Giurgi Vasile), Dunca Alina (Şc.Gim. „N. Iorga”, prof. Bretan Andrei); Premiul III: Ciocan Florin (Şc.Gim. „L. Blaga”, prof. Nagy Anamaria), Mociran Eduard (Şc.Gim. „G. Coşbuc”, prof. Pop Cosmin), Corneştean Loretta (C.N. „D. Vodă” Sighet, prof. Giurgi Vasile), Giesswein Alexia (C.N. „V. Lucaciu”, prof. Boroica Gabriela), Spaczai Carla Noemi (C.N. „D. Vodă” Sighet , prof. Giurgi Vasile), Breban Alexandru (Şc.Gim. „N.Stănescu”, prof. Breitkopf Marieta), Chiuzbăian Rareş (Şc.Gim. „A. Iancu”, prof. Rotaru Dumitru), Curac Mihai (Şc.Gim. „N. Iorga”, prof. Bretan Andrei), Zubaşcu Maria (Şc.Gim. „L. Rebreanu” Dragomireşti, prof. Zubaşcu Daniela), Mihăilescu Dara (C.N. „D. Vodă” Sighet , prof. Giurgi Vasile), Tivadar Maria Simona (C.N. „D. Vodă” Sighet , prof. Giurgi Vasile)Clasa a VIII-a: Premiul I: Matei Bledea Alexandru (C.N. „Gh. Şincai”, prof. Muşuroia Nicolae), Zelina Paul (C.N. „V. Lucaciu”, prof. Boroica Gabriela), Mureşan Ioan (C.N. „Gh. Şincai”, prof. Muşuroia Nicolae); Premiul II: Stepan Dacian

8

Page 9: SOCIETATEA DE ŞTIINŢE MATEMATICE DIN - ISJ … matematica 2015... · Web viewpentru punctaj maxim la sectiunea liceu Zelina Mihai, clasa a X-a, C.N. „V. Lucaciu”, prof. Boroica

(Şc.Gim. „G. Coşbuc” Sighet , prof. Zetea Bogdan), Mercea Ioana (C.N. „Gh. Şincai”, prof. Muşuroia Nicolae), Cotârlan Codrin (C.N. „D. Vodă” Sighet , prof. Bedeoan Loredana), Tămîian Rareş (C.N. „Gh. Şincai”, prof. Muşuroia Nicolae), Ghişa Remus (C.N. „Gh. Şincai”, prof. Muşuroia Nicolae), Coman Ilişca Anişoara (Şc.Gim. Vişeu de jos , prof. Tomoiagă Ioan), Diaconescu Mălina (C.N. „V. Lucaciu”, prof. Boroica Gabriela), Mardar Paul (C.N. „Gh. Şincai”, prof. Muşuroia Nicolae), Petz Alin (C.N. „V. Lucaciu”, prof. Boroica Gabriela), Şuştic Alexandra (Liceul Borşa, prof. Mihali Marinela), Dicu Dan Alexandru (C.N. „D. Vodă” Sighet , prof. Bedeoan Loredana).Premiul III: Buzilă-Gârda Andra (Şc.Gim. „G. Coşbuc”, prof. Zah Ştefan), Mih Mara (Şc.Gim. „G. Coşbuc” Sighet , prof. Zetea Bogdan), Toma Antonio (Şc.Gim. „L. Blaga”, prof. Nagy Anamaria), Ionuţi Bogdan (Şc.Gim. „N. Iorga”, prof. Bretan Andrei), Lemian Diana (Şc.Gim. „S. Bărnuţiu”, prof. Cadar Maria), Andercău Florina (Şc.Gim. „V. Alecsandri” , prof. Pop Sever), Bîrle Silviu-Adrian (Şc.Gim. „D. Cantemir”, prof. Hossu Călin), Bledea Dragoş Ioan (C.N. „D. Vodă” Sighet , prof. Bedeoan Loredana), Bordeianu Lucia (Şc.Gim. „L. Blaga”, prof. Nagy Anamaria), Hodor Alexandru (Şc.Gim. „G. Coşbuc” Sighet , prof. Zetea Bogdan), Vale Bogdan (Şc.Gim. „I. L. Caragiale”, prof. Ionescu Sorin), Ardelean Ariana (Şc.Gim. „L. Blaga”, prof. Nagy Anamaria), Băban Diana (Şc.Gim. „G. Coşbuc”, prof. Zah Ştefan), Koblicica Andrei (C.N. „V. Lucaciu”, prof. Boroica Gabriela), Petrovan Claudia (Şc.Gim. „G. Coşbuc” Sighet , prof. Zetea Bogdan), Şuteu Ionuţ (Şc.Gim. „O. Goga”, prof. Popovic Ioana), Tomoiagă Radu (Şc.Gim. „G. Coşbuc” Sighet , prof. Zetea Bogdan), Uţă Adrian (Şc.Gim. „Al. I. Cuza”, prof. Caltea Amalia), Varady Iulia (Şc.Gim. „N. Iorga”, prof. Bretan Andrei).Clasa a IX-a: Premiul I: Lucaciu Sergiu (C.N. „Gh. Şincai”, prof. Heuberger Cristian), Pop Vlad (C.N. „Gh. Şincai”, prof. Heuberger Cristian), Hagău Iulian (C.N. „Gh. Şincai”, prof. Heuberger Dana); Premiul II: Bojor Barbu (C.N. „Gh. Şincai”, prof. Heuberger Cristian), Mărieş Maria (C.N. „Gh. Şincai”, prof. Heuberger Cristian), Berinde Thomas (C.N. „Gh. Şincai”, prof. Boroica Gheorghe), Tămâian Andrei (C.N. „Gh. Şincai”, prof. Petruţiu Crina); Premiul III: Neţa Răzvan (C.N. „Gh. Şincai”, prof. Heuberger Cristian), Cudrici Carina (C.N. „D.Vodă”, prof. Giurgi Vasile), Palca Mihaela (C.N. „Gh. Şincai”, prof. Heuberger Cristian), Darolţi Larisa (C.N. „V. Lucaciu”, prof. Darolţi Erika), Filip Marian (Lic.Teor. „P. Rareş”, prof. Miholca Gavril), Ilieş Dragoş (C.N. „D. Vodă”, prof. Giurgi Vasile).Clasa a X-a: Premiul I: Zelina Mihai (C.N. „V. Lucaciu”, prof. Boroica Gabriela); Premiul II: Ilieş Andreea (C.N. „D. Vodă”, prof. Giurgi Vasile); Premiul III: Sântejudean Tudor (C.N. „Gh. Şincai”, prof. Bojor Florin), Vraja Iulian (C.N. „D. Vodă”, prof. Giurgi Vasile), Chişcă Andrei (C.N. „Gh. Şincai”, prof. Bojor Florin), Ionas Emanuela (Lic.Teor. „P. Rareş”, prof. Ionas Mirela), Iosif Andrei (C.N. „Gh. Şincai”, prof. Bojor Florin).Clasa a XI-a: Premiul I: Butnar Adrian (C.N. „Gh. Şincai”, prof. Muşuroia Nicolae); Premiul II: Cotan Paul (C.N. „Gh. Şincai”, prof. Heuberger Dana),Ţînţar Oana (C.N. „Gh. Şincai”, prof. Muşuroia Nicolae), Pop Darius (C.N. „D. Vodă”, prof. Giurgi Vasile), Păcurar Aralda (C.N. „D. Vodă”, prof. Giurgi Vasile), Buftea Mădălina (C.N.

9

Page 10: SOCIETATEA DE ŞTIINŢE MATEMATICE DIN - ISJ … matematica 2015... · Web viewpentru punctaj maxim la sectiunea liceu Zelina Mihai, clasa a X-a, C.N. „V. Lucaciu”, prof. Boroica

„D.Vodă”, prof. Giurgi Vasile), Danci Bianca (C.N. „D. Vodă”, prof. Giurgi Vasile), Tyekar Dan (C.N. „V. Lucaciu”, prof. Zlampareţ Horia).Clasa a XII-a: Premiul I: Gotha Guntter (Lic.Teor. „N. Laszlo”, prof. Longaver Ludovic), Bud Cristian (C.N. „Gh. Şincai”, prof. Heuberger Dana); Premiul II: Ciurte Tudor (C.N. „Gh. Şincai”, prof. Boroica Gheorghe); Premiul III: Şimon Gheorghe (Lic.Teor. „B. Vodă” Vişeu, prof. Pop Vesel Floare).

OLIMPIADA DE MATEMATICĂ - etapa națională ediția 2015

La etapa naţională a Olimpiadei de Matematică, judeţul Maramureş a avut 7

participanţi la clasele V-VI şi 13 participanţi la clasele VII-XII. Ca urmare a rezultatelor bune obţinute an de an, judeţul Maramureş a avut 2 membri în comisia centrală: prof. Andrei Bretan la clasele V-VI şi prof. univ. dr. Vasile Berinde la clasele VII-XII. Din cei 20 participanţi, 17 au obţinut premii, menţiuni şi medalii ale SSMR. Aceste rezultate sunt rodul activităţii deosebite desfăşurate în cele 43 grupe de excelenţă şi în cele 3 grupe de super-excelenţă şi a implicării familiilor acestor elevi în deplasarea la diverse concursuri interjudeţene şi regionale. Modul de selecţie serios şi responsabil realizat la etapa locală şi judeţeană a dus la o reprezentare bună la etapa naţională. Felicitări se cuvin profesorilor de excepţie pe care îi are şcoala de matematică din Maramureş şi elevilor care pe lângă vocaţia pe care o au depun eforturi susţinute pentru a reprezenta cu cinste şcoala de provenienţă şi judeţul din care face parte.

Vă prezentăm lista elevilor şi a premiilor: Zlampareţ George, clasa a V-a, Colegiul Naţional „Gh. Şincai” Baia Mare,

profesor Muşuroaia Nicolae, medalie de argint SSMR; Robu Raluca Ioana, clasa a V-a, Colegiul Naţional „V. Lucaciu” Baia Mare,

profesor Bretan Andrei, participare; Gulin Tudor, clasa a V-a, Colegiul Naţional „Gh. Şincai” Baia Mare, profesor

Muşuroaia Nicolae, participare; Dragoş Andreea-Rita, clasa a V-a, Colegiul Naţional „V. Lucaciu” Baia Mare,

profesor Covaciu Traian, medalie de bronz SSMR; Talpoş Carina, clasa a VI-a, Şcoala Gimnazială „N. Iorga” Baia Mare, profesor

Bretan Andrei, medalie de argint SSMR; Ciceu Denis, clasa a VI-a, Şcoala Gimnazială „N. Iorga” Baia Mare, profesor

Bretan Andrei, medalie de bronz SSMR; Zaharie Oana, clasa a VI-a, Şcoala Gimnazială „N. Iorga” Baia Mare, profesor

Bretan Andrei, medalie de bronz SSMR; Robu Vlad, clasa a VII-a, Şcoala Gimnazială „N. Iorga”, Baia Mare profesor

Bretan Andrei, Premiul II MECS şi Medalie aur SSMR; Pop Călin, clasa a VII-a, Şcoala Gimnazială „N. Iorga”, Baia Mare profesor

Bretan Andrei, Menţiune MECS şi Medalie aur SSMR;

10

Page 11: SOCIETATEA DE ŞTIINŢE MATEMATICE DIN - ISJ … matematica 2015... · Web viewpentru punctaj maxim la sectiunea liceu Zelina Mihai, clasa a X-a, C.N. „V. Lucaciu”, prof. Boroica

Boroica Adrian, clasa a VII-a, Colegiul Naţional „Gh. Şincai” Baia Mare, profesor Bojor Florin, Medalie argint SSMR;

Becsi Paul, clasa a VII-a, Colegiul Naţional „Gh. Şincai” Baia Mare, profesor Bojor Florin, Medalie argint SSMR;

Matei Bledea Alexandru, clasa a VIII-a, Colegiul Naţional „Gh. Şincai” Baia Mare, profesor Muşuroaia Nicolae, Menţiune MECS şi Medalie aur SSMR;

Zelina Paul, clasa a VIII-a, Colegiul Naţional „V. Lucaciu” Baia Mare, profesor Boroica Gabriela, Medalie bronz SSMR;

Lucaciu Sergiu, clasa a IX-a, Colegiul Naţional „Gh. Şincai” Baia Mare, profesor Heuberger Cristian, Medalie bronz SSMR;

Hagău Iulian, clasa a IX-a, Colegiul Naţional „Gh. Şincai” Baia Mare, profesor Heuberger Dana, participare;

Zelina Mihai Andrei, clasa a X-a, Colegiul Naţional „V. Lucaciu” Baia Mare, profesor Boroica Gabriela, Medalie argint SSMR;

Ilieş Andreea, clasa a X-a, Colegiul Naţional „D. Vodă”, Sighetu Marmaţiei, profesor Giurgi Vasile, Medalie bronz SSMR;

Butnar Adrian, clasa a XI-a, Colegiul Naţional „Gh. Şincai” Baia Mare, profesor Muşuroaia Nicolae, Medalie bronz SSMR;

Bud Cristian, clasa a XII-a, Colegiul Naţional „Gh. Şincai” Baia Mare, profesor Heuberger Dana, Menţiunea MECS şi Medalie argint SSMR;

Gotha Guntter, clasa a XII-a, Liceul Teoretic „N. Laszlo” Baia Mare, profesor Longaver Ludovic, Medalie bronz SSMR.Judeţul Maramureş a avut 8 participanţi la barajul pentru formarea celor 2 loturi

lărgite, de juniori şi seniori în vederea participării la concursurile şi olimpiadele internaţionale: Robu Vlad, Pop Călin, Boroica Adrian, Becsi Paul, Matei Bledea Alexandru, Zelina Paul, Zelina Mihai Andrei, Bud Cristian. Dintre aceştia s-au calificat : Robu Vlad şi Pop Călin, clasa a VII-a, Şcoala Gimnazială „N. Iorga”, Baia Mare, profesor Bretan Andrei, fac parte din lotul lărgit de juniori şi Zelina Mihai, clasa a X-a, Colegiul Naţional „V. Lucaciu” Baia Mare, profesor Boroica Gabriela, a făcut parte din lotul lărgit de seniori al României. Robu Vlad s-a calificat în lotul restrâns al României la disciplina matematică, ocupând locul 7 la finele barajului.

CONCURSUL DEMATEMATIC Ă APLICATĂ „ADOLF HAIMOVICI” etapa judeţeană- 2015

Clasa a IX-a: Premiul I: Morar Teofana, filiera teoretică, profil umanist, C.N. „ M. Eminescu” Baia Mare; Horţ Iulia, filiera teoretică, profil real, C.N.„ Gh. Şincai” Baia Mare; Rus Salomon Timea, filiera tehnologică, profil servicii, C. Ec. „N. Titulescu” Baia Mare; Premiul II: Coman Raluca, filiera teoretică, profil umanist, C.N. „ M. Eminescu” Baia Mare; Szabo Erik, filiera teoretică, profil umanist, C.N. „M. Eminescu” Baia Mare; Buciuta Daniel, filiera teoretică, profil real, C.N. „D. Vodă” Sighetu Marmaţiei; Ilieş Andrea, profil servicii, C.Ec. „N. Titulescu” Baia Mare;

11

Page 12: SOCIETATEA DE ŞTIINŢE MATEMATICE DIN - ISJ … matematica 2015... · Web viewpentru punctaj maxim la sectiunea liceu Zelina Mihai, clasa a X-a, C.N. „V. Lucaciu”, prof. Boroica

Premiul III: Kiss Dana, filiera teoretică, profil umanist, C.N. „Gh. Şincai” Baia Mare; Ungur David Antonio,filiera tehnologică, profil servicii, C.Ec. „P.Viteazul” Cavnic; Mureşan Andrea, filiera tehnologică, profil servicii, C.Ec. „N. Titulescu”, Baia Mare; Pintea Alexandru Ionuţ, filiera tehnologică, profil tehnic , C.T. „G. Bariţiu” Baia MareClasa a X-a: Premiul I: Moldovan Diana, filiera teoretică, profil umanist, C.N. „M. Eminescu” Baia Mare; Danci Ştefania, filiera teoretică, profil real, C.N. „Gh. Şincai” Baia Mare; Marchiş Alexandru, filiera tehnologică, profil servicii, C.Ec. „N. Titulescu” Baia Mare; Nastai Sorina, filiera tehnologică, profil servicii, C.Ec. „N. Titulescu” Baia Mare; Premiul II: Tarba Florina Bianca, filiera teoretică, profil umanist, Colegiul Naţional „Gh. Şincai” Baia Mare; Dala Vlad, filiera teoretică, profil real, Colegiul Naţional „Gh. Şincai” Baia Mare; Tohătan Adrian, filiera tehnologică, profil servicii, Colegiul Economic „N. Titulescu” Baia Mare; Premiul III: Guţ Simina, filiera tehnologică, profil servicii, C.Ec. „N. Titulescu” Baia Mare; Ilieş Andrei, filiera teoretică, profil real, C.N. „Gh. Şincai” Baia MareClasa a XI-a: Premiul I: Kanya Iulia Alexandra, filiera teoretică, profil umanist, C.N. „Gh. Şincai” Baia Mare; Bonaţ Raluca, filiera teoretică, profil real, C.N. „V. Lucaciu” Baia Mare; Herczeg Adrian, filiera tehnologică, profil servicii, C.Ec. „N. Titulescu” Baia Mare; Baiaş Dora, filiera tehnologică, profil tehnic, Liceul de Arte Baia Mare; Premiul II: Mureşan Tudor, filiera teoretică, profil umanist, C.N. „Gh. Şincai” Baia Mare; Pop Andreea, filiera teoretică, profil umanist, Lic.Teor. „P. Rareş” Târgu Lăpuş; Balin Bianca, filiera teoretică, profil real, C.N. „V. Lucaciu” Baia Mare; Ianoş Simona Aurelia, filiera tehnologică, profil servicii, Liceul Teologic Penticostal Baia Mare; Micaş Vasile, filiera tehnologică, profil tehnic, C.T. „A. Saligny” Baia Mare; Morar Roxana, filiera tehnologică, profil tehnic, Liceul de Arte Baia Mare; Premiul III: Balogh Andreea, filiera teoretică, profil umanist, C.N. „M. Eminescu” Baia Mare; Sztrelenczuk Krista, , filiera teoretică, profil real, C.N. „Gh. Şincai” Baia Mare; Cărăjan Răzvan, filiera tehnologică, profil servicii, C.Ec. „N. Titulescu” Baia Mare; Albiciuc Octaviu Andrei, filiera tehnologică, profil tehnic, L.T. Marmaţia Sighetu Marmaţiei.Clasa a XII-a: Premiul I: Pop Mădălina, filiera teoretică, profil umanist, C.N. „M. Eminescu” Baia Mare; Hanţig Ştefan Cristian, filiera teoretică, profil real, Lic.Teor. „E. Racoviţă” Baia Mare; Bora Ioan Florin, filiera tehnologică, profil servicii, L.T.A. „A. Berinde” Seini; Pop Darius, filiera tehnologică, profil servicii, C.Ec. „N. Titulescu” Baia Mare; Chiş Ioan, filiera tehnologică, profil tehnic, C.T. „G. Bariţiu” Baia Mare; Premiul II: Tămăşan David Florin, filiera teoretică, profil uman, Liceul Teologic Penticostal Baia Mare; Băbuţiu Mădălina, filiera teoretică, profil real, C.N. „Gh. Şincai” Baia Mare; Borca Vlad, filiera teoretică, profil real, C.N. „Gh. Şincai” Baia Mare; Hereş Debora Lucica Flavia, filiera tehnologică, profil servicii, Liceul Teologic Penticostal Baia Mare; Şter Paula Andreea Elena, filiera tehnologică, profil tehnic, C.T. „G. Bariţiu” Baia Mare; Premiul III: Gheţie Ioana, filiera teoretică, profil real, Lic.Teor. „E. Racoviţă” Baia Mare; Bora Vlad David, filiera tehnologică, profil servicii, L.T.A. „A. Berinde”, Seini; Burdeţ Ioana Florina, filiera tehnologică, profil servicii, Lic.Teor. „B. Vodă” Vişeu; Ciorban Paula, filiera tehnologică, profil servicii,

12

Page 13: SOCIETATEA DE ŞTIINŢE MATEMATICE DIN - ISJ … matematica 2015... · Web viewpentru punctaj maxim la sectiunea liceu Zelina Mihai, clasa a X-a, C.N. „V. Lucaciu”, prof. Boroica

C.Ec. „ N. Titulescu” Baia Mare; Molnar Silviu, filiera tehnologică, profil tehnic, C.T. „A. Saligny” Baia Mare

CONCURSUL DE MATEMATICĂ APLICATĂ „ADOLF HAIMOVICI” – etapa naţională

1-3 mai 2015

Concursul de Matematică Aplicată „Adolf Haimovici” se adresează elevilor din grupurile şcolare, filiera tehnologică: profil tehnic, filiera tehnologică, profil servicii şi resurse naturale, respectiv protecţia mediului, precum şi liceelor teoretice cu profil real, specializarea ştiinţele naturii, profil umanist, specializarea filologie (clasele IX-X) alături de specializarea ştiinţe sociale (clasele IX-XII). Problemele date la concurs au pus în evidenţă atât cunoştinţele stricte de matematică ale concurenţilor, cât şi creativitatea şi spiritul practic al acestora, iar gradul de dificultate al acestora a variat de la mediu la ridicat.

Anul acesta a avut loc ediţia a XIX-a a concursului la Iaşi în perioada 1-3 mai 2015. Au participat 38 de judeţe. Judeţul Maramureş a avut un lot format din 18 elevi care au fost însoţiţi de doamna profesoară Darolţi Erika de la Colegiul Naţional „V. Lucaciu”; 12 elevi au obţinut premii şi menţiuni: Premiul II: Rus Salamon Timea, clasa a IX-a, de la C.Ec. „N.Titulescu”, profesor Horje Daniel; Premiul III: Hanţig Ştefan Cristian, clasa a XII-a, de la Lic.Teor. „E. Racoviţă”, profesor Podină Camelia; Menţiuni: Horţ Iulia Alexandra, clasa a IX-a, de la C.N. „Gh. Şincai”, profesor Bojor Meda; Marchiş Alexandru, clasa a X-a, de la C.Ec. „N.Titulescu”, profesor Zlampareţ Mihaela; Nastai Sorina Ana Maria, clasa a X-a, de la C.Ec. „N.Titulescu”, profesor Friedrich Gabriela; Danci Ştefania, clasa a X-a, de la C.N. „Gh. Şincai”, profesor Petruţiu Crina; Dala Vlad Radu, clasa a X-a, de la C.N. „Gh.Şincai”, profesor Petruţiu Crina; Moldovanu Diana Maria, clasa a X-a, de la C.N. „M. Eminescu”, profesor Crăciun Marius; Herczeg Adrian Zoltan, clasa a XI-a, de la C.Ec. „N.Titulescu”, profesor Zlampareţ Mihaela; Kanya Iulia Alexandra, clasa a XI-a, de la C.N. „Gh. Şincai”, profesor Pop Adrian; Pop Darius Cătălin, clasa a XII-a, C.Ec. „N. Titulescu”, profesor Zlampareţ Mihaela; Chiş Ioan, clasa a XII-a, de la C.T. „G. Bariţiu”, profesor Pop Anca; Diplomă de participare: Bora Ioan Florin, clasa a XII-a, de la L.T.A. „A. Berinde” Seini, profesor Berciu Ioan; Baias Dora, clasa a XI-a, de la Liceul de Arte, profesor Bunu Iulian; Bonaţ Raluca, clasa a XI-a, de la C.N. „V.Lucaciu”, profesor Darolţi Erika; Morar Teofana, clasa a IX-a, de la C.N. „M. Eminescu”, profesor Ciolte Daniela; Pop Mădălina, clasa a XII-a, de la C.N. „M. Eminescu”, profesor Ciolte Daniela; Mureşan Tudor, clasa a XI-a, de la C.N. „Gh. Şincai”, profesor Pop Adrian.

TABĂRA JUDEŢEANĂ DE MATEMATICĂ A ELEVILOREtapa a XVII-a

Organizatori: Filiala MM a SSMR, Centrul Judeţean pentru Tineri Capabili de Performanţă;

13

Page 14: SOCIETATEA DE ŞTIINŢE MATEMATICE DIN - ISJ … matematica 2015... · Web viewpentru punctaj maxim la sectiunea liceu Zelina Mihai, clasa a X-a, C.N. „V. Lucaciu”, prof. Boroica

Loc de desfăşurare: Şcoala Gimnazială „G. Coşbuc”, pentru clasele IV-VII; Colegiul Naţional „Gh. Şincai” pentru clasele VIII-XII;Coordonatori: prof. univ. dr. Berinde Vasile, U.T. Cluj-Napoca, Centrul Univ. Nord; prof. Maiorescu Gheorghe,vicepreședinte Filiala MM a SSMR; Director de tabără: prof. Ienuţaş Vasile, prof. Muşuroia Nicolae;Profesori participanţi: Barbur Simona, Birta Adriana, Brisc Viorica, Bojor Florin, Bojor Meda, Boloş Mihai, Boroica Gabriela, Boroica Gheorghe, Bretan Andrei, Bunu Iulian, Cadar Maria, Caltea Amalia, Chende Paula, Cioclu Costel, Coroian Emilia, Coza Ioana, Darolţi Erika, Erdei Mariana, Fănățan Nelu, Fărcaş Natalia, Friedrich Gabriela, Heuberger Cristian, Heuberger Daniela, Horge Daniel, Horvat Marc Andrei, Hossu Călin, Husti Florentina, Kalisch Maria, Ienuţaş Monica, Ienuţaş Vasile, Ionescu Sorin, Longaver Ludovic, Lopată Angela, Muşuroia Nicolae, Nagy Anamaria, Neaga Nadina, Petruţiu Crina, Podina Camelia, Polgar Corina, Pop Adela, Pop Adrian, Pop Anca , Pop Andreea, Pop Cosmin, Pop Radu, Pop Sever, Popovic Ioana, Râmbu Gheorghe, Rotaru Dumitru, Schweichoffer Clara, Sfara Gheorghe, Stark Andreea, Ştiru Aurica, Szerasz Maria, Şerba Lucia, Tărău Rodica, Tomșa Magdalena, Vlad Vasile, Zah Ştefan, Zlampareţ Horia, Zlampareţ Mihaela.PREMII:Clasa a IV-a: Premiul de excelenţă: Muntean Tudor (Şc.Gim. „N. Iorga”); Premiul I: Varga Andrei (Şc.Gim. „N. Iorga”); Pruna Stefana (Şc.Gim. „N. Iorga”); Miclăuş Carmen (Şc.Gim. „D. Cantemir”); Biriş Marc (Șc.Gim. „N. Stănescu”); Onea Iulian (Şc.Gim. „N. Iorga”); Pop Larisa (Șc.Gim. „D. Cantemir”); Pop-Săvan Raul (Şc.Gim. „O. Goga”); Tuş Traian (Șc.Gim. „N. Stănescu”); David Marian (Șc.Gim. „N. Iorga”); Ianoş Andreea (Șc.Gim. „N. Stănescu”); Premiul II: Sava Rareş (Șc.Gim. „N. Stănescu”); Takacs Andra (Șc.Gim. „D. Cantemir”); Arduszădan Darius (Șc.Gim. „N. Stănescu”); Deksuc Timea (Șc.Gim. „O. Goga”); Indrecan Călin (Șc.Gim. „D. Cantemir”); Pop Rahela (Șc.Gim. „D. Cantemir”); Şpan Mihai (Șc.Gim. „N. Iorga”); Tibil Oana (Şc.Gim. „G. Coşbuc”); Tiut Cristian (Şc.Gim. „G. Coşbuc”); Ariciu Andra (Șc.Gim. „D. Cantemir”); Luţaş Tudor (Șc.Gim. „G. Coșbuc „); Bărăian Alexia (Șc.Gim. „N. Stănescu”); Gheţe Ruxandra(Şc.Gim. „A. Iancu”); Joo Tamaş (Şc.Gim. Lăpuşel); Moruţan Maria (Şc.Gim. „L. Blaga”); Nodiş Teodora (Șc.Gim. „O. Goga”); Petruţ Alexandra (Șc.Gim. „D. Cantemir”); Tar Ivett (Șc.Gim. „O. Goga”); Cosma Andrei (Șc.Gim. „N. Stănescu”); Premiul III: Ocian Oana (Șc.Gim. „D. Cantemir”); Tamâian Răzvan (Șc.Gim. „N. Iorga”); Tătaru Andrei (Șc.Gim. „N. Stănescu”); Ţiclea Andrei (Şc.Gim. „S. Bărnuţiu”); Trandafirescu Alexa (Șc.Gim. „D. Cantemir”); Ţol Erik, (Şc.Gimn. „A. Iancu”); Varga Renata (Şc.Gim. „G. Coşbuc”); Balint Alex (Şc.Gim. „D. Cantemir”); Matei Alice (Șc.Gim. „D. Cantemir”); Opriş Iuliana (Şc.Gim. „I. L. Caragiale”); Rogozsan Robert (Şc.Gim. „I. L. Caragiale”); Vasiuţ Alexandra (Șc.Gim. „D. Cantemir”); Cardoş Alexandra (Şc. Gim. „Al. Ivasiuc”); Lupan Alexandra(Şc.Gim. „M. Sadoveanu”); Gheorghiu Camelia (Şc.Gim. „I. L. Caragiale”); Lupşe Iasmina (Șc.Gim. „D. Cantemir”); Săsăran Noris (Şc.Gim. „I. L. Caragiale”); Seregi Norber (Șc.Gim. „D. Cantemir”); Şerban Raul (Șc.Gim. „N. Iorga”); Pop Tudor (Şc.Gim. „D. Cantemir”); Buda Larisa (Şc.Gim. „I. L. Caragiale”);

14

Page 15: SOCIETATEA DE ŞTIINŢE MATEMATICE DIN - ISJ … matematica 2015... · Web viewpentru punctaj maxim la sectiunea liceu Zelina Mihai, clasa a X-a, C.N. „V. Lucaciu”, prof. Boroica

Moldovan Ruxandra (Şc.Gim. „G. Coşbuc”); Sălăjan Mircea (Șc.Gim. „N. Iorga”) ; Costin Andrei (Şc.Gim. Dumbrăviţa); Vari Ioana (Şc.Gim. Dumbrăviţa); Coman David (Șc.Gim. „D. Cantemir”); Coroian David (Șc.Gim. „D. Cantemir”); Felleg Kristina (Şc.Gimn. „N. Stănescu”); Puşcaş Darius (Șc.Gim. „N. Iorga”); Marinca Bogdan (Șc.Gim. „N. Iorga”)Clasa a V-a: Premiul de excelenţă: Dragoş Andreea (C.N. „V. Lucaciu”); Koblicica Vlad (C.N. „V. Lucaciu”); Lazea Darius (C.N. „Gh. Şincai”); Premiul I: Leordean Raul (C.N. „V. Lucaciu”); Mecea Corina (Şc.Gim. „G. Coşbuc”); Pop Andreea (C.N. „V. Lucaciu”); Robu Raluca (C.N. „V. Lucaciu”); Rus George (Şc.Gim. „N. Sadoveanu”); Zlampareţ George (C.N. „Gh. Şincai”), Gulin Tudor (C.N. „Gh. Şincai”); Suciu Petru (Liceul de Arte); Popa Leona (Şc. Gim. „Dr. V. Babeş”); Vădean Cătălin (C.N. „Gh. Şincai”); Vrînceanu Flaviu (C.N. „Gh. Şincai”); Premiul II: Danci Patricia (C.N. „V. Lucaciu”); Iliuţă Filip (C.N. „Gh. Şincai”); Peştean Maria (Şc. Gim. Lăpuşel); Scurtu Ciprian (Șc.Gim. „N. Stănescu”); Suru Lia (C.N. „Gh. Şincai”); Oşan Andreea (C.N. „Gh. Şincai”); Suciu Bogdan (Șc.Gim. „D. Cantemir”); Dan Izabela (C.N. „Gh. Şincai”); Ciule Alexandru (Șc.Gim. „N. Iorga”); Filip Radu (C.N. „V. Lucaciu”) Polgar Alessia (C.N. „Gh. Şincai”); Trif Cristina (C.N. „V. Lucaciu”); Pintea Denisa (Şc.Gim. „G. Coşbuc”); Tătar Carina (C.T. Auto Baia Sprie); Silaghi Andrei (Şc.Gim. „Dr. V. Babeş”); Premiul III: Andreicuţ Elena Maria (C.N. „Gh. Şincai”); Ciociu Tudor (Şc.Gim. „I. L. Caragiale”); Gârdu Ionuţ (Şc.Gim. „L. Blaga”); Palincsar Lukas (C.N. „V. Lucaciu”); Pop-Ştrempel Alexandru (C.N. „Gh. Şincai”); Şeu David (Şc.Gim. „G. Coşbuc”); Mesani Raysa Ariana (Şc.Gim. „Al. I. Cuza”); Samoilă Denis Valentin (Şc.Gim. „Al. I. Cuza”); Cardoş Raul (C.N. „V. Lucaciu”); Mardar Lorena (C.N. „Gh. Şincai”); Danciu Amalia (Şc.Gim. „Ghe. Lupan” Groşi); Mureşan Rareş (Șc.Gim. „D. Cantemir”); Pop Mara (C.N. „Gh. Şincai”); Şimon Iarina (C.N. „V. Lucaciu”); Trif Alexandru (Şc.Gim. „G. Coşbuc”); Meszaroş Elisa (C.N. „V. Lucaciu”); Moge Antonia (C.N. „Gh. Şincai”); Velea Alexia (Șc.Gim. „N. Iorga”); Bonaţ Elida (C.N. „V. Lucaciu”); Drule Daniela (Şc.Gim. „G. Coşbuc”); Iştovan Irina (Şc.Gim. Lăpuşel); Man Carla (Șc.Gim. „N. Iorga”); Caba Brenda (Șc.Gim. „D. Cantemir”); Man Mihai (Şc.Gim. Nr.18); Pop Natalia (Șc.Gim. „N. Iorga”); Tomoiagă Vlad (Şc. Gim. „L. Blaga”); Turda Maria (Şc. Gim. „G. Coşbuc”); Vincze Denisa (C.N. „V. Lucaciu”); Bertolutti Lorenzo (C.N. „Gh. Şincai”); Chelement Cecilia (Șc.Gim. „D. Cantemir”); China Răzvan (Şc.Gim. „V. Alecsandri”); Pop Ioana (Șc.Gim. „D. Cantemir”); Rîpă Vasile (C.N. „V. Lucaciu”)Clasa a VI-a: Premiul de excelenţă: Ciceu Denis (Șc.Gim. „N. Iorga”); Talpoş Carina (Șc.Gim. „N. Iorga”); Premiul I: Maxim Sonia (C.N. „Gh. Şincai”); Turda Raul (C.N. „Gh. Şincai”); Afrăsinei Cătălin (C.N. „Gh. Şincai”); Herzal Radu (C.N. „Gh. Şincai”); Mariş Radu (C.N. „Gh. Şincai”); Graur Luca (Lic. „N. Bălcescu” Cluj); Lendel Andreea (C.N. „Gh. Şincai”); Giuroiu Tudor (C.N. „Gh. Şincai”); Premiul II: Săsăran Tania (C.N. „V. Lucaciu”); Biriş Erik (C.N. „Gh. Şincai”); Zaharie Oana (Șc.Gim. „N. Iorga”); Maroşan Leea (Şc.Gim. „A. Iancu”); Dragoş Monica (Șc.Gim. „N. Iorga”); Şuba Giulia (Şc.Gim. „I. L. Caragiale”); Onea Andrei Vlad (Şc.Gim. „L. Blaga”); Vanciu Daria (Șc.Gim. „N. Iorga”); Mercea Cezar (Șc.Gim. „N. Iorga”);

15

Page 16: SOCIETATEA DE ŞTIINŢE MATEMATICE DIN - ISJ … matematica 2015... · Web viewpentru punctaj maxim la sectiunea liceu Zelina Mihai, clasa a X-a, C.N. „V. Lucaciu”, prof. Boroica

Petrovan Alex (Şc.Gim. „ M. Sadoveanu”); Teglaş Bogdan (C.N. „Gh. Şincai”); Zăgreanu Luca (C.N. „Gh. Şincai”); Premiul III: Belous Bazil (C.N. „Gh. Şincai”); Bonte Eduard (Şc.Gim. „I. L. Caragiale”); Herţeg Ioana (C.N. „V. Lucaciu”); Dănilă Alexandru (Șc.Gim. „N. Iorga”); Tătaru Mihai (C.N. „Gh. Şincai”); Scoropan Cristian (Șc.Gim. „N. Iorga”); Verdeş Mara (C.N. „Gh. Şincai”); Buzgo Bianca (Şc.Gim. „L. Blaga”); Balogh David (Șc.Gim. „N. Iorga”); Bot Alex (Şc.Gim. „M. Sadoveanu”); Cupşa Cosmin (Şc.Gim. Asuaj); Shirazi Sarah (C.N. „V. Lucaciu”); Bălan Alexandra (Şc.Gim. „A. Iancu”); Lupşe Bianca (Şc.Gim. „G. Coşbuc”); Nagy Sara (Şc.Gim. „A. Iancu”); Bonto Iulia (Șc.Gim. „N. Iorga”); Şchiau Alex (Şc.Gim. „L. Blaga”); Horgoş Raul (Şc.Gim. „G. Coşbuc”); Coman Anda (C.N. „Gh. Şincai”); Darolţi Laura (C.N. „V. Lucaciu”)Clasa a VII-a: Premiul de excelenţă: Becsi Paul (C.N. „Gh. Şincai”); Premiul I: Robu Vlad (Șc.Gim. „N. Iorga”); Boroica Adrian (C.N. „Gh. Şincai”); Pop Călin (Șc.Gim. „N. Iorga”); Ilieş Iulia (C.N. „Gh. Şincai”); Andreicuţ Teofil (C.N. „Gh. Şincai”); Moldovan Nicolae (Şc.Gim. „G. Coşbuc”); Couţi Anamaria (C.N. „V. Lucaciu”); Rusnak Erika (C.N. „Gh. Şincai”); Premiul II: Filip Anda (C.N. „Gh. Şincai”); Horvat-Marc Andreea (C.N. „V. Lucaciu”); Ştef Eduard (C.N. „Gh. Şincai”); Breban Alexandru (Șc.Gim. „N. Stănescu”); Onea Vlad (Șc.Gim. „N. Iorga”); Ciocan Florin (Şc.Gim. „L. Blaga”); Palca Robert (C.N. „Gh. Şincai”); Dunca Alina (Șc.Gim. „N. Iorga”); Mociran Eduard (Şc.Gim. „G. Coşbuc”); Duma Luana (Şc.Gim. „L. Blaga”); Szabo Mark (C.N. „Gh. Şincai”); Premiul III: Ciolte Irina (Şc.Gim. „G. Coşbuc”); Erni Karin (C.N. „Gh. Şincai”); Horţ Mihai (Şc.Gim. „L. Blaga”); Tirizică Ionuţ (Șc.Gim. „N. Iorga”); Nicolaescu Tudor (C.N. „Gh. Şincai”); Făt Daria (Liceul de Arte); Pop Andrei (Şc.Gim. Sălsig); Bercea Melisa (Şc.Gim. „L. Blaga”); Bumbar Iulia (Şc.Gim. „L. Blaga”); Indrea Noris (Şc.Gim. „L. Blaga”); Nechita Amalia (Şc.Gim. „Ghe. Lupan” Groşi); Niţu Dariana (Șc.Gim. „N. Iorga”); Bîrlea Cristian (C.N. „Gh. Şincai”); Costinaş Gabriel (Şc.Gim. „Al. I. Cuza”); Dulfu Alex (C.N. „V. Lucaciu”) Clasa a VIII-a: Premiul de excelenţă: Zelina Paul (C.N. „ V. Lucaciu”); Premiul I: David Cătălin Ioan (C.N. „Gh. Șincai”); Diaconescu Mălina (C.N. „V. Lucaciu”); Bordeanu Lucia (Şc.Gim. „L. Blaga”); Buzila Gârda Andra (Şc.Gim. „G. Coșbuc”); Manolache Răzvan (Şc.Gim. „D. Cantemir”); Varady Iulia (Şc.Gim.”N. Iorga”); Premiul II: Pop Darius (Şc.Gim.”N. Iorga”); Zigler Alexandru (Şc.Gim.”N. Iorga”); Ardelean Horaţiu (Şc.Gim.”Al. I. Cuza”); Koblicica Andrei (C.N. „ V. Lucaciu”); Konyves Beatrix (Şc.Gim. „O. Goga”); Petruţ Andreea (Şc.Gim. „N. Iorga”); Băban Diana (Şc.Gim. „G. Coșbuc”); Tămîian Rareş Vasile (C.N. „Gh. Șincai”); Conţiu Alexandru (Şc.Gim.”N. Iorga”), Oşan Ştefania (C.N. „Gh. Șincai”), Roatiş Cristina (Şc.Gim. „D. Cantemir”); Ianoş Raul (Şc.Gim. „I.L. Caragiale); Ionuţ Bogdan (Şc.Gim. „N. Iorga”); Pop Alexandra (C.N. „Gh. Șincai”); Pop Darius (Şc.Gim. „L. Blaga”); Premiul III: Lemian Diana (Şc.Gim. „S. Bărnuţiu”); Buşecan Iulia (Şc.Gim. Fărcaşa); Dorca Răzvan (Şc.Gim.”N. Iorga”); Ghirasim Andrei (Şc.Gim. „G. Coșbuc”); Andercău Florina (Şc.Gim. „V. Alecsandri”); Buciuman Ionuţ (Şc.Gim. „D. Cantemir”); Oros Cristian (Şc.Gim. „G. Coșbuc”); Mardar Paul Cezar (C.N. „Gh.

16

Page 17: SOCIETATEA DE ŞTIINŢE MATEMATICE DIN - ISJ … matematica 2015... · Web viewpentru punctaj maxim la sectiunea liceu Zelina Mihai, clasa a X-a, C.N. „V. Lucaciu”, prof. Boroica

Șincai”); Gălăţanu Alexandra (C.N. „Gh. Șincai”); Radu Patricia (C.N. „Gh. Șincai”); Sabău Carla (Şc.Gim.”A. Iancu”); Şişeştean Radu (Şc.Gim. „V. Lucaciu” Şişeşti); Şuteu Ionuţ (Şc.Gim.”O. Goga”); Ardelean Ariana (Şc.Gim. „L. Blaga”); Babiciu Andreea (Şc.Gim. Şişeşti); Buşecan Maria (C.N. „Gh. Șincai”); Misaroş Andreea (Şc.Gim.”A. Iancu”); Uţă Adrian (Şc.Gim. „A.I.Cuza”); Vale Bogdan (Şc.Gim. I. L.”Caragiale”); Matei David Antoniu (C.N. „Gh. Șincai”); Mîrza Iza (Şc.Gim.”N. Iorga”); Muller Andrada (Şc.Gim.”A. Iancu”); Ghişa Ramona (Şc.Gim. Şişeşti); Ilieş Răzvan (Şc.Gim. „N. Iorga”); Moraru Dora (Şc.Gim. „G. Coșbuc”); Muthi Sonia (C.N. „Gh. Șincai”); Sava Bogdan (C.N. „Gh. Șincai”); Bîrle Bodan (Şc.Gim. „D. Cantemir”); Moldovan Andra (Şc.Gim. „D. Cantemir”); Şişeştean Radu (Şc.Gim. Şişeşti)Clasa a IX-a: Premiul de excelenţă: Tămâian Andrei (C.N. „Gheorghe Şincai”); Premiul I: Pop Vlad (C.N. „Gh. Șincai”); Neţa Răzvan (C.N. „Gh. Șincai”); Premiul II: Lucaciu Sergiu (C.N. „Gh. Șincai”; Mărieş Maria (C.N. „Gh. Șincai”); Berinde Thomas (C.N. „Gh. Șincai”); Premiul III: Suciu Răzvan (C.N. „Gh. Șincai”); Buciuman Adrian (C.N. „Gh. Șincai”);Bojor Barbu (C.N. „Gh. Șincai”), Danil Lidia (C.N. „V. Lucaciu”)Clasa a X-a: Premiul de excelenţă: Zelina Mihai (C.N. „V. Lucaciu”); Premiul I: Sântejudean Tudor (C.N. „Gh. Șincai”); Chişcă Andrei (C.N. „Gh. Șincai”); Premiul II: Crăciun George (C.N. „Gh. Șincai”); Goteciuc Horaţiu (C.N. „V. Lucaciu”); Premiul III: Oana Laura (C.N. „Gh. Șincai”); Covaci Alexandra (C.N. „V. Lucaciu”); Griguţa Paula (C.N. „Gh. Șincai”); Olari George (C.N. „Gh. Șincai”); Mureşan Diana (C.N. „Gheorghe Şincai)Clasa a XI-a: Premiul de excelenţă: Cotan Paul (C.N. „Gh. Șincai”); Premiul I : Tînțar Oana (C.N. „Gh. Șincai”); Butnar Adrian (C.N. „Gheorghe Șincai); Premiul II: Zicher Blanka (C.N. „Gh. Șincai”); Todoran Larisa (C.N. „Gh. Șincai”); Premiul III: Ari Paul (C.N. „Gh. Șincai”); Bocuţ Oana (C.N. „Gh. Șincai”);Tyekar Dan,(C.N. „V. Lucaciu”); Voiţ Radu (C.N. „Gh. Șincai”)Clasa a XII-a M1: Premiul de excelenţă: Ciurte Tudor (C.N. „Gh. Șincai”); Premiul I: Bud Cristian (C.N. „Gh. Șincai”); Fodoruţ Ionuţ (C.N. „V. Lucaciu”); Gotha Guntter (Lic.Teor. „N. Laszlo”); Premiul II: Nicolaescu Andrei (C.N. „Gh. Șincai”); Miclea Andrei (C.N. „Gh. Șincai”); Ghiţescu Nicoleta (C.N. „V. Lucaciu”); Vele Corina (C.N. „Gh. Șincai”); Gyarmathy Timea (Lic.Teor. „N. Laszlo”); Heredea Oana (C.N. „Gh. Șincai”); Premiul III: Bălan Alexandru (C.N. „V. Lucaciu”); Rus Alexandru (C.N. „V. Lucaciu” ); Chiş Ioan (C.T. „G. Bariţiu”); Anton Costa (C.N. „Gh. Șincai”); Peter Cătălin (C.N. „Gh. Șincai”); Pop Andra (C.N. „Gh. Șincai”); Tomoiagă Ana (C.N. „Gh. Șincai”); Moldovan Teodora (C.N. „V. Lucaciu”); Rus Elena (C.N. „Gh. Șincai”); Dumitru Daniela (C.N. „Gh. Șincai”); Rad Gabriel (Lic.Teor. „E. Racoviţă”); Zaharie Sergiu (C.N. „Gh. Șincai”)Clasa a XII-a M2: Premiul de excelenţă: Hanţig Ştefan (Lic.Teor. „E. Racoviţă”); Premiul I: Santa Maria (C.N. „Gh. Șincai”); Premiul II: Pop Darius (C.Ec. „N. Titulescu”); Şandor Sebastian (C.Ec. „N. Titulescu”); Premiul III: Olar Adrian (C.Ec. „N. Titulescu”); Biriş Mădălina (Lic.Teor. „E. Racoviţă”); David Camelia (C.N. „Gh.

17

Page 18: SOCIETATEA DE ŞTIINŢE MATEMATICE DIN - ISJ … matematica 2015... · Web viewpentru punctaj maxim la sectiunea liceu Zelina Mihai, clasa a X-a, C.N. „V. Lucaciu”, prof. Boroica

Șincai”); Koza Andrei (C.Ec. „N. Titulescu”); Pop Rebeca (C.Ec. „N. Titulescu”); Tomoiagă Ileana (Lic.Teor. „E. Racoviţă”); Văleanu Andreea (C.Ec. „N. Titulescu”); Marco Adelin (C.Ec. „N. Titulescu”); Săsăran Dan (C.Ec. „N. Titulescu”).

CONCURSUL INTERJUDEȚEAN „OLIMPIADA SATELOR DIN TRANSILVANIA”

Ediția a V-a, Bistrița Năsăud 2015

Periada de desfășurare: 24-26 aprilie 2015Loc de desfășurare: localitatea Șanț, județul Bistrița –NăsăudRezultatele concursului:PREMIUL I: Șimon Adrian, clasa a IV-a, Șc.Gim. Vișeu de Jos, învățător Pop Anișoara (punctaj maxim); Zubașcu Maria, clasa a VII-a, Șc.Gim. „Liviu Rebreanu” Dragomirești, profesor Zubaşcu Daniela; PREMIUL II: Iștovan Irina, clasa a V-a, Șc.Gimn. Lăpușel, profesor Mureșan Corina; PREMIUL III: Zubașcu Adina, clasa a V-a, Șc.Gim. „Liviu Rebreanu” Dragomirești, profesor Zubașcu Daniela; Pop Raul , clasa a VI-a, Şc.Gimn. Vișeu de Jos, profesor Tomoiagă Ioan; Mențiuni: Dolca Diana Alina, clasa a IV-a, Șc.Gim. „Liviu Rebreanu” Dragomirești, învățător Ofrim Maria Floare; Dunca Ioniță, clasa a IV-a, Șc.Gim. „Liviu Rebreanu” Dragomirești, învățător Bodea Maria; Pascu Marius Ionuț, clasa a VII-a, Șc.Gim. Vișeu de Jos, profesor Pop Gavrilă

Au mai făcut parte din lotul județului Maramureș următorii elevi: Coman Anișoara, clasa a VIII-a, Șc.Gim. Vișeu de Jos; Puicar Eugen, clasa a VIII-a Șc.Gim. Strâmtura; Vancea Teodor, clasa a VII-a, Șc.Gim. „Mihai Eminescu” Săliștea de Sus; Tomoiagă Iulia, clasa a VII-a, Șc.Gim. Vișeu de Jos; Feier Andrei, clasa a VI-a, Șc.Gim. „Dr Ilie Lazăr” Giulești; Florean Constantin, clasa a V-a, Șc.Gim. Copalnic Mănăștur; Țicală Andreea, clasa a VI-a , Șc.Gim. „M. Eminescu” Săliștea de Sus; Mihalca Grigore, clasa a VI-a, Șc.Gim. Vișeu de Jos.

Ediția a VI-a va avea loc în județul Mureș.

CONCURSUL INTERJUDEŢEAN DE MATEMATICĂ „ARGUMENT”Ediţia a VII-a, 2014, Baia Mare

Organizatori: Catedra de matematică a Colegiului Naţional „Gh.Şincai” în parteneriat cu Asociaţia „Argument”Locul de desfăşurare: Colegiul Naţional „Gh. Şincai” Perioada: 8-9 noiembrie 2014Preşedintele concursului: conf. univ. Vasile Pop de la Universitatea Tehnică din Cluj NapocaParticipanţi: loturile Colegiilor Naţionale: „A. Mureşanu” Dej, „M. Eminescu” Satu Mare, „Al. Papiu Ilarian” Târgu Mureş, „Silvania” Zalău, „D. Vodă” Sighetu Marmaţiei, „V. Lucaciu” Baia Mare, „Gh. Şincai” Baia Mare, precum şi elevi de gimnaziu de la şcolile reprezentative din oraş

18

Page 19: SOCIETATEA DE ŞTIINŢE MATEMATICE DIN - ISJ … matematica 2015... · Web viewpentru punctaj maxim la sectiunea liceu Zelina Mihai, clasa a X-a, C.N. „V. Lucaciu”, prof. Boroica

Partenerii concursului: Primăria şi Consiliul Local al Municipiului Baia Mare; Consiliul Judeţean Maramureş; Inspectoratul Şcolar Judeţean MaramureşSponsori: S.C. Baeninnger S.R.L., S.C. Zicher S.R.L., Eurohotel Baia Mare, Divas S.R.L., Asociaţia „Argument”Lista premianţilor:Clasa a V-a: Premiul I: Hossu Iulia (C.N. „D. Vodă” Sighetu Marmaţiei); Zlampareţ George (C.N. „Gh. Şincai” Baia Mare); Iliuță Filip (C.N. „Gh. Şincai” Baia Mare); Premiul II: Robu Raluca (C.N. „V. Lucaciu” Baia Mare); Lazea Darius (C.N.” Gh. Şincai” Baia Mare); Cozmuta Tudor (Şc. Gim. „N. Iorga” Baia Mare); Premiul III: Vădean Cătălin (C.N. „Gh. Şincai” Baia Mare); Ştirbu Silvia (C.N. „V. Lucaciu” Baia Mare); Mecea Corina (Şc.Gim. „G. Coşbuc” Baia Mare); Şeu David (Şc.Gim. „G. Coşbuc” Baia Mare)Clasa a VI-a: Premiul I: Talpoş Carina (Şc.Gim. „N. Iorga” Baia Mare); Zaharie Oana (Şc.Gim.”N. Iorga” Baia Mare); Premiul II: Cionte Sergiu (C.N. „Gh. Şincai” Baia Mare); Treista Georgiana (C.N. „D. Vodă” Sighetu Marmaţiei); Premiul III: Turda Raul (C.N. „Gh. Şincai” Baia Mare); Lazăr Laurenţiu (C.N. „D. Vodă” Sighetu Marmaţiei) Clasa a VII-a: Premiul I: Robu Vlad (Şc.Gim.„N. Iorga” Baia Mare); Pop Călin (Şc.Gim. „N. Iorga” Baia Mare); Premiul II: Boroica Adrian (C.N. „Gh. Şincai” Baia Mare); Ilieş Iulia (C.N. „Gh. Şincai” Baia Mare); Francioli Daria (C.N. „V. Lucaciu” Baia Mare); Roman Ioana (C.N. „M. Eminescu” Satu Mare); Premiul III: Becsi Paul (C.N. „Gh. Şincai” Baia Mare), Moldovan Nicolae (Şc.Gim.”G. Coşbuc” Baia Mare)Clasa a VIII–a: Premiul I: Zelina Paul (C.N. „V. Lucaciu” Baia Mare); Premiul II: Matei Bledea Alexandru (C.N.”Gh. Şincai” Baia Mare); Mercea Ioana (C.N. „Gh. Şincai” Baia Mare); Premiul III: Cotârlan Codrin (C.N. „D. Vodă” Sighetu Marmaţiei); Tomoiagă Radu (Şc.Gim. „G. Coşbuc” Sighetu Marmaţiei); Stepan Dacian (Şc.Gim. „G. Coşbuc” Sighetu Marmaţiei)Clasa a IX-a: Premiul I: Tămâian Andrei (C.N. „Gh. Şincai” Baia Mare); Tenie Anda (C.N. „Al. Papiu Ilarian” Tg. Mureş); Premiul II: Lucaciu Sergiu (C.N. „Gh. Şincai” Baia Mare); Pop Vlad (C.N. „Gh. Şincai” Baia Mare); Premiul III: Mărieş Maria (C.N. „Gh. Şincai” Baia Mare)Clasa a X-a: Premiul I: Buna-Mărginean Alex (C.N. „Al. Papiu Ilarian” Tg. Mureş); Zelina Mihai (C.N. „V. Lucaciu” Baia Mare); Premiul II: Sântejudean Tudor (C.N.” Gh. Şincai” Baia Mare); Premiul III: Nicuşor Andrei (C.N. „Al. Papiu Ilarian” Tg.Mureş); Oprea Maria (C.N. „Al. Papiu Ilarian” Tg. Mureş)Clasa a XI-a: Premiul I: Sabău Vlad (C.N. „Al. Papiu Ilarian” Tg. Mureş); Premiul II: Butnar Adrian (C.N. „Gh. Şincai” Baia Mare); Cotan Paul (C.N. „Gh. Şincai” Baia Mare); Premiul III: Georgescu Sabina (C.N. „Unirea” Tg. Mureş)Clasa a XII-a: Premiul I: Moldovan Bogdan (Lic.Teor. „O.Ghibu” Cluj Napoca); Premiul II: Bud Cristian (C.N. „Gh. Şincai” Baia Mare); Şerban Ştefana (Lic.Teor. „P. Maior” Sighişoara); Gotha Guntter (Lic.Teor. „N. Laszlo” Baia Mare); Premiul III: Bura Lucia (C.N. „M. Eminescu” Satu Mare); Marele premiu „Dumitru Angheluţă”, pentru cel mai mare punctaj obţinut în concurs dintre elevii de liceu,

19

Page 20: SOCIETATEA DE ŞTIINŢE MATEMATICE DIN - ISJ … matematica 2015... · Web viewpentru punctaj maxim la sectiunea liceu Zelina Mihai, clasa a X-a, C.N. „V. Lucaciu”, prof. Boroica

instituit în memoria marelui profesor de matematică al Colegiului Naţional „Gh. Şincai” Baia Mare, a fost câştigat de elevul Moldovan Bogdan de la Liceul Teoretic „O.Ghibu” Cluj Napoca.

CONCURSUL INTERJUDEŢEAN AL CENTRELOR DE EXCELENŢĂ„TINERE SPERANŢE”

Ediţia a X-a, 12 - 13 decembrie 2014, Baia Mare

Proba pe echipeNumărul elevilor participanţi la proba pe echipe: 354 elevi (clasa a V-a 123 elevi; clasa a VI-a 87 elevi; clasa a VII-a 69 elevi; clasa a VIII-a 75 elevi)Marele Trofeu a fost câştigat de echipa: Zlampareţ George, Iliuţă Filip, Gulin Tudor, clasa a V-a, C.N. „Gh. Şincai” Baia Mare, prof. Muşuroia NicolaePremii:Clasa a V-a: Premiul I: Zlampareţ George, Iliuţă Filip, Gulin Tudor, C.N. „Gh. Şincai” Baia Mare, prof. Muşuroia Nicolae; Premiul II: Mujdar Milan, Covrig Larisa, Hosu Iulia, C.N. „D. Vodă” Sighetu Marmaţiei, prof. Bedeoan Loredana; Premiul III: Braica Patricia, Pop Sebastian, Tot Marc, C.N. „M. Eminescu” Satu Mare, prof. Pop Ovidu; Menţiuni: Lazea Darius, Andreicuţ Elena, Vădean Cătălin, C.N. „Gh. Şincai” Baia Mare, prof. Bojor MedaClasa a VI-a: Premiul I: Maxim Sonia, Turda Raul, Giuroiu Tudor, C.N. „Gh. Şincai” Baia Mare, prof. Boroica Gheorghe; Premiul II: Lazăr Laurenţiu, Treista Ioana, Rapolţi Daria, C.N. „D. Vodă” Sighetu Marmaţiei, prof. Bedeoan Loredana; Premiul III: Herzal Radu, Biriş Erik , Afrăsinei Cătălin, C.N. „Gh. Şincai” Baia Mare, prof. Heuberger Dana Menţiuni: Zaharie Ioana, Talpoş Carina, Ciceu Denis, Şc. Gim. „N. Iorga” Baia Mare, prof. Bretan AndreiClasa a VII-a: Premiul I: Mociran Iulia, Onea Vlad, Pop Călin, Şc.Gim. „N. Iorga” Baia Mare, prof. Bretan Andrei și Boroica Adrian, Andreicuţ Teofil, Becsi Paul, C.N.”Gh. Şincai” Baia Mare, prof. Bojor Florin; Premiul II: Filip Anda, Ilieş Iulia, Rusznak Erika, C.N. „Gh. Şincai” Baia Mare, prof. Bojor Florin; Premiul III: Moldovan Nicolae, Mociran Eduard, Ciolte Irina, Şc.Gim. „G. Coşbuc” Baia Mare, prof. Pop Cosmin; Menţiuni: Filip Aaron, Francioli Daria, Horvat Marc Andreea, C.N. „V. Lucaciu” Baia Mare, prof. Sfara Gheorghe; Clasa a VIII-a: Premiul I: Târlişan Ioan-Paul, Pop Sergiu, Szasz Bogdan, Şc. Gim. „M. Eminescu” Năsăud, Şc. Gim. Dobric, Şc.Gim. „Gr. Silasi”, Beclean, prof. Roman Cerasela, prof. Pop Alexandru, prof. Marc Vasile şi Matei Bledea Alexandru, Mercea Ioana, Ghisa Remus, C.N. „Gh. Şincai” Baia Mare, prof. Muşuroia Nicolae; Premiul II: Zelina Paul, Diaconescu Mălina, Petz Alin, C.N. „V. Lucaciu” Baia Mare, prof. Boroica Gabriela; Premiul III: Reu Teodora, Bumb Paul George, Deac Alex Claudiu, Şc.Gim. „L. Blaga” Bistriţa, prof. Pop Stela; Menţiuni: Băban Diana, Buzilă-Gârda Andra, Moraru Dora, Şc.Gim. „G. Coşbuc” Baia Mare, prof. Zah Ştefan; Proba individuală

20

Page 21: SOCIETATEA DE ŞTIINŢE MATEMATICE DIN - ISJ … matematica 2015... · Web viewpentru punctaj maxim la sectiunea liceu Zelina Mihai, clasa a X-a, C.N. „V. Lucaciu”, prof. Boroica

Numărul elevilor participanţi la proba individuală: 314 elevi (clasa a V-a 104 elevi; clasa a VI-a 87 elevi; clasa a VII-a 57 elevi; clasa a VIII-a 65 elevi)PremiiClasa a V-a: Premiul I: Iliuţă Filip, C.N. „Gh. Şincai” Baia Mare, prof. Muşuroia Nicolae; Premiul II: Zlampareţ George, C.N. „Gh. Şincai” Baia Mare, prof. Muşuroia Nicolae; Robu Raluca, C.N. „V. Lucaciu” Baia Mare, prof. Bretan Andrei; Premiul III: Mujdar Milan, C.N. „D. Vodă” Sighetu Marmaţiei, prof. Bedeoan Loredana; Menţiuni: Gulin Tudor , C.N. „Gh. Şincai” Baia Mare, prof. Muşuroia Nicolae şi Pop Andrea, C.N. „V. Lucaciu” Baia Mare, prof. Bretan AndreiClasa a VI-a: Premiul I: Talpoş Carina, Şc. Gim. „N. Iorga” Baia Mare, prof. Bretan Andrei; Premiul II: Pop Raul , Şc. Gim. Vişeu de Jos, prof. Tomoiagă Ioan; Premiul III: Giuroiu Tudor, C.N. „Gh. Şincai” Baia Mare, prof. Boroica Gheorghe; Menţiuni: Biriş Erik, C.N. „Gh. Şincai” Baia Mare, prof. Heuberger Dana şi Treista Ioana Georgiana, C.N. „D. Vodă” Sighetu Marmaţiei, prof. Bedeoan LoredanaClasa a VII-a: Premiul I: Pop Călin, Şc. Gim. „N. Iorga” Baia Mare, prof. Bretan Andrei; Premiul II: Moldovan Nicolae, Şc. Gim.”G. Coşbuc” Baia Mare, prof. Pop Cosmin; Premiul III: Boroica Adrian, C.N. „Gh. Şincai” Baia Mare, prof. Bojor Florin; Menţiuni: Ilieş Iulia, C.N. „Gh. Şincai”,Baia Mare, prof.Bojor Florin şi Francioli Daria, C.N. „V. Lucaciu” Baia Mare, prof. Sfara GheorgheClasa a VIII-a: Premiul I: Târlişan Ioan–Paul, Şc. Gim. „M. Eminescu” Năsăud, prof. Roman Cerasela; Premiul II: Mercea Ioana, C.N. „Gh. Şincai” Baia Mare, prof. Muşuroia Nicolae; Premiul III: Stepan Dacian, Şc. Gim.”G. Coşbuc” Sighetu Marmaţiei, prof. Zetea Bogdan și Zelina Paul, C.N. „V. Lucaciu” Baia Mare, prof. Boroica Gabriela; Menţiuni: Deac Alex Claudiu, Şc. Gim. „L. Blaga” Bistriţa, prof. Pop Stela şi Dicu Dan Alexandru, C.N. „D. Vodă” Sighetu Marmaţiei, prof. Bedeoan Loredana

CONCURSUL INTERJUDEŢEAN „PRIN LABIRINTUL MATEMATICII”Ediţia a X-a, Baia Mare, 23-25 ianuarie 2015

La aceasta ediţie aniversară au participat 215 de elevi din şcolile şi liceele judeţului nostru, dar şi din judeţul Botoşani; au fost premiaţi în număr de 105 elevi astfel s-au acordat 17 premii I, 11 premii II, 11 premii III , 7 menţiuni speciale şi 69 de menţiuni.

Le mulţumim gazdelor Colegiul Naţional „V. Lucaciu” Baia Mare, pentru modul în care au realizat această ediţie, cu un număr record de participare.Lista elevilor premianţiClasa a IV-a: Premiul I: Costin Oana (Şc.Gim. „N. Stănescu” Baia Mare); Premiul II: Muntean Tudor (Şc.Gim. „N. Iorga” Baia Mare); Premiul III: Onea Iulian (Şc.Gim. „N. Iorga” Baia Mare); Menţiune specială: Tuş Traian (Şc.Gim. „N. Stănescu” Baia Mare); Menţiuni: Şpan Mihai, Coman David, Biriş Marc, Tibil Oana, Mărcuş Alexandru, Marchiş Marcus, Szmuck Patrick, Rus Tudor, Tunyogi Kevin, Buia Theodor, Lucăceanu Maria, Petruţ Alexandra, Crişan Samuel, Raţ Carmina, Mureşan Andreas, Pop Darius

21

Page 22: SOCIETATEA DE ŞTIINŢE MATEMATICE DIN - ISJ … matematica 2015... · Web viewpentru punctaj maxim la sectiunea liceu Zelina Mihai, clasa a X-a, C.N. „V. Lucaciu”, prof. Boroica

Clasa a V-a: Premiul I: Dragoş Andreea (C.N.”V. Lucaciu” Baia Mare); Lazea Darius şi Zlampareţ George (C.N. „Gh. Şincai” Baia Mare); Premiul II: Vădean Cătălin (C.N. „Gh. Şincai” Baia Mare); Premiul III: Hosu Iulia (C.N. „D. Vodă” Sighetu Marmaţiei); Menţiune specială: Gulin Tudor (C.N. „Gh. Şincai” Baia Mare); Menţiuni: Şimon Iarina, Mujdar Milan, Mesaroş Radu, Suciu Bogdan, Mecea Corina, Bragaru Maria, Danci Patricia, Borcuti Oana, Korponay Albert, Şeu David, Ştirbu Silvia, Koblicica Vlad, Munteanu Vlad, Pop Andreea, Vâţă Alexandra, Bonaţ Elida, Plavoşin VladClasa a VI-a: Premiul I: Talpoş Carina (Şc. Gim. „N. Iorga” Baia Mare); Premiul II: Turda Raul (C.N. „Gh. Şincai” Baia Mare); Premiul III: Pop Raul Marius (Şc. Gim. Vişeu de Jos); Menţiune specială: Ciceu Denis (Şc. Gim. „N. Iorga” Baia Mare); Menţiuni: Treista Georgiana, Giuroiu Tudor, Stan Teodora, Maxim Sonia, Lazăr Laurenţiu, Vanciu Daria, Ghişe Teodora, Biriş Erik, Zaharie Oana, Pietraru Alfred, Lendel Andreea, Doia Bogdan, Herzal Radu, Săsăran Tania, Dragoş Monica, Mercea CezarClasa a VII-a: Premiul I: Boroica Adrian (C.N. „Gh. Şincai” Baia Mare); Horvat-Marc Andreea (C.N.”V. Lucaciu” Baia Mare); Pop Călin George şi Robu Vlad Nicolae (Şc. Gim. „N. Iorga” Baia Mare); Premiul II: Becsi Paul (C.N. „Gh. Şincai” Baia Mare); Premiul III: Andreicuţ Teofil (C.N. „Gh. Şincai” Baia Mare); Gaborean Marcel (C. N. „D. Vodă” Sighetu Marmaţiei); Menţiune specială: Moldovan Nicolae (Şc. Gim. „G. Coşbuc” Baia Mare) ; Menţiuni: Giesswein Alexia, Breban Alexandru, Francioli Daria, Ilieş Iulia, Couţi Anamaria, Onea Vlad Florin, Filip Anda, Huţanu Diana, Nechita Bianca, Mociran Eduard, Stan AndreeaClasa a VIII-a: Premiul I: Matei Bledea Alexandru (C.N. „Gh. Şincai” Baia Mare); Zelina Paul (C.N.”V. Lucaciu” Baia Mare); Premiul II: Ghişa Remus (C.N. „Gh. Şincai” Baia Mare); Premiul III: Cotârlan Codrin (C. N. „D. Vodă” Sighetu Marmaţiei); Menţiune specială: Diaconescu Malina (C.N.”V. Lucaciu” Baia Mare); Menţiuni: Petz Alin, Conţiu Alexandru, Mureşan Ioan Alexandru, Raileanu AlexandruClasa a IX-a: Premiul I: Pop Vlad (C.N. „Gh. Şincai” Baia Mare); Premiul II: Lucaciu Sergiu şi Tămâian Andrei (C.N. „Gh. Şincai” Baia Mare); Premiul III: Mărieş Maria (C.N. „Gh. Şincai” Baia Mare); Menţiune specială: Bojor Barbu (C.N. „Gh. Şincai Baia Mare); Menţiuni: Cudrici Carina, Neţa Răzvan, Palca Mihaela, Darolţi LarisaClasa a X-a: Premiul I: Zelina Mihai (C.N.”V. Lucaciu” Baia Mare); Premiul II: Sântejudean Tudor (C.N. „Gh. Şincai” Baia Mare)Clasa a XI-a: Premiul I: Ţînţar Oana (C.N. „Gh. Şincai” Baia Mare); Premiul II: Butnar Adrian (C.N. „Gh. Şincai” Baia Mare); Danci Bianca şi Pop Darius (C.N. „D. Vodă” Sighetu Marmaţiei); Premiul III: Păcurar Aralda Dora (C.N. „D. Vodă” Sighetu Marmaţiei); Menţiune specială: Tyekar Dan (C.N. „V. Lucaciu” Baia Mare)Clasa a XII-a: Premiul I: Ciurte Tudor (C.N. „Gh. Şincai” Baia Mare); Gotha Güntter (Lic.Teor. „N. Laszlo” Baia Mare); Petruş Andrei (C.N.”V. Lucaciu” Baia Mare); Premiul III: Fodoruţ Ioan (C.N.”V. Lucaciu” Baia Mare)

22

Page 23: SOCIETATEA DE ŞTIINŢE MATEMATICE DIN - ISJ … matematica 2015... · Web viewpentru punctaj maxim la sectiunea liceu Zelina Mihai, clasa a X-a, C.N. „V. Lucaciu”, prof. Boroica

CONCURSUL „SIGMA”Ediţia a XX-a, 9 mai 2015, Sighetu Marmaţiei

Organizator : Colegiul Naţional „Dragoş Vodă” Sighetu MarmaţieiSponsorii concursului: Mecanica SA Sighet, Banca Cooperatistă TISA, Nana&Darius Conta Expert, Urgent Curier Sighet şi ASP Dragoş Vodă.

Sâmbătă 9 mai 2015 la Colegiul Naţional „Dragoş Vodă” Sighetu Marmaţiei s-a desfăşurat a XX–a ediţie a concursului SIGMA. Au participat 161 de elevi la matematică şi 28 la informatică.Premii :Clasa a IV-a: Premiul I: Giurgi Bogdan (Şc.Gim. „G.Coşbuc” Sighetu Marmaţiei); Premiul II: Rodilă Andrei (Lic.Ped. „Regele Ferdinand” Sighetu Marmaţiei); Premiul III: Moişi Samuel Emanuel (Lic.Ped. „Regele Ferdinand” Sighetu Marmaţiei); Menţiuni:Mintău Andrei, Onişa Ariana, Şteţiu Stephanie, Gherlan Marius, Dolca Diana Alina, Szebeni Amalia, Zubaşcu LarisaClasa a V-a: Premiul I: Mujdar Milan (C.N. „D. Vodă” Sighetu Marmaţiei); Premiul II:Andreicuţ Elena (C.N. „Gh. Şincai” Baia Mare); Premiul III: Hosu Iulia (C.N. „D. Vodă” Sighetu Marmaţiei); Menţiuni: Man Robert Antonio, Pop Ştrempel Alexandru, Hotea Mihai, Mecea CorinaClasa a VI-a: Premiul I: Treista Ioana Georgiana (C.N. „D. Vodă” Sighetu Marmaţiei);Premiul II: Lazăr Laurenţiu (C.N. „D. Vodă” Sighetu Marmaţiei);Premiul III: Rad Vlăduţ (Şc.Gim. „G.Coşbuc” Sighetu Marmaţiei); Menţiuni: Rapolţi Daria Elena, Szekely Bianca, Buftea Denisa, Ilieş BogdanClasa aVII-a: Premiul I: Andreicuţ Teofil (C. N. „Gh. Şincai” Baia Mare); Premiul II: Moldovan Nicolae (Şc.Gim. „G. Coşbuc” Baia Mare); Premiul III: Ţiplea Ştefan Alexandru (C.N. „D. Vodă” Sighetu Marmaţiei); Menţiuni: Corneştean Loretta Jasmina, Ilieş Iulia Maria, Gaborean Vasile Marcel, Spaczai Carla NoemiClasa aVIII-a: Premiul I: Ștefan Dacian (Şc.Gim. „G.Coşbuc” Sighetu Marmaţiei); Premiul II: Cotârlan Codrin (C.N. „D. Vodă” Sighetu Marmaţiei); Premiul III: Hodor Alexandru (Şc.Gim. „G.Coşbuc” Sighetu Marmaţiei); Menţiuni: Mih Mara, Bledea Dragoş, Petrovan Claudia, Merza Bîrlea GiorgianaClasa a IX-a: Premiul II: Ilieş Dragoş Ionuţ (C.N. „D. Vodă” Sighetu Marmaţiei)Clasa a X-a: Premiul I: Ilieş Andreea (C.N. „ D. Vodă” Sighetu Marmaţiei); Premiul II: Vraja Iulian (C.N. „D. Vodă” Sighetu Marmaţiei); Premiul III: Oniga Gabriel (C.N. „D. Vodă” Sighetu Marmaţiei)Clasa a XI-a: Premiul I: Pop Darius (C.N. „D. Vodă” Sighetu Marmaţiei); Premiul II: Buftea Mădălina (C.N. „D. Vodă” Sighetu Marmaţiei); Premiul III: Tomoiagă Alexandru (C.N. „D. Vodă” Sighetu Marmaţiei)

23

Page 24: SOCIETATEA DE ŞTIINŢE MATEMATICE DIN - ISJ … matematica 2015... · Web viewpentru punctaj maxim la sectiunea liceu Zelina Mihai, clasa a X-a, C.N. „V. Lucaciu”, prof. Boroica

CONCURSUL DE MATEMATICĂ „AS”Ediţia a VI-a, Baia Mare, 24 ianuarie 2015

La această ediţie au participat 62 de elevi din Centrul judeţean pentru tinerii capabili de performanţă de la Colegiul National „Vasile Lucaciu” Baia Mare; au fost premiaţi în număr de 25 elevi; astfel s-au acordat 4 premii I, 4 premii II, 4 premii III şi 13 de menţiuni.

Le mulţumim gazdelor Colegiul Naţional „ Vasile Lucaciu”, Baia Mare, pentru modul în care au realizat această ediţie.Lista elevilor premianţiClasa a IX-a: Premiul I: Bolte Adrian (C.Ec. „N.Titulescu” Baia Mare); Premiul II: Sălăjan Bogdan (C.Ec. „N.Titulescu”,Baia Mare); Premiul III: Lazăr Andrada (C.Ec. „N.Titulescu” Baia Mare)Clasa a X-a: Premiul I: Nastai Sorina (C.Ec. „N.Titulescu” Baia Mare); Premiul II: Guţ Simina Florina (C.Ec. „N.Titulescu” Baia Mare); Premiul III: Varga Bretan Andrei (C.T. „G. Bariţiu” Baia Mare); Menţiuni: Gherheş BiancaClasa a XI-a: Premiul I: Herczeg Adrian (C.Ec. „N.Titulescu” Baia Mare); Premiul II: Sabou Denis (C.Ec. „N.Titulescu” Baia Mare); Premiul III: Ianos Simona (Liceul Teologic Penticostal Baia Mare); Menţiuni: Nan Andra, Pop Claudiu, Roman Mihai, Micaş Vasile, Nagy RaduClasa a XII-a: Premiul I: Chiş Ioan (C.T. „G. Bariţiu” Baia Mare); Premiul II: Hanţig Ştefan Cristian (Lic.Teor. „E. Racoviţă” Baia Mare); Premiul III: Gheţie Ioana (Lic.Teor. „E. Racoviţă” Baia Mare); Menţiuni: Vinţ Crina, Konyicska Timea Brigitta, Mogagea Andreea, Ferenţi Ionuţ, Hereş Debora, Tomoiaga Ileana, Libotean Diana

CONCURSUL NAŢIONAL DE MATEMATICĂ „LUMINAMATH”Ediţia a XVIII-a, 22 noiembrie 2014

Luminamath este un concurs de matematică destinat elevilor din clasele II-VIII, care le permite să-şi vadă nivelul de pregătire la nivel naţional. Anul acesta din judeţul Maramureş au participat un număr de 1237 elevi: la clasa a II-a 137 elevi, a III-a 293 elevi, a IV-a 225 elevi, la clasa a V-a 250 elevi, la clasa a VI-a 144 elevi, la clasa a VII-a 96 elevi, la clasa a VIII-a 92 elevi.Centre de concurs: Grup Şcolar Borşa; Şcoala Gimnazială „Dr. Ilie Lazăr” Giuleşti; Şcoala Gimnazială „George Coşbuc” Baia Mare; S.A.M. Nr.1 Poienile de Sub Munte; Şcoala Gimnazială „Nichita Stănescu” Baia Mare; Şcoala Gimnazială Nr.2 Sighetu Marmaţiei; Liceul Tehnologic „Octavian Goga” Rozavlea; Şcoala Gimnazială „Mihai Eminescu” Săliştea de Sus; Şcoala Gimnazială Gârdani

La nivel naţional s-au obţinut 22 de premii în bani şi tabără la Potigrafu: Becsi Paul, C.N. „Gh. Şincai” Baia Mare; Biriş Erik, C.N. „Gh. Şincai” Baia Mare; Bledea Alexandru, C.N. „Gh. Şincai” Baia Mare; Bonea Radu, C.N. „V. Lucaciu” Baia Mare; Boroica Adrian, C.N. „Gh. Şincai” Baia Mare; Chende Mihaela, Şc. Gim. „G.

24

Page 25: SOCIETATEA DE ŞTIINŢE MATEMATICE DIN - ISJ … matematica 2015... · Web viewpentru punctaj maxim la sectiunea liceu Zelina Mihai, clasa a X-a, C.N. „V. Lucaciu”, prof. Boroica

Coşbuc” Baia Mare; Ciceu Denis, Şc.Gim. „N. Iorga” Baia Mare; Cosar Antonia, Şc.Gim. „G. Coşbuc” Baia Mare; Dragoş Andreea, C.N. „V. Lucaciu” Baia Mare; Gheţe Ruxandra, Şc.Gim. „A. Iancu” Baia Mare; Koblicica Andrei, C.N. „V. Lucaciu” Baia Mare; Mercea Ioana, C.N. „Gh. Şincai” Baia Mare; Muntean Tudor, Şc. Gim.„N. Iorga” Baia Mare; Onea Iulian, Şc. Gim.„N. Iorga” Baia Mare; Pop Călin, Şc.Gim. „N. Iorga” Baia Mare; Robu Dacian, Şc.Gim. „G. Coşbuc” Baia Mare; Robu Vlad, Şc.Gim. „N. Iorga” Baia Mare; Săcălean Ada, Şc.Gim. „G. Coşbuc” Baia Mare; Sajerli Alesia, Şc.Gim. „A. Iancu” Baia Mare; Şpan Mihai, Şc.Gim. „N. Iorga” Baia Mare; Ţiplea Ştefan, C.N. „D. Vodă” Sighetu Marmaţiei; Vultur Vlad, Şc.Gim. „G. Coşbuc” Baia Mare.

CONCURSUL INTERJUDEŢEAN DE MATEMATICĂ ŞI INFORMATICĂ „GR. C. MOISIL”

Ediţia a XXX-a, 2015- Cluj Napoca

Locul de desfăşurare: Cluj NapocaPerioada: 27-29 martie 2015Sponsori: FABRICA DE AVIOANE - UACE EUROPE din Dumbrăvița, Filiala MM A SSMR și Asociația profesorilor de informatică „IN4MATICA MARAMUREŞ”Delegaţia judeţului Maramureş a fost formată din: Inspector școlar pentru matematică prof. Maiorescu Gheorghe; prof. Fărcaş Natalia, C.N. „V. Lucaciu” Baia Mare; prof. Bojor Florin, prof. Boroica Gheorghe, prof. Heuberger Dana, prof. Heuberger Cristian, prof. Mușuroia Nicolae, prof. Pop Adrian, C.N. „Gh. Şincai” Baia Mare; prof. Bretan Andrei, Şc.Gim. „N. Iorga” Baia Mare; prof. Giurgi Vasile, C.N. „D. Vodă” Sighetu MarmațieiElevii din Maramureș au obținut următoarele rezultate la disciplina matematică:Premiul I: Taploș Carina, clasa a VI-a, Sc.Gim. „ N. Iorga”, prof. Bretan Andrei; Zelina Paul, clasa a VIII-a, C.N. „V. Lucaciu”, prof. Boroica Gabriela; Zelina Mihai, clasa a X-a, C.N. „V. Lucaciu”, prof. Boroica Gabriela. Premiul II: Robu Vlad, clasa a VII-a, Sc. Gim. „N. Iorga”, prof. Bretan Andrei; Lucaciu Sergiu, clasa a IX-a, C.N. „ Gh. Sincai”, prof. Heuberger Cristian. Premiul III: Dragoș Andreea, clasa a V-a, C.N. „V. Lucaciu”, prof. Covaciu Traian; Zaharie Oana, clasa a VI-a, Sc. Gim. „ N. Iorga”, prof. Bretan Andrei; Pop Călin, clasa a VII-a, Sc.Gim. „N. Iorga”, prof. Bretan Andrei. Mentiuni: Zlampareț George, clasa a V-a, C.N. „ Gh. Sincai”, prof. Musuroia Nicolae; Becsi Paul, clasa a VII-a, C.N. „ Gh. Sincai”, prof. Bojor Florin; Boroica Adrian, clasa a VII-a, C.N. „ Gh. Sincai”, prof. Bojor Florin; Matei Bledea Alexandru, clasa a VIII-a, C.N.”Gh.Sincai”, prof.Musuroia Nicolae; Mureșan Ioan, clasa a VIII-a, C.N. „Gh. Sincai”, prof. Musuroia Nicolae; Butnar Adrian, clasa a XI-a, C.N. „Gh. Sincai”, prof. Musuroia Nicolae; Bud Cristian, clasa a XII-a, C.N. „Gh. Sincai”, prof. Heuberger Dana. Premiul special pentru punctaj maxim la sectiunea liceu Zelina Mihai, clasa a X-a, C.N. „V. Lucaciu”, prof. Boroica Gabriela.

25

Page 26: SOCIETATEA DE ŞTIINŢE MATEMATICE DIN - ISJ … matematica 2015... · Web viewpentru punctaj maxim la sectiunea liceu Zelina Mihai, clasa a X-a, C.N. „V. Lucaciu”, prof. Boroica

CONCURSUL INTERJUDEŢEAN DE MATEMATICĂ „DIMITRIE POMPEIU”

Ediţia a XV-a, 8-10 mai 2015, Botoşani

Locul de desfăşurare: C.N. „M. Eminescu”, BotoşaniProf. însoţitori: Fărcaş Natalia şi Zlampareţ Horia, C.N. „V. Lucaciu” Baia MareElevii din judeţul Maramureş au obţinut următoarele rezultate:Clasa a V-a: Menţiune I: Lazea Darius (C.N. „Gh. Şincai” Baia Mare); Menţiuni: Gulin Tudor, Zlampareț George (C.N. „Gh. Şincai” Baia Mare); Dragoş Andreea, Robu Raluca, Trif Cristina, Pop Andreea, Ştirbu Silvia (C.N.”V. Lucaciu” Baia Mare)Clasa a VI-a: Menţiune I: Ciceu Denis (Şc.Gim. „N. Iorga” Baia Mare); Menţiuni: Talpoş Carina, Zaharie Oana (Şc.Gim. „N. Iorga” Baia Mare); Turda Raul (C.N. „Gh. Şincai” Baia Mare); Săsăran Tania (C.N.”V. Lucaciu” Baia Mare); Participare: Riglea Teodora (C.N. „Gh. Şincai” Baia Mare)Clasa a VII-a: Premiul I: Boroica Adrian (C.N. „Gh. Şincai” Baia Mare); Premiul II: Becsi Paul (C.N. „Gh. Şincai” Baia Mare); Menţiuni: Francioli Daria (C.N. ”V. Lucaciu” Baia Mare); Onea Vlad Florin (Şc.Gim. „N. Iorga” Baia Mare); Premiul special: Francioli Daria (C.N.”V. Lucaciu” Baia Mare); Onea Vlad Florin (Şc.Gim. „N. Iorga” Baia Mare)Clasa a VIII-a: Premiul II: Matei Bledea Alexandru (C.N. „Gh. Şincai”,Baia Mare); Premiul III: Mureşan Ioan Alexandru (C.N. „Gh. Şincai” Baia Mare); Menţiuni: Zelina Paul, Diaconescu Malina, Petz Alin (C.N.”V. Lucaciu” Baia Mare); Premiul special: Matei Bledea Alexandru (C.N. „Gh. Şincai” Baia Mare)Clasa a IX-a: Menţiuni: Lucaciu Sergiu (C.N. „Gh. Şincai” Baia Mare); Participare: Hagău Iulian (C.N. „Gh. Şincai” Baia Mare)Clasa a X-a: Premiul I: Zelina Mihai (C.N.”V. Lucaciu” Baia Mare); Premiul II: Sântejudean Tudor (C.N. „Gh. Şincai” Baia Mare)Clasa a XI-a: Menţiuni: Butnar Adrian, Cotan Paul (C.N. „Gh. Şincai” Baia Mare)

CONCURS REGIONAL DE MATEMATICĂ APLICATĂ ÎN ECONOMIE „ECOMAT”

Ediţia a IV-a, 23 mai 2015, Cluj-Napoca

La concurs au participat elevi de la Colegiul Economic „N. Titulescu” obţinând următoarele rezultate: Premiul II: Herczeg Adrian, clasa a XI-a; Mențiune I: Pop Darius Cătălin, clasa a XII-a; Nastai Sorina Ana-Maria, clasa a X-a; Mențiune III: Marchiş Alexandru, clasa a X-a; Mențiune IV: Tohătan Adrian, clasa a X-a;Elevii: Mureşan Andreea Florentina, clasa a IX-a; Guţ Simina Florina, clasa a X-a; Kis Alexandra Mihaela, clasa a X-a; Dicsi Raynold, clasa a X-a; Roman Aurel Mihai, clasa a XI-a s-au situat în jumătatea superioară a clasamentului.

26

Page 27: SOCIETATEA DE ŞTIINŢE MATEMATICE DIN - ISJ … matematica 2015... · Web viewpentru punctaj maxim la sectiunea liceu Zelina Mihai, clasa a X-a, C.N. „V. Lucaciu”, prof. Boroica

CONCURSUL INTERJUDEŢEAN „TEODOR TOPAN”Ediţia a IX-a, 13 decembrie 2014, Şimleu Silvaniei

Locul de desfăşurare: Șimleu SilvanieiProf. însoţitori: Bojor Meda, C.N. „Gh. Şincai” Baia Mare și Giurgi Vasile, C.N. „D.Vodă” Sighetu MarmaţieiElevii din judeţul Maramureş au obţinut următoarele rezultate:Clasa a V-a: Menţiune: Marina-Fiţ Andrei (C.N. „D. Vodă” Sighetu Marmaţiei); Participare: Tămaş Iulia, Boroica Denisa Maria (C.N. „D. Vodă”,Sighetu Marmaţiei)Clasa a VI-a: Participare: Marina-Fiţ Paul, Lihet Cătălin, Ivanciuc Dragoş (C.N. „D. Vodă” Sighetu Marmaţiei)Clasa a VII-a: Menţiuni: Ţiplea Ştefan (C.N. „D. Vodă” Sighetu Marmaţiei); Participare: Gaborean Marcel (C.N. „D. Vodă” Sighetu Marmaţiei)Clasa a VIII-a: Premiul III: Cotârlan Codrin (C.N. „D. Vodă” Sighetu Marmaţiei)Clasa a IX-a: Premiul I: Tămâian Andrei (C.N. „Gh. Şincai” Baia Mare); Premiul III: Horț Iulia (C.N. „Gh. Şincai” Baia Mare); Menţiuni: Roşca Oana (C.N. „D. Vodă” Sighetu Marmaţiei); Participare: Cudrici Carina (C.N. „D. Vodă” Sighetu Marmaţiei), Bojor Barbu (C.N. „Gh. Şincai” Baia Mare)Clasa a X-a: Premiul I: Zelina Mihai (C.N. „V. Lucaciu” Baia Mare)Participare: Crăciun George, Sântejudean Tudor (C.N. „Gh. Şincai” Baia Mare); Vraja Iulian, Ilieş Andreea (C.N. „D. Vodă” Sighetu Marmaţiei),Clasa a XI-a: Premiul III: Cotan Paul (C.N. „Gh. Şincai” Baia Mare); Menţiuni: Butnar Adrian (C.N. „Gh. Şincai” Baia Mare); Participare: Păcurar Aralda, Danci Bianca, Pop Darius (C.N. „D. Vodă” Sighetu Marmaţiei); Tînțar Oana (C.N. „Gh. Şincai” Baia Mare)Clasa a XII-a: Premiul II: Bud Cristian (C.N. „Gh. Şincai” Baia Mare)

CONCURSUL INTERJUDEŢEAN DE MATEMATICĂ „AL. PAPIU ILARIAN”

Ediţia a XIX-a, 2014 Târgu- Mureș

Prof. însoţitor: Heuberger Dana, C.N. „Gh. Şincai” Baia Mare

Elevii din judeţul Maramureş au obţinut următoarele rezultate:Clasa a IX –a: Menţiune specială: Tămâian Andrei (C.N. „Gh. Şincai” Baia Mare)Clasa a X-a: Menţiune: Sântejudean Tudor (C.N. „Gh. Şincai” Baia Mare)Clasa a XI-a: Menţiune specială: Cotan Paul (C.N. „Gh. Şincai” Baia Mare); Menţiune: Butnar Adrian (C.N. „Gh. Şincai” Baia Mare)Clasa a XII-a: Menţiune specială: Bud Cristian (C.N. „Gh. Şincai” Baia Mare); Menţiune: Miclea Andrei (C.N. „Gh. Şincai” Baia Mare)

27

Page 28: SOCIETATEA DE ŞTIINŢE MATEMATICE DIN - ISJ … matematica 2015... · Web viewpentru punctaj maxim la sectiunea liceu Zelina Mihai, clasa a X-a, C.N. „V. Lucaciu”, prof. Boroica

CONCURSUL INTERJUDEŢEAN DE MATEMATICĂ „ȚIGANETEA”Ediţia a XV-a, 23 mai 2015, Dej

Prof. însoţitori: Heuberger Cristian, C.N. „Gh. Şincai” Baia Mare, Zetea Bogdan, Şc. Gim. „G. Coşbuc” Sighetu Marmaţiei, Zetea Teodora, C.N. „D. Vodă” Sighetu Marmaţiei, Ienuțaș Vasile, Şc. Gim. „G. Coşbuc” Baia MareElevii din judeţul Maramureş au obţinut următoarele rezultate:Clasa a V-a: Premiul I: Zlampareț George (C.N. „Gh. Şincai” Baia Mare); Premiul II: Lazea Darius (C.N. „Gh. Şincai” Baia Mare); Menţiuni: Pop Andreea Maria, Dragoş Andreea, Filip Radu, Ştirbu Silvia, Robu Raluca (C.N.”V. Lucaciu” Baia Mare); Gulin Tudor (C.N. „Gh. Şincai” Baia Mare); Mecea Corina (Şc.Gim. „G. Coşbuc” Baia Mare)Clasa a VI–a: Premiul III: Turda Raul (C.N. „Gh. Şincai” Baia Mare); Menţiuni: Ciceu Denis, Talpoş Carina, Zaharie Oana (Şc.Gim. „N. Iorga” Baia Mare); Rad Vlăduţ, Ilieş Bogdan (Şc.Gim. „G. Coşbuc” Sighetu Marmaţiei); Riglea Teodora (C.N. „Gh. Şincai” Baia Mare)

CONCURSUL „GH. ŞINCAI PENTRU MICII MATEMATICIENI”Ediţia a VI-a, 29 aprilie 2015

Organizatori: Catedra de matematică a Colegiului Naţional „Gh. Şincai” şi Asociaţia „Argument”Locul de desfăşurare: Colegiul Naţional „Gh.Şincai”Perioada: 29 aprilie 2015Participanţi: 94 elevi de clasa a IV-a de la şcolile gimnaziale din oraşPremii: Premiul I: Biriş Marc (Şc.Gim. „N. Stănescu”, înv. Pop Luminiţa); Muntean Tudor (Şc.Gim. „N. Iorga”, înv. Covaciu Viorica); Rus Tudor (Şc.Gim. „N. Iorga”, înv. Tarţa Diana); Premiul II: Tiut Cristian (Şc.Gim. „G. Coşbuc”, înv. Dragoş Maria); Premiul III: Marchiş Marcus (Şc. Gim. „D. Cantemir”, înv. Birle Delia), Gheţe Ruxandra (Şc. Gim. „A. Iancu”, înv. Coza Ioana)

AMERICAN MATHEMATICS COMPETITION

AMC este organizată pe trei nivele: AMC 8 – pentru şcoală generală, AMC 10 – pentru clasele IX-X şi AMC 12- pentru clasele XI-XII. Concursul propriu zis are loc în România, dar lucrările se corectează în America. De acolo vin şi diplomele şi insignele pentru cei care se clasează în top. În Maramureş s-a organizat la Colegiul Național „Vasile Lucaciu” și au participat câte 40 de elevi, pentru şcoală generală şi liceu. AMC 8

28

Page 29: SOCIETATEA DE ŞTIINŢE MATEMATICE DIN - ISJ … matematica 2015... · Web viewpentru punctaj maxim la sectiunea liceu Zelina Mihai, clasa a X-a, C.N. „V. Lucaciu”, prof. Boroica

Locul I, cu diplomă şi insignă: Alexandru Matei Bledea, CN „Gheorghe Şincai”; Locul II: Nicolae Moldovan, Şcoala „George Coşbuc” ; Locul III: Paul Becsi, CN „Gheorghe Şincai” şi Paul Zelina, C.N. „Vasile Lucaciu”. Honor Roll of Distinction, pentru clasarea între primii 1% performer: Matei Alexandru Bledea şi Nicolae Moldovan (la AMC participă elevi din toată lumea). Honor Roll Certificate: Alţi 11 elevi s-au clasat printre primii 5% performeri din lume: Paul Becsi, CN „Gheorghe Şincai”; Paul Zelina, CN „Vasile Lucaciu”; Codrin Cotârlan, C.N. „Dragoş Vodă” Sighetu Marmaţiei; Alin Petz, CN. „Vasile Lucaciu”; Călin Pop, Şc. Gim. „N. Iorga”; Dacian Stepan, Şc. Gim. „George Coşbuc”, Sighetu Marmaţiei; Ştefan Ţiplea, C.N. „Dragoş Vodă”; Alexandru Mureşan, Ioana Mercea, Remus Ghişa, toţi de la CN „Gheorghe Şincai”; Alexandru Breban, Șc. Gim. „N. Stănescu”. Certificate of achievement: Erik Biriş şi Radu Herzal, CN „Gheorghe Şincai” şi Laurenţiu Lazăr, C.N. „D. Vodă” Sighetu Marmaţiei. AMC 10: Mihai Zelina, Certificate of Distinction, cu insignă; AIME: Mihai Zelina, Certificate of participation; AMC 12: Locul I, cu insignă – Adrian Butnar, CN. „Gheorghe Şincai”

CONCURSUL MATE-INFOEdiţia a II-a, 16 mai 2015, Baia Mare

Organizator: Departamentul de Matematică şi Informatică, Facultatea de Ştiinţe, Centrul Universitar Nord Baia Mare PREMII:Premiul I: Gotha Guntter, Lic. Teoretic ,,Németh László” Baia Mare; Premiul II: Ciople Laszlo, Liceul Teoretic ,,Németh László” Baia Mare;Premiul III: Timiș Adrian, C. N. „Gh. Şincai” Baia Mare; Menţiuni: Rus Alexandru, C.N. „V. Lucaciu” Baia Mare; Marian Sergiu Mihai, C.N. „Gh. Şincai” Baia Mare şi Petruș Raul, C.N. „Gh. Şincai” Baia Mare.

Pe lângă premiile acordate, concursul a oferit, în plus, un avantaj semnificativ pentru elevii care, după obţinerea diplomei de bacalaureat, se vor înscrie la Facultatea de Ştiinţe, Centrul Universitar Nord din Baia Mare, la specializările coordonate de către Departamentul de Matematică şi Informatică, specializările Matematică informatică, Informatică, Informatică economică. Nota obţinută la concursul MATE-INFO Baia Mare va putea fi folosită la calculul mediei de admitere, în locul notei obţinute la bacalaureat la disciplinele matematică sau informatică, notă care, alături de media generală de la bacalaureat, este luată în considerare la calculul mediei de admitere. 

În plus, începând cu ediţia a 2-a a concursului MATE-INFO Baia Mare, se acordă premii speciale, sub forma unei burse de instalare, acelor participanţi la concursului MATE-INFO Baia Mare care, în urma concursului de admitere, au fost înmatriculaţi ca studenţi la una dintre specializările Matematică informatică, Informatică, Informatică economică, de la Facultatea de Ştiinţe, C. Universitar Nord din Baia Mare. Bursa de instalare, în valoare de 400 lei, se va acorda în luna

29

Page 30: SOCIETATEA DE ŞTIINŢE MATEMATICE DIN - ISJ … matematica 2015... · Web viewpentru punctaj maxim la sectiunea liceu Zelina Mihai, clasa a X-a, C.N. „V. Lucaciu”, prof. Boroica

octombrie 2015 acelor participanţi la concursul MATE-INFO Baia Mare, care au obţinut cel puţin media 8 în concurs.

CONCURSUL INTERNAȚIONAL AL LICEELOR MAGHIAREEdiția a XXIV-a, 5-12 aprilie 2015, Serbia- Subotica

Mențiune: Gotha Guntter și Gyarmathy Timea, Lic.Teor. „Nemeth Laszlo” Baia Mare, prof. Longaver Ludovic

CONCURSUL DE TINERE TALENTE ÎN CERCETARE ȘI INOVAREUngaria 2015

Premiul III: Gotha Guntter cu lucrarea „ Îmbunătățirea inegalității Hadwinger-Finsler” de la Lic.Teor. „Nemeth Laszlo” Baia Mare, prof. Longaver LudovicMențiune: Bakk Eszter cu lucrarea „Aplicațiile determinanților în geometria analitică” de la Lic.Teor. „Nemeth Laszlo” Baia Mare, prof. Zakany Monika

SESIUNEA INTERJUDEŢEANĂ DE REFERATE ŞI COMUNICĂRI ŞTIINŢIFICE ALE ELEVILOR „ FATĂ-N FAŢĂ CU ADEVĂRUL”

EDIŢIA A XV-A, 13 XII 2014

Loc de desfășurare: Liceul Teoretic „ Emil Racoviţă”Colectivul de organizare: prof. Moldovan Ana Lucreţia Maria, inspector general ISJMM; prof. univ. dr. Berinde Vasile, preşedinte Filiala Maramureș a S.S.M.R; prof. Pop Mariana, director Liceul Teoretic ,,Emil Racoviţă”; prof. Ilieşiu Felicia, director adjunct Liceul Teoretic „Emil Racoviţă”; prof. Maiorescu Gheorghe, inspector de matematica ISJMM; prof. Ardelean Codruţa, Liceul Teoretic ,,Emil Racoviţă”; prof. Cioclu Costel, Liceul Teoretic ,,Emil Racoviţă”; prof. Huminiuc Monica, Liceul Teoretic ,,Emil Racoviţă”; prof. Marian Claudia, Liceul Teoretic ,,Emil Racoviţă”; prof. Mădăras Iulia, Liceul Teoretic ,,Emil Racoviţă”; prof. dr. Petrean Liviu, Liceul Teoretic ,,Emil Racoviţă”, prof. Moanţa Florin, Liceul Teoretic ,,Emil Racoviţă”; prof. Ocean Cristina, Liceul Teoretic ,,Emil Racoviţă”; prof. Podina Camelia, Liceul Teoretic ,,Emil Racoviţă”Parteneri: Consiliul Judeţean Maramureş, Consiliul Local Baia Mare, Societatea de Ştiinţe Matematice-Filiala MaramureşREZULTATELE OBŢINUTECLASA a VII-a: PREMIUL II: Şugar Maria Dorina şi Ghiran Melania, Lic. de Arte Baia Mare, prof. coord. Bunu Iulian; PREMIUL III: Tomoiagă Maria şi Tomoiagă Andrei, Şc.Gim. nr.1 Moisei, prof. coord. Şteţcu Mia; CLASA a VIII-a: PREMIUL I: Petz Alin şi Zelina Paul, C.N. „V. Lucaciu” Baia Mare, prof.coord: Boroica Gabriela; PREMIUL II: Stepan Dacian, Şc.Gim. „G. Coşbuc” Sighetu Marmaţiei, prof. coord. Zetea Bogdan; PREMIUL III: Mih Mara, Şc. Gim. „G. Coşbuc” Sighetu Marmaţiei, prof. coord. Zetea Bogdan; MENŢIUNI: Andercău Florina şi Chiş Narcisa, Şc.Gim. „Vasile Alecsandri” Baia Mare, prof.

30

Page 31: SOCIETATEA DE ŞTIINŢE MATEMATICE DIN - ISJ … matematica 2015... · Web viewpentru punctaj maxim la sectiunea liceu Zelina Mihai, clasa a X-a, C.N. „V. Lucaciu”, prof. Boroica

coord. Pop Sever; Ardelean Michael, Şc.Gim. „Vasile Alecsandri” Baia Mare, prof. coord. Sălăjan Raluca; Macovei Mădălina şi Tiseliţă Ioana, L.T. „Octavian Goga” Rozavlea, prof. coord. Bledea Lidia; Coman Lucian şi Ştefancu Dragoş, Şc.Gim. nr.1 Moisei, prof. coord. Urda MariaCLASA a IX-a M1: PREMIUL I : Rus Beatrice şi Damşa Ioan Dinu, C.N. „Gh. Şincai”Baia Mare, prof. coord.: Pop Adrian; PREMIUL II: Tămâian Andrei şi Buciuman Adrian, C.N. „Gh. Şincai” Baia Mare, prof. coord.: Petruţiu Crina; PREMIUL III:Puşcaşu Iulia şi Gheorghe Mihai, C.N. „A. Mureşanu” Dej, prof. coord. Magdaş Camelia; MENŢIUNI SPECIALE: Sălăjan Alexandra şi Meţac Anda, C.N. „Gh. Şincai” Baia Mare, prof. coord. Petruţiu Crina; Moldovan Gabriel şi Grigorescu Ionuţ, Lic.Teor. „E. Racoviţă” Baia Mare, prof. coord.Ocean Cristina; Marian Bianca şi Porumb Bogdan, C.N. ,,V. Lucaciu” Baia Mare, prof. coord.Fărcaş Natalia; Ardelean Andrei, Lic.Teor. ,,E. Racoviţă” Baia Mare, prof. coord. Ocean Cristina; Paşca Paul şi Tămaş Bogdan, C.N. „Gh. Şincai” Baia Mare, prof. coord.Petruţiu Crina; Pop Vădean Iulia şi Tamas Miruna , Lic.Teor. ,,E. Racoviţă” Baia Mare, prof. coord. Ocean Cristina; Nicoară Roxana şi Barz Tania , C.N. „Gh. Şincai” Baia Mare, prof. coord. Petruţiu Crina; Verdeş Alexandra şi Chelaru Ştefania, C.N. „Gh. Şincai” Baia Mare, prof. coord. Petruţiu CrinaCLASA a IX-a M2: PREMIUL I: Rodila Mădălina şi Dragoş Alexandra, C.T. „C.D. Neniţescu” Baia Mare, prof. coord. Balogh Erika; PREMIUL II: Rus Cătălin şi Gheţi Damian, C.T. „A.P. Ilarian” Zalău, prof, coord. Matyas Mirel; PREMIUL III: Marian George şi Sabou Gabriel, C.T. „A.P. Ilarian” Zalău, prof. coord. Matyas Mirel; MENŢIUNI SPECIALE: Sergiu Fălcuşan, C.T. „Transilvania” Baia Mare, prof. coord. Seciu Mihaela; Hosu Gabriel şi Marincaş Voicu, C.T. „A. Saligny” Baia Mare, prof. coord. Şerba Lucia; Pop Anamaria şi Puşcaş Daniela, Liceul Teologic Penticostal Baia Mare, prof. coord. Boitor Ramona; Iunian Constantin şi Filimon Bogdan, C.T. „A. Saligny” Baia Mare, prof. coord. Brisc Viorica; Comiati Denis şi Borota Marius, C.T „A. Saligny” Baia Mare, prof. coord. Şerba Lucia; MENŢIUNI: Zsiga Erik şi Ardelean Cristian, C.T. „A. Saligny” Baia Mare, prof. coord. Şerba Lucia.CLASA a X-a M1: PREMIUL I: Sandu Ioana şi Mihali Mara, C.N. „V. Lucaciu” Baia Mare, prof. coord. Boroica Gabriela; PREMIUL II: Dodon Alex şi Bărbos Rareş, Co.N. „A. Mureşanu” Dej, prof. coord. Jecan Eugen; PREMIUL III: Bonta Alexandra şi Cadar Ana, C.N. „V. Lucaciu” Baia Mare, prof. coord. Boroica Gabriela; MENŢIUNI SPECIALE: Mărginean Puiu şi Mureşan Bianca, C.N. „A. Mureşanu” Dej prof. coord. Jecan Eugen; Jecan Monica şi Hegheduş Raluca, C.T. „A. Papiu Ilarian” Zalău, prof. coord. Sîrb Vasile; Ţineghe Cezara, Lic.Teor. „E. Racoviţă” Baia Mare, prof. coord. Ocean Cristina; Pitaru Robert şi Terec Alexandru, C.N. „Gh. Şincai” Baia Mare, prof. coord. Petruţiu CrinaCLASA a X-a M2: PREMIUL I: Michiş Daliana şi Butica Luiza, C.N. „V. Lucaciu” Baia Mare prof. coord. Fărcaş Natalia; PREMIUL II: Rădăuceanu Andra şi Botoi Roxana, C.N. „Gh. Şincai” Baia Mare, prof. coord. Petruţiu Crina; PREMIUL III: Mâţ Ionela şi Temple Alexandra, C.N. „V. Lucaciu” Baia Mare, prof. coord. Fărcaş Natalia; MENŢIUNI SPECIALE: Pop Dora şi Niţulescu Iris, C.N. „Gh. Şincai” Baia

31

Page 32: SOCIETATEA DE ŞTIINŢE MATEMATICE DIN - ISJ … matematica 2015... · Web viewpentru punctaj maxim la sectiunea liceu Zelina Mihai, clasa a X-a, C.N. „V. Lucaciu”, prof. Boroica

Mare, prof. coord. Petruţiu Crina; Danci Stefania şi Mariş Adriana, C.N. „Gh. Şincai” Baia Mare, prof. coord. Petruţiu Crina; Dumitraşcu Bianca şi Caraszan Larisa, Lic.Teor. „E. Racoviţă” Baia Mare, prof. coord. Petrean Liviu; Ardelean Alexandra şi Sălăjan Flavia, C.T. „A. Papiu Ilarian” Zalău, prof. coord. Sîrb Vasile; Zlampa Natalia şi Domnita Diana, C.N. „Gh. Şincai” Baia Mare, prof. coord. Petruţiu Crina; MENŢIUNI: Sălăjan Mădălina şi Sas Ştefania, C.N. „Gh. Şincai” Baia Mare, prof. coord. Petruţiu Crina; Pop Carla şi Mărieş Ioana, Lic.Teor. „E. Racoviţă” Baia Mare, prof. coord. Petrean LiviuCLASA a XI-a M1: PREMIUL I: Sortan Raluca,Balog Bogdan, C.T. „A. Papiu Ilarian” Zalău, prof. coord. Sîrb Vasile; PREMIUL II: Avram Lara, C.N. „Gh. Şincai” Baia Mare, prof. coord. Heuberger Dana; PREMIUL III: Ungureanu Radu şi Voit Radu, C.N. „Gh. Şincai” Baia Mare, prof. coord. Muşuroia Nicolae; MENŢIUNE SPECIALĂ: Bonta Florin şi Barbos Sergiu, C.N. „A. Mureşanu” Dejprof. coord. Jecan EugenCLASA a XI-a M2: PREMIUL I: Timiş Raul şi Mureşan Denis, Seminarul Teologic Liceal „Sf. Iosif Mărturisitorul” Baia Mare, prof. coord. Pop Radu;PREMIUL II :Zăgrean Alexandra şi Pop Gabriel, C.T. „George Bariţiu” Baia Mare, prof. coord. Pop Anca; PREMIUL III: Buciuman Vanessa şi Tomoiagă Daniela, Liceul Teologic Penticostal Baia Mare, prof. coord. Tarău Rodica; MENŢIUNI SPECIALE: Micaş Vasile, C.T. „A. Saligny” Baia Mare, prof. coord. Şerba Lucia; Pop Tabita şi Novacovici Andreea, Liceul Teologic Penticostal Baia Mare, prof. coord. Tarău Rodica; Dincă Natalia şi Szasz Robert, Lic.Teor. „E. Racoviţă” Baia Mare, prof. coord. Petrean Liviu; Chindriş Mihai şi Feidi Denisa, Lic.Teor. „E. Racoviţă” Baia Mare, prof. coord. Petrean Liviu; David Andreea şi Zele Loredana, Lic.Teor. „E. Racoviţă” Baia Mare, prof. coord. Moanţă FlorinCLASA a XII-a M2: PREMIUL I: Copceac Raul şi Buciu Andrei, C.T. „A. Papiu Ilarian” Zalău, prof. coord. Matyas Mirel; PREMIUL II:Gheţie Ioana şi Ferenţi Ionuţ, Lic.Teor. „E. Racoviţă” Baia Mare, prof. coord. Podina Camelia; PREMIUL III: Filip Nicoleta şi Crăciunaş Alexandra, C.T. „A. Vlaicu” Baia Mare, prof. coord. Mic Leon; MENŢIUNI SPECIALE: Cosma Sergiu, L.T. „Someş”, Dej, prof. coord. Pugna Georgiana; Molnar Silviu şi Filip Alex, C.T. „A. Saligny” Baia Mare, prof. coord. Brisc Viorica; Pop Gabriela, Lic.Teor. „E. Racoviţă” Baia Mare, prof. coord. Mădăras Iulia; Brânduşe Ioan şi Cosma Andrei, C.T. „A. Saligny” Baia Mare, prof. coord. Birta Adriana; Gheţe Mădălina şi Marco Adelin, C.Ec. „N.Titulescu” Baia Mare, prof. coord. Horge Daniel; MENŢIUNI : Bledea Vasile şi Mureşan Cosmin, C.T. „A.Saligny” Baia Mare, prof. coord. Birta Adriana; Pop Dănuţ şi Vasian Eugen, C.T. „A. Saligny” Baia Mare, prof. coord. Birta AdrianaCLASA a XII-a M1: PREMIUL I:Nicolaescu Andrei, C.N. „Gh. Şincai” Baia Mare, Prof. coord. Heuberger Dana; PREMIUL II: Şimon Gheorghe Ionut şi Alb Vasile Cosmin, Lic.Teor. „Bogdan Vodă” Vişeu de Sus, prof. coord. Vesel-Pop Floare-Ioana; PREMIUL III: Ciurte Tudor şi Rus Elena, C.N. „Gh. Şincai” Baia Mare, prof. coord. Boroica Gheorghe; MENŢIUNE SPECIALĂ: Ciople Laszlo şi Zavaczki Peter, Lic.Teor. „N. Laszlo” Baia Mare, prof. coord. Longaver Ludovic.

32

Page 33: SOCIETATEA DE ŞTIINŢE MATEMATICE DIN - ISJ … matematica 2015... · Web viewpentru punctaj maxim la sectiunea liceu Zelina Mihai, clasa a X-a, C.N. „V. Lucaciu”, prof. Boroica

La sesiune au participat 92 elevi coordonaţi de 60 de profesori; un număr de 30 de profesori au fost membrii în cele 8 birouri ale secţiunilor sesiunii.

Din cele 101 de lucrări prezentate s-au acordat premii astfel: 9 premii I, 10 premii II, 10 premii III, 43 de menţiuni.

LISTA METODIȘTILOR DIN JUDEȚUL MARAMUREȘ LA MATEMATICĂ2014-2015

Maiorescu Gheorghe, Inspector școlar pentru matematicăȘerba Lucia, Colegiul Tehnic „A. Saligny” Baia MareBrisc Viorica, Colegiul Tehnic „Anghel Saligny” Baia MareMastan Eliza, Liceul Teoretic „Nemeth Laszlo” Baia MareBoga Ovidiu, Liceul Teoretic „Gr. Moisil” Târgu LăpușGrigor Mihai, Liceul Teoretic „Marmația” Sighetu MarmațieiPuț Liliana, Liceul Pedagogic „Regele Ferdinand” Sighetu MarmațieiTomoiagă Ion, Școala Gimnazială Vișeu de JosPop Anca, Colegiul Ttehnic „George Bariţiu” Baia MareRus Ancuța, Colegiul Tehnic „George Bariţiu” Baia MarePopovic Ioana, Școala Gimnazială „Octavian Goga” Baia MareMihali Marinela, Liceul BorșaMureșan Daniela, Liceul Teoretic „Ioan Buteanu” Șomcuta MareBorcut Marin, Colegiul Economic „Pintea Viteazul” Cavnic

MEMBRII CONSILIULUI CONSULTATIV AL PROFESORILOR DE MATEMATICĂ

Berci Ioan,Colegiul Naţional „Dragoş Vodă” Sighetu MarmaţieiBojor Meda, Colegiul Naţional „Gheorghe Şincai” Baia MareBreitkopf Marieta, Şcoala Gimnazială „Nichita Stănescu” Baia MareBretan Andrei, Şcoala Gimnazială „Nicolae Iorga” Baia MareGherasin Gheorghe, Liceul Teoretic „Regele Ferdinanad” Sighetu MarmaţieiIenuţaş Vasile, Şcoala Gimnazială „George Coşbuc” Baia MareLopată Angela, Școala Gimnazială GârdaniMastan Eliza, Liceul Teoretic „Nemeth Laszlo” Baia MareMic Vasile, Şcoala Gimnazială „George Coşbuc” Sighetu MarmaţieiMihoșca Gavril, Liceul Teoretic „Petru Rareș” Târgu LăpușMihali Marinela, Liceul BorșaPetruțiu Crina, Colegiul Naţional „Gheorghe Şincai” Baia MarePop Vesel Floare, Liceul Teoretic „Bogdan Vodă” Vişeu de Sus,Sabău Ştefan, Colegiul Naţional „Vasile Lucaciu” Baia MareSfara Gheorghe, Colegiul Naţional „Vasile Lucaciu” Baia MareŞerba Lucia, Colegiul Tehnic „Anghel Saligny” Baia MareTemian Gavril, Colegiul Economic „Nicolae Titulescu” Baia Mare

33

Page 34: SOCIETATEA DE ŞTIINŢE MATEMATICE DIN - ISJ … matematica 2015... · Web viewpentru punctaj maxim la sectiunea liceu Zelina Mihai, clasa a X-a, C.N. „V. Lucaciu”, prof. Boroica

Urda Maria, Şcoala Gimnazială Numărul 1 MoiseiBorșa Raul, Colegiul Economic „Nicolae Titulescu” Baia Mare- Observator SLIMMLISTA CADRELOR DIDACTICE CARE AU PRIMIT GRADAŢII DE MERIT

2015

Nr. crt. Nume şi prenume Unitatea şcolară Vechimea în

învăţământ1 Maiorescu Gheorghe Lic.Teor. „Emil Racoviță” Baia Mare 432 Boroica Gheorghe C.N. „Gh. Șincai” Baia Mare 273 Heuberger Dana C.N. „Gh. Șincai” Baia Mare 284 Mușuroaia Nicolae C.N. „Gh. Șincai” Baia Mare 355 Bedeoan Loredana C.N. „D. Vodă” Sighetu Marmației 216 Giurgi Vasile C.N. „D. Vodă” Sighetu Marmației 187 Pop Mariana Lic.Teor. „P. Rareș” Târgu Lăpuș 18

TEMATICA CERCURILOR PEDAGOGICE LA DISCIPLINA MATEMATICĂ

din anul şcolar 2014-2015

Semestrul I (12- decembrie 2014, ora 14)ZONA BAIA MAREGRUPELE I-IV GIMNAZIULoc de desfăşurare: Școala Gimnazială „N. Iorga” Baia MareTematica:

1. Plan de măsuri pentru optimizarea rezultatelor la examenele naţionale, concursurile şi olimpiadele şcolare precum şi la evaluări naţionale- prof. Ienuțaș Vasile

2. Modele de subiecte la Evaluare Națională cls. II, IV, VI și la Evaluare Națională cls.VIII- prof. Ienuțaș Vasile

3. Desfășurarea concursului „Tinere Speranțe”4. Diverse

Responsabili: prof. Boloş Mihai; prof. Bretan Andrei; prof. Pop Marinel; prof. Ienuţaş Vasile; prof. Breitkopf Marieta; prof. Neaga Nadina;prof. Bunu Iulian; prof. Cosma SimonaGRUPA COLEGII/LICEE TEORETICE ŞI TEHNOLOGICE BAIA MARELoc de desfăşurare: Liceul Teoretic „Emil Racoviţă” Baia Mare Data: 13 decembrie 2014 ora 8.30Tematică:

1. Plan de măsuri pentru optimizarea rezultatelor la examenele naţionale, concursurile şi olimpiadele şcolare precum şi la evaluări naţionale- prof. Podină Camelia

34

Page 35: SOCIETATEA DE ŞTIINŢE MATEMATICE DIN - ISJ … matematica 2015... · Web viewpentru punctaj maxim la sectiunea liceu Zelina Mihai, clasa a X-a, C.N. „V. Lucaciu”, prof. Boroica

2. Probleme cu inegalități din variante de bacalaureat- prof. Ocean Cristina3. Tehnici, procedee utilizate în predarea-învățarea matematicii pentru

dezvoltarea gândirii creatoare- prof. Podină Camelia4. Sesiunea interjudeţeană de referate şi comunicări ştiinţifice „Faţă-n faţă cu

adevărul” ediţia XVResponsabili: prof. Muşuroia Nicolae; prof. Şerba Lucia; prof.Temian GavrilZONA SIGHETU MARMAŢIEI

I. GRUPA ȘCOLI GIMNAZIALELoc de desfăşurare: Şcoala Gimnazială „Mihaly de Apşa” Sighetu Marmatiei Tematică:

1. Plan de măsuri pentru optimizarea rezultatelor la examenele naţionale, concursurile şi olimpiadele şcolare, precum şi la evaluări naţionale

2. Rezolvări de probleme din Gazeta Matematică3. Diverse- popularizarea concursului online ptofu’ de mate, adresa:

http://mate.info.ro/News.aspx?ID=775 propuneri de probleme pentru gazeta matematica, anuarul 2014, cotizatia SSM 2014 si 2015

Responsabili: prof. Bizău Ioan; prof. Mic Vasile1. GRUPA LICEULoc de desfăşurare: Liceul ,,Taras Sevcenko’’Tematică:

1. Examenul de Bacalaureat 2015 în viziunea metodologiilor în vigoare2. Probleme care conţin partea întreagă şi partea fracţionară a unui număr

întâlnite la olimpiadele şcolare din Ucraina şi Rusia în anii 2009-2012- prof. Ferţadi Marian Liceul ,,Taras Sevcenko’’ Sighetu Marmaţiei

3. Rezolvări de probleme din G.M.11/2014- prof. Gherasin Gheorghe, Liceul Pedagogic ,,Regele Ferdinand’’Sighetu Marmaţiei

4. DiverseResponsabil: prof. Gherasin Gheorghe

ZONA VALEA IZEILoc de desfăşurare: Grupul Școlar „Octavian Goga” RozavleaTematică:

1. ,,Probleme practice de geometrie-aspecte metodico- științifice” –referat susținut de prof. Vlad Marie și prof. Chiș Maria-Marioara de la Școala Gimnazială ,,Mihai Eminescu’’ Sălistea de Sus

2. Lecții deschise susținute de prof. Bledea Livia (clasa a VI-a ,,Elemente de geometrie. Figuri geometrice”) și prof. Berinde Vasile( clasa a XI-a, ,,Matrici și determinanți- recapitulare”), Grupul Școlar ,,Octavian Goga” Rozavlea

Responsabili: prof. Chiș Maria-Marioara; prof. Borodi ElenaZONA LĂPUŞLoc de desfăşurare: Școala Gimnazială Copalnic Mănăștur, structura Preluca NouăTematică:

1. Plan de măsuri privind creșterea rezultatelor la examenele naționale, concursurile și olimpiadele școlare

35

Page 36: SOCIETATEA DE ŞTIINŢE MATEMATICE DIN - ISJ … matematica 2015... · Web viewpentru punctaj maxim la sectiunea liceu Zelina Mihai, clasa a X-a, C.N. „V. Lucaciu”, prof. Boroica

2. Rezolvări de probleme din Gazeta Matematică - prof Ovidiu Bobb 3. Lecție deschisă- clasa a VIII-a ,,Operații cu rapoarte algebrice”- prof. Mariana

Costin4. Diverse

Responsabili: prof. Boga Ovidiu; prof. Roman MarianaZONA CODRULoc de desfășurare: Școala Gimnazială SălsigTematica:

1. Analiza rezultatelor la Examenul de Evaluare Națională 2014 ,olimpiade și concursuri școlare, disciplina matematică, la nivel zonal

2. Plan de măsuri pentru optimizarea rezultatelor la examenele naţionale, concursurile şi olimpiadele şcolare precum şi la evaluări naţionale.

3. Asupra unor probleme din Gazeta Matematică- prof Chivu Gheorghe, Școala Gimnazială Sălsig, prof. Pop Monica, Școala Gimnazială „Lucian Blaga” Fărcașa

4. DiverseResponsabil: prof. Lopată Angela, prof. Pop TraianZONA BORŞA- VIŞEU LICEULoc de desfăşurare: Liceul BorșaTematică:

1. Plan de măsuri pentru optimizarea rezultatelor la examenele naționale (Bacalaureat și Evaluare Naţională), precum și la concursurile și olimpiadele școlare- prof. Vesel-Pop Floarea, prof. Pintea Ioan, prof. Țicală Lenuța, prof. Șimon Paula

2. Rezolvări de probleme din Gazeta Matematică- prof. Mihali Marinela, prof. Chindriș Lazăr, prof. Rad Daniel

3. Modele subiecte Bacalaureat și Evaluare Națională- prof. Timiș Lenuța, prof. Pantlik Iosif

Responsabili: prof. Vesel-Pop Floarea; prof. Mihali MarinelaZONA BORŞA-MOISEILoc de desfăşurare: Şcoala Gimnazială Nr. 4 BorşaTematică:

1. Plan de măsuri pentru optimizarea rezultatelor la examenele naţionale, concursurile şi olimpiadele şcolare precum şi la evaluări naţionale

2. Rezolvări de probleme din Gazeta Matematică- prof. Coman ElenaResponsabili: prof. Urda Maria; prof. Coman Ioana ZONA VIȘEULoc de desfăşurare: Liceul Tehnologic Nr.4 Vișeu de SusTematică:

3. Plan de măsuri pentru optimizarea rezultatelor la examenele naționale, concursurile și olimpiadele școlare

4. Abordarea transdisciplinara a matematicii –prof. Petreuș Ioan 5. Rezolvări de probleme din gazeta matematică

36

Page 37: SOCIETATEA DE ŞTIINŢE MATEMATICE DIN - ISJ … matematica 2015... · Web viewpentru punctaj maxim la sectiunea liceu Zelina Mihai, clasa a X-a, C.N. „V. Lucaciu”, prof. Boroica

Responsabil: prof. Tomoiagă Ioan

Semestrul al II-lea (30 aprilie 2015)ZONA BAIA MAREGRUPA 1 ŞCOLI GIMNAZIALE BAIA MARELocul de desfăşurare: Școala Gimnazială nr.18 Baia MareTematică:

1. Situaţia rezultatelor obţinute la simularea Evaluării Naţionale pentru elevii clasei a VIII-a;

2. Planuri de măsuri elaborate la nivelul unităţilor şcolare pentru obţinerea de rezultate bune şi foarte bune la Evaluarea Naţională din iunie 2015 şi creşterea procentului de promovabilitate;

3. Concursurile de matematică şi Olimpiada de matematică-subiecte propuse şi rezultate obţinute;

4. Rezolvarea problemelor folosind metoda reducerii la absurd- prof. Ardelean Codruța, Șc. Gimn. „Al. I. Cuza” Baia Mare

5. Probleme din Gazeta Matematică;6. Diverse

Responsabili: prof. Cosma Simona; prof. Bunu IulianGRUPA 2 ŞCOLI GIMNAZIALE BAIA MARELocul de desfăşurare: Școala Gimnazială „Nicolae Iorga” Baia MareTematica:

1. Prezentarea rezultatelor la Olimpiadele şi Concursurile şcolare – etapa municipală, judeţeană şi naţională – prof. Bretan Andrei

2. Prezentarea de variante de subiecte pentru Evaluarea Naţională clasa a VI-a, clasa a VIII-a, precum şi discutarea simulării Evaluării Naţionale clasa a VIII-a - prof. Boloş Mihai

3. Prezentarea unui model de subiect pentru examenul de titularizare în învăţământ – prof. Codrea Lucica Ioana

Responsabili: prof. Boloş Mihai; prof. Bretan AndreiGRUPA 3 ŞCOLI GIMNAZIALE BAIA MARELocul de desfăşurare: Școala Gimnazială „Dr. V. Babeș” Baia Mare Tematică:

1. Lecție deschisă „Media Aritmetică” – clasa a V-a – profesor Neaga Nadina, Școala Gimnazială „Dr.V. Babeș” Baia Mare;

2. Prezentarea referatului cu titlul „Metode și procedee de rezolvare a problemelor de geometrie în gimnaziu” – profesor Schweighoffer Clara, Școala Gimnazială „Dr. V. Babeș” Baia Mare;

3. Probleme de concurs – profesor Breitkopf Marieta, Școala Gimnazială „N. Stănescu” Baia Mare;

4. DiverseResponsabili de cerc: prof. Breitkopf Marieta; prof. Neaga NadinaGRUPA 4 ŞCOLI GIMNAZIALE BAIA MARE

37

Page 38: SOCIETATEA DE ŞTIINŢE MATEMATICE DIN - ISJ … matematica 2015... · Web viewpentru punctaj maxim la sectiunea liceu Zelina Mihai, clasa a X-a, C.N. „V. Lucaciu”, prof. Boroica

Locul de desfăşurare: Liceul cu Program Sportiv Baia MareTematică:

1. Modalități de prezentare a modificărilor din programa clasei a VIII-a: ecuația de gradul II și corpurile rotunde ;

2. Metode de învățare centrate pe elev utilizate în cadrul orelor de matematică- prof. Ghirasin Silvia, Școala Gimnazială Coaș;

3. Probleme din Gazeta Matematică- prof. Pop Marinel, LPS Baia Mare; prof. Ienuțaș Vasile, Șc. Gim. „G. Coșbuc” Baia Mare;

Responsabili: prof. Pop Marinel; prof. Ienuţaş VasileGRUPA COLEGII/LICEE TEORETICE ŞI TEHNOLOGICE BAIA MARELocul de desfăşurare: Colegiul Național „Gheorghe Șincai” Baia MareTematica:

1. Analiza rezultatelor și discutarea subiectelor de la simularea examenului de bacalaureat și evaluare națională la clasa a VIII-a. Exemple concrete din școlile și liceele arondate- prof. Lucia Șerba, prof. Temian Gavril, prof.  Nicolae Mușuroia 

2. Analiza rezultatelor elevilor din județul Maramureș la O.N.M.- prof. Nicolae Mușuroia 

3. Rezolvarea și comentarea unor probleme de la O.N.M- prof. Gheorghe Boroica, prof. Florin Bojor 

Responsabili: prof. Muşuroia Nicolae; prof. Şerba Lucia; prof.Temian GavrilZONA LĂPUŞLocul de desfăşuare: Liceul Tehnologic „Grigore Moisil” Târgu LăpușTematică:

4. Modele de subiecte pentru examenul de Evaluare Națională5. Referat metodic: Metode de rezolvare a problemelor de aritmetică- prof Maria

Burzo6. Lecție deschisă la clasa a VI-a „Operații cu numere intregi”- prof Ovidiu

Boga7. Diverse

Responsabili: prof. Boga Ovidiu, prof. Roman MarianaZONA VIŞEULocul de deşfăşurare: Școala Gimnazială Nr.7 Vișeu de Sus Tematica:

8. Prezentarea metodologiei de evaluare la matematică la clasa a VI-a și a testelor Pisa

9. Modalități de eficientizare a pregătirii elevilor pentru susținerea tedtelor de Evaluare națională la clasa a VIII-a – prof. Pop Ana- Maria

10. Analiza rezultatelor elevilor la faza județeană a Olimpiadei de MatematicăResponsabil: prof Tomoiagă IoanZONA BORŞA-MOISEILocul de desfășurare : Şcoala Gimnazială Nr. 9 Borşa-Structura Şcoala Gimnazială Nr.3 Borşa

38

Page 39: SOCIETATEA DE ŞTIINŢE MATEMATICE DIN - ISJ … matematica 2015... · Web viewpentru punctaj maxim la sectiunea liceu Zelina Mihai, clasa a X-a, C.N. „V. Lucaciu”, prof. Boroica

Tematică: 1. Subiecte examene naţionale- comentarii metodice asupra modelelor de teste

publicate de MECS2. Consideraţii metodice privind efectuarea unor lucrări practice de geometrie

prin măsurători în teren- prof. Şteţco IoanResponsabili: prof. Urda Maria; prof. Coman IoanaZONA VALEA IZEI Locul de deşfăşurare: Școala Gimnazială StrâmturaTematica:

1. Raport privind ,,Analiza situației la Simularea Națională, an școlar 2014-2015 și analiza situației la Evaluarea Națională, an școlar 2013-2014”- prof. Chiș Maria-Marioara Școala Gimnazială ,,Mihai Eminescu” Săliștea de Sus

2. Referat cu tema : ,,Metode de imbunatatire a rezultatelor la Evaluarea Naționala’’- prof. Vlad Elena Școala Gimnazială Bogdan-Vodă

Responsabili: prof. Chiș Maria-Marioara; prof. Borodi ElenaZONA BORŞA-VIŞEU LICEULocul de desfăşuare: Liceul Teoretic ,, Bogdan Vodă „ Vișeu de SusTematică:

1. Bacalaureat (Evaluare naţională) - Pregătirea elevilor la matematică pentru examenele naționale; Calendarul consultaţiilor, evaluarea ritmică, teste de simulare; Rezultatele obţinute la clasă şi notele obţinute în simulări

2. Olimpiada de matematică şi alte concursuri – analiza rezultatelor3. Rezolvări de probleme din Gazeta Matematică- prof. Vesel-Pop Floarea, prof.

Rad Daniel, prof. Șimon Paula4. Centre de greutate- prof. Mihali Marinela, prof. Timiș Lenuța

Responsabil de cerc: prof. Vesel-Pop Floarea; prof. Mihali MarinelaZONA SIGHETU MARMAŢIEI

I. GRUPA ŞCOLI GIMNAZIALE Locul de deşfăşurare: Școala Gimnazială „George Coșbuc” Sighetu MarmațieiTematica:

1. Analiza rezultatelor la Olimpiada de matematică faza locală, județeană și națională – prof. Zetea Bogdan

2. Impactul simulării Examenului de Evaluare Națională asupra elevilor în vederea conștientizării acestora privind necesitatea accelerării pregătirii pentru examen – prof. Mic Vasile

3. Discuții asupra diverselor modele de teste pentru Evaluarea Națională care au cuprins întreaga programă a Examenului de Evaluare Națională – prof. Costel Drăgoi

4. DiverseResponsabil: prof. Bizău Ioan; prof. Mic Vasile

II. GRUPA LICEULocul de deşfăşurare: Liceul Pedagogic „Regele Ferdinand” Sighetu MarmațieiTematica:

39

Page 40: SOCIETATEA DE ŞTIINŢE MATEMATICE DIN - ISJ … matematica 2015... · Web viewpentru punctaj maxim la sectiunea liceu Zelina Mihai, clasa a X-a, C.N. „V. Lucaciu”, prof. Boroica

1. Stadiul pregătirii examenului de Bacalaureat 20152. Subiect practic: „Probleme de bacalaureat”-catedra de matematică a Lic.

ped. ,,Regele Ferdinand”3. Diverse

Responsabil: prof. Gherasin GheorgheZONA CODRULoc de desfășurare: Liceul Tehnologic „Dr. Florian Ulmeanu” UlmeniTematica:

1. Evaluarea continuă, parte integrantă a procesului de predare-învățare referat prof. Cozma Ciprian, prof .Deac Gheorghe Daniel, prof Chiriac Nicolae, Liceul Tehnologic „Dr. Florian Ulmeanu” Ulmeni

2. Propuneri referitoare la înființarea, la nivel zonal, a unui centru de pregătire suplimentară a elevilor capabili de performanțe

3. Sesiunea interjudețeană de comunicări științifice a profesorilor de matematică, Zalău

4. DiverseResponsabil: Lopată Angela

SESIUNEA INTERJUDEŢEANĂ DE COMUNICĂRI ŞTIINŢIFICE ŞI METODICE A PROFESORILOR DE MATEMATICĂ DIN JUDEŢELE

MARAMUREŞ, SATU-MARE ŞI SĂLAJZalău, 23 mai 2015

Lucrări prezentate în cadrul sesiunii:I. Categoria teoretico-științifică

Prezentări grafice ale unor noțiuni matematiceLector univ. dr. HORVAT MARC ANDREI, Universitatea Tehnică Cluj Napoca, Centrul Univ. Nord Baia Mare

Puncte fixe triple: lungul drum de la teorie la practicăProf. univ. dr. BERINDE VASILE, Universitatea Tehnică Cluj Napoca, Centrul Univ. Nord Baia Mare; Prof. BORCUT MARIN, Colegiul Economic „Pintea Viteazul” Cavnic

Asupra unei clase de ecuații în numere întregiProf. univ. dr. BERINDE VASILE, Universitatea Tehnică Cluj Napoca, Centrul Univ. Nord Baia Mare; Prof. DAN SÂNZIANA, Școala Gimnazială „Vasile Lucaciu” Șișești

Asupra unei probleme din G.M. Prof. HEUBERGER DANA și Prof.HEUBERGER CRISTIAN, Colegiul Naţional „Gheorghe Șincai” Baia Mare

Asupra unor matrice Prof. MUȘUROIA NICOLAE, Colegiul Naţional „Gheorghe Șincai” Baia Mare

Puncte fixe pe axa reală

40

Page 41: SOCIETATEA DE ŞTIINŢE MATEMATICE DIN - ISJ … matematica 2015... · Web viewpentru punctaj maxim la sectiunea liceu Zelina Mihai, clasa a X-a, C.N. „V. Lucaciu”, prof. Boroica

Prof. BOROICA GHEORGHE, Colegiul Naţional „Gheorghe Șincai” Baia Mare

Inegalități cu integraleProf. POP ADRIAN, Colegiul Naţional „Gheorghe Șincai” Baia Mare

O nouă clasă de curbe BezierLector univ. dr. PIŞCORAN LAURIAN-IOAN, Universitatea Tehnică Cluj Napoca, Centrul Univ. Nord Baia Mare; Prof. BARBU CĂTĂLIN IONEL, Colegiul Național „Vasile Alecsandri” Bacău

Identităţi şi probleme de reprezentare Prof. BOROICA GABRIELA, Colegiul Naţional „Vasile Lucaciu” Baia Mare

Inegalităţi de tip Schur Prof. FĂRCAŞ NATALIA, Colegiul Naţional „Vasile Lucaciu” Baia Mare

Reducerea la absurd în gimnaziu și liceu Prof. BOLOGA MONICA LUCIANA, C.N. „Gh. Sincai” Baia Mare; Prof. CIOCLU COSTEL, Liceul Teoretic „Emil Racoviță” Baia Mare

Asupra unei inegalitati date la ONM 2015 Prof. BANCOȘ MARIN, Universitatea Tehnica Cluj Napoca, Centrul Univ. Nord Baia Mare

Poliedre convexeProf. POP COSMIN, Școala Gimnazială „George Coșbuc” Baia Mare

Avantaje și dezavantaje ale trigonometriei în rezolvarea problemelor de geometrieProf. IENUȚAȘ VASILE, Școala Gimnazială „George Coșbuc” Baia Mare

Eficiența manualelor digitale în predarea matematiciiProf. IENUȚAȘ MONICA, Școala Gimnazială „AVRAM IANCU” Baia Mare

II. Categoria metodico-didacticăAsupra unor tipuri de probabilități

Prof. ȘERBA LUCIA, Colegiul Tehnic „Anghel Saligny” Baia Mare Imagini și grafice - auxiliare în rezolvări de probleme ingenioase

Prof. MIC LEON, C.T. „Aurel Vlaicu” Baia Mare Numere prime. Probleme reprezentative

Prof. POP MARIA MONICA, Școala Gimnazială „Lucian Blaga” Fărcașa Tehnici de instruire diferențiată

Prof. LOPATĂ ANGELA, Școala Gimnazială Gârdani Matematica și muzica - abordare transdisciplinară

Prof. STARK ANDREA, Școala Gimnazială „Nicolae Iorga” Baia MareProbleme de concurs

Prof. BRETAN ANDREI, Școala Gimnazială „Nicolae Iorga” Baia MareProbleme de colorare

Prof. ZAH ȘTEFAN-MIHAI, Școala Gimnazială „Nicolae Iorga” Baia MareProbleme de matematică întâlnite în literatura română

Prof. IENUȚAȘ ADRIANA, Școala Gimnazială „George Coșbuc” Baia Mare

41

Page 42: SOCIETATEA DE ŞTIINŢE MATEMATICE DIN - ISJ … matematica 2015... · Web viewpentru punctaj maxim la sectiunea liceu Zelina Mihai, clasa a X-a, C.N. „V. Lucaciu”, prof. Boroica

LISTA ELEVILOR DIN MARAMUREŞ CARE AU APĂRUT LA RUBRICA REZOLVITORILOR ÎN GAZETA MATEMATICĂ

Elevii care au cumulat un punctaj minim 400 începând cu nr. 4/2014 până la nr. 3/2015 după:1. Pop Călin George, cls. a VII –a, Șc. Gim. „N. Iorga „ Baia Mare1470/10/2014+630/2/2015=2310p2. Șimon Gheorghe Ionuț- cls. a XII-a, Lic. „Bogdan Vodă” Vișeu de Sus110/5/2014+(110+100)/11/2014+140/12/2014+140/2/2015=600p3. Lopată Georgiana- cls.a VI-a, Șc. Gim. „L. Blaga” Fărcașa80/9/2014+(140+120+170)/11/2014=510p4. Mariș Cătălin- cls. a VI-a, Lic. Borșa90/5/2014+80/4-7-8/2011+(140+40)/11/2014+80/12/2014+(100+60+110)/2/2015=700p 5. Ardelean Rachela Adriana- cls.a-VI-a, Șc. Gim. „L. Blaga” Fărcașa(310+240)/9/2014+120/2/2015=670p6. Covaciu Denisa- cls.a-VI-a, Șc. Gim. „L. Blaga” Fărcașa(300+200)/9/2014+120/2/2015=620p7. Dunca Denisa- cls. a-VI-a, Șc. Gim. „L. Blaga” Fărcașa(230+210)9/2014+60/2/2015=500p8. Hoban Șerban- cls.a-VI-a, Șc. Gim. „L. Blaga” Baia Mare70/5/2014+(130+160)10/2014+60/11/2014+80/12/2014+100/2/2015=600p9. Zăgreanu Luca- cls.a-VI-a, C.N. „Gh. Șincai” Baia Mare300/5/2014+200/12/2014 =500p10. Bogolonovici Andreea-cls.a-VI-a, Lic.Ped. „Regele Ferdinand” Sighetu Marmaţiei80/5/2014+110/6-7-8/2014+130/10/2014+90/2/2015=410p

LISTA PROFESORILOR AL CĂROR NUME A FOST MENȚIONAT DE CEL PUȚIN PATRU ORI ÎN NUMERELE ÎNCEPAND CU 4/2014 PÂNĂ LA 3/2015

ALE GAZETEI MATEMATICE

1. POP MONICA – Școala Gimnazială „Lucian Blaga” Fărcașa – 7 apariții2. MUȘUROIA NICOLAE – Colegiul Național „Gheorghe Șincai” Baia Mare – 5 apariții3. POP VESEL FLOARE – Liceul „Bogdan Vodă” Vișeul de Sus – 4 apariții4. KELLER OTTO –Școala Gimnazială „Lucian Blaga” Baia Mare – 6 apariții5. HEUBERGER DANA – Colegiul Național „Gheorghe Șincai” Baia Mare – 4 apariții6. MIHALI MARINELA – Liceul Borșa – 5 apariții7. SĂLĂJAN RALUCA – Școala Gimnazială „Vasile Alecsandri” Baia Mare – 4 apariții8. LOPATĂ ANGELA – Școala Gimnazială Gârdani – 4 apariții

42

Page 43: SOCIETATEA DE ŞTIINŢE MATEMATICE DIN - ISJ … matematica 2015... · Web viewpentru punctaj maxim la sectiunea liceu Zelina Mihai, clasa a X-a, C.N. „V. Lucaciu”, prof. Boroica

ELEVI CARE AU TRIMIS SOLUȚII LA PROBLEME PENTRU CONCURSUL GAZETA MATEMATICĂ ȘI VIITORI OLIMPICI

1. BOTIȘ IULIA MARIA, CIURDAȘ DARIANA ENEIDA, MEREUȚ ANDREI RAMON, MEREUȚ CRISTINA DAIANA, NICOARĂ LARISA MARIA, PILANCIU CECILIA MARIA – clasa a VI-a – Școala Gimnazială „Lucian Blaga” Fărcașa, îndrumați de dna prof. POP MONICA2. a) LAZEA DARIUS – clasa a V-a – Colegiul Național „Gheorghe Șincai” Baia Mare, îndrumați de dna prof. BOJOR MEDAb) CHINA RĂZVAN – clasa a V-a – Școala Gimnazială „Vasile Alecsandri” Baia Mare, POP ROXANA MARIA – clasa a VI-a – Școala Gimnazială „Vasile Alecsandri” Baia Mare, îndrumați de dna prof. SĂLĂJAN RALUCA ANAMARIAc) BARTHA EMERIC, DUMITRAȘ IULIA, KHAN JASMIN, POP MARIA ROXANA, ONȚ ALEXIA DAIANA – clasa a V-a - Școala Gimnazială „Lucian Blaga” Fărcașa, îndrumați de dna prof. POP MONICA3. a) BARTHA EMERIC, DUMITRAȘ IULIA, POP MARIA ROXANA, ONȚ ALEXIA DAIANA – clasa a V-a - Școala Gimnazială „Lucian Blaga” Fărcașa, îndrumați de dna prof. POP MONICAb) BOTIȘ IULIA MARIA, CIURDAȘ DARIANA ENEIDA, MEREUȚ ANDREI RAMON, MEREUȚ CRISTINA DAIANA, NICOARĂ LARISA MARIA, PILANCIU CECILIA MARIA – clasa a VI-a – Școala Gimnazială „Lucian Blaga” Fărcașa, îndrumați de dna prof. POP MONICA4. a) MERCEA IOANA – clasa a VII-a, COTAN PAUL - clasa a X-a, Colegiul Național „Gheorghe Șincai” Baia Mare, îndrumați de profesorii HEUBERGER DANA și MUSUROIA NICOLAEb) TRIF ANDREI – clasa a VI-a, BODNAR ANDREEA LAURA – clasa a VIII-a – Colegiul Național „Vasile Lucaciu” Baia Mare, îndrumați de profesorii BOROICA GABRIELA și BOB ROBERTc) BARTHA EMERIC, DUMITRAȘ IULIA, BĂBAȘ RAUL FLORIN, LUPȘE LUIZA MARIA, - clasa a IV-a , BOTIȘ IULIA MARIA, BUȘECAN ANDA, CIURDAȘ DARIANA ENEIDA, MEREUȚ ANDREI RAMON, MEREUȚ CRISTINA DAIANA, NICOARĂ LARISA MARIA, PILANCIU CECILIA MARIA – clasa a V-a , BUȘECAN IULIA – clasa a VII-a, GÂTA ANAMARIA – clasa a VIII-a Școala Gimnazială „Lucian Blaga” Fărcașa, îndrumați de profesorii POP MONICA și BOLOȘ AURICA5. a) MERCEA IOANA – clasa a VII-a - Colegiul Național „Gheorghe Șincai” Baia Mare, îndrumați de dl prof. MUSUROIA NICOLAE, BODNAR ANDREEA LAURA – clasa a VIII-a – Colegiul Național „Vasile Lucaciu” Baia Mare, îndrumați de dl. Prof. BOB ROBERT,b) BLIDAR ALEXANDRA ANAMARIA, BLIDAR MELISA MĂDĂLINA – clasa a V-a – Școala Gimnazială „Vasile Alecsandri” Baia Mare, c) BARTHA EMERIC, BUȘECAN ANDA, DUMITRAȘ IULIA BĂBAȘ RAUL FLORIN, LUPȘE LUIZA MARIA, - clasa a IV-a, BOTIȘ IULIA MARIA,

43

Page 44: SOCIETATEA DE ŞTIINŢE MATEMATICE DIN - ISJ … matematica 2015... · Web viewpentru punctaj maxim la sectiunea liceu Zelina Mihai, clasa a X-a, C.N. „V. Lucaciu”, prof. Boroica

CIURDAȘ DARIANA ENEIDA, MEREUȚ ANDREI RAMON, MEREUȚ CRISTINA DAIANA, NICOARĂ LARISA MARIA, PILANCIU CECILIA MARIA – clasa a V-a, BUȘECAN IULIA – clasa a VII-a Școala Gimnazială „Lucian Blaga” Fărcașa, îndrumați de profesorii POP MONICA și BOLOȘ AURICA

CENTRUL JUDEȚEAN PENTRU TINERII CAPABILI DE PERFORMANȚĂ

Total ore/săptămână: 311, din care 97 ore/săptămână matematică. CENTRUL DE SUPER-PERFORMANȚĂ

- BAIA MARE – 3 ore/săptămână pentru clasele V-VI – 3 ore/săptămână pentru clasele VII-VIII

- SIGHETU MARMAȚIEI – 3 ore/săptămână pentru clasele V-VIIICENTRUL DE EXCELENȚĂ DE LA C.N. „GH. ȘINCAI” BAIA MARE2 ore/săptămână pentru fiecare clasă (V-XII) – total 16 ore/săptămână. CENTRUL DE EXCELENȚĂ DE LA C.N. „V. LUCACIU” BAIA MARE 2 ore/săptămână pentru fiecare clasă (V-XII) – total 16 ore/săptămână. CENTRUL DE EXCELENȚĂ DE LA ŞC. GIM. „G.COȘBUC” BAIA MARE2 ore/săptămână pentru fiecare clasă (V-VIII) – total 8 ore/săptămână. CENTRUL DE EXCELENȚĂ DE LA ŞC. GIM. „N. IORGA” BAIA MARE2 ore/săptămână pentru fiecare clasă (V-VIII) – total 8 ore/săptămână. CENTRUL DE EXCELENȚĂ DE LA ŞC. GIM. „G. COȘBUC” SIGHETU MARMAȚIEI2 ore/săptămână pentru fiecare clasă (V-VIII) – total 8 ore/săptămână. CENTRUL DE EXCELENȚĂ DE LA C.N. „DRAGOȘ VODĂ” SIGHETU MARMAȚIEI2 ore/săptămână pentru fiecare clasă (V-XII) – total 16 ore/săptămână. CENTRUL DE EXCELENȚĂ DE LA LIC. BORȘA2 ore/săptămână pentru clasele V-VI; 2 ore/săptămână pentru clasele VII-VIII; total 4 ore/săptămână CENTRUL DE EXCELENȚĂ DE LA LIC. TEOR. „BOGDAN VODĂ” VIŞEU DE SUS2 ore/săptămână pentru clasele V-VIII; 2 ore/săptămână pentru clasele IX-XII; 4 ore/săptămână Școala Gimnazială Vișeu de Jos- total 8 ore/săptămânăCENTRUL DE EXCELENȚĂ DE LA LIC. TEOR. „PETRU RAREȘ” TÂRGU LĂPUȘ2 ore/săptămână pentru clasele IX-X; 2 ore/săptămână pentru clasele XI-XII; total 4 ore/săptămână

44

Page 45: SOCIETATEA DE ŞTIINŢE MATEMATICE DIN - ISJ … matematica 2015... · Web viewpentru punctaj maxim la sectiunea liceu Zelina Mihai, clasa a X-a, C.N. „V. Lucaciu”, prof. Boroica

SUBIECTE PROPUSE LA CONCURSURILE DE MATEMATICĂ

CONCURSUL INTERJUDEŢEAN DE MATEMATICĂ „TINERE SPERANŢE”

Ediţia a X - a, 12 - 13 decembrie 2014

Proba individuală

Clasa a- IV -aProblema 1. Aflaţi numărul de forma xyz, ştiind că:

1 + 2 + 3 + … + 100 = 5050 · x {[50 : ( 6y – 7 ) + 2 · 5 · 8 ] : ( 5 · 6 ) + 4}– 6 = 1 z este ultima cifră a produsului 2 · 3 · 4 · 5 · 9 · 7

Problema 2. De 8 Martie mama primeşte de la cei cinci copii ai săi flori reprezentând numere naturale consecutive impare. Câte flori primeşte de la fiecare copil dacă, adunând suma florilor primite cu dublul acesteia, se obţine 195 ?Problema 3. Într-o familie sunt 2 fraţi, elevi ai unei şcoli gimnaziale. La sfârşitul fiecărui an şcolar, fiecare copil primeşte un număr de cărţi egal cu numărul clasei absolvite. După sfârşitul anului şcolar 2013- 2014, cei doi fraţi vor avea împreună 36 de cărţi. Ce clasă va absolvi în anul 2014 fiecare din cei doi fraţi? ( Cei doi fraţi promovează clasa în fiecare an.)Problema 4. Se consideră numărul n = 12345...20132014, obţinut prin alăturarea tuturor numerelor naturale de la 1 la 2014.a) Câte cifre are numărul n ?b) Care este a 2014-a cifră a numărului n ( socotind de la stânga spre dreapta) ?

Clasa a V- aProblema 1. Calculaţi (b+2c ) (2 a+b ) (c−a )ştiind că a+b+c=31 şi2 a+3b+4 c=105.Problema 2. Pătratul unui număr x de două cifre, scris în baza zece este de forma: x2=a (a+b ) b , unde a , a+b , bsunt cifre, iar a≠ 0a) Arătaţi căa=b ş ic ă a este pătratul unui număr natural;b) Determinaţi toate numerele naturale x cu proprietatea din enunţ.Problema 3. Se împarte un număr natural impar pe rând la numerele 2, 4, 6,…,18 şi 20. Suma resturilor obţinute este 98. Arătaţi că există exact doua resturi egale.Problema 4. O enciclopedie cu paginile numerotate de la 1 la 500 este împărţită in capitole, astfel încât fiecare capitol are exact 20 de pagini si cel puţin 25 de ilustraţii. O pagina conţine cel mult 5 ilustraţii. Prima pagină din dreapta a enciclopediei are numărul 1.a) Câte cifre s-au folosit la numerotarea paginilor din primele 15 capitole?b) Care este produsul cifrelor folosite la numerotarea oricăror 5 foi consecutive din enciclopedie? (O foaie are 2 pagini).

45

Page 46: SOCIETATEA DE ŞTIINŢE MATEMATICE DIN - ISJ … matematica 2015... · Web viewpentru punctaj maxim la sectiunea liceu Zelina Mihai, clasa a X-a, C.N. „V. Lucaciu”, prof. Boroica

c) Sa se arate ca enciclopedia conţine cel mult 375 de pagini fără nici o ilustraţie?

CLASA a VI –a

Problema 1. Determinaţi numerele de forma , scris în baza zece, ştiind că a, b, b–a sunt numere prime şi a +b = p2, unde p este număr prim.Problema 2. Stadional Naţional Arena are 55 600 de locuri numerotate de la 1 la 55 600. La meciul Steaua - Chelsea Londra, disputant în luna martie 2013 toate locurile au fost ocupate cu suporterii celor două echipe. Spunem că doi suporteri sunt „prieteni” dacă numărul locului unuia dintre ei divide numărul locului celuilalt.a) Arătaţi că există un suporter „prieten” cu toţi ceilalţi.b) Determinaţi câţi „prieteni” are suporterul de pe locul 2014.c) Există un suporter care are exact 101 de prieteni şi care să aibă numărul locului un număr prim ? Justificaţi.Problema 3. Fie segmentul [AB] cu AB = 1 m. Punctele M1, M2, M3, … , M24∈(AB) împart segmentul [AB] în segmente cu lungimea de 4 cm, iar punctele N 1, N2, N3, … , N19 ∈(AB) împart segmentul [AB] în segmente cu lungimea de 5 cm. Câte puncte M i

(i = 1, 2, 3, … , 24) coincid cu puncte N j ( j = 1, 2, 3, … , 19) ? Câte segmente cu lungimea de 1 cm sunt ? Problema 4. În jurul punctului O se formează unghiurile AOB, BOC, COD şi DOA astfel încât unghiurile AOB şi BOC sunt complementare, iar unghiurile AOD şi COD

sunt suplementare. Dacă [OM este bisectoarea BOC, [ON este bisectoarea COD şi

[OP este bisectoarea AOD, aflaţi:

a) m ( NOP);

b) m ( AOB), ştiind că m( MOP) = 1320;

c) m ( AOB), ştiind că m( COD) este de 4 ori mai mare decât m( BOC).

Clasa a VII-aProblema 1. Numerele a1, a2, …, a2008 ϵ N sunt invers proporţionale cu numerele

1 , 12

, 13

,…, 12008 şi a1+ a3+ …+a2007 =20082.

a) Aflaţi numerele a1, a2, …,a2008 ;

b) Calculaţi suma S=12+1+1a1 a2

+ 22+2+1a2a3

+…+20072+2007+1a2007 a2008

.

Problema 2. Fie a, b, c >0 astfel încât a+b+c=1.

Să se calculeze valoarea expresiei E = a2+aa+bc

+ b2+bb+ca

+ c2+cc+ab

.

Problema 3. Pe laturile unui paralelogram ABCD se construiesc în exterior pătrate. Să se arate că centrele acelor pătrate sunt vârfurile unui pătrat .

46

Page 47: SOCIETATEA DE ŞTIINŢE MATEMATICE DIN - ISJ … matematica 2015... · Web viewpentru punctaj maxim la sectiunea liceu Zelina Mihai, clasa a X-a, C.N. „V. Lucaciu”, prof. Boroica

Problema 4. Se consideră ABC în care m(∡B) = 40˚, m(∡C) = 30˚. Pe latura (BC) se consideră punctele D și E, astfel încât m(∡DAB) = 40˚ și m(∡EAC) = 30˚. Fie G intersecția paralelei prin D la dreapta AB cu latura (AC). Dacă {J }=AE ∩ BG, arătați că [JB ]≡ [JC ].

Clasa a VIII - aProblema 1. a) Determinaţi cifrele a şi b, a≠b pentru care √2 , ( a )+3 ,(b)∈Q.b) Dacă a şi b ∈N ¿, calculaţi

√1+[( 1a+ a+2b

2ab+…+ a+99 b

99 ab )−( 12 b

+ 13b

+…+ 199 b )] ∙a

Problema 2. Determinaţi valorile întregi ale lui x pentru carex2+22 x+1

+ x2+42 x+3

+ x2+62 x+5

+…+ x2+40202 x+4019

=2010

Problema 3. Se consideră punctele necoplanare A, B, C, D. Bisectoarea unghiului BAC intersectează segmentul [BC] în punctul M, iar bisectoarea unghiului BAD intersectează segmentul [BD] în punctul N. Demonstraţi că MN || CD dacă şi numai dacă AC = AD.Problema 4. Date fiind punctele A, B şi C, determinaţi punctele P din spaţiu cu

proprietatea .

Proba pe echipe

Clasa a V- aProblema 1. Se consideră numărul natural:A=(4 ∙52 )2 : [ 32014 :32012 + ( 21007 ∙ 4503¿ :8671]

– (22 ∙ 52+2∙ 5 ) ∙ [ (52 )3 :252−23 ∙2 ]+5−7 atural :TE−Cls . a777a) Calculaţi numărul A.b) Determinaţi numărul natural x pentru care3x<A<3x+1

Problema 2. Determinaţi numerele naturale de forma abcd , ştiind că 2∙ abcd= (acd−2¿ ∙ 19

Problema 3. Fie numărul B=99 …… ..99⏟ – 99. .… ...99⏟−99.……99⏟−¿ …… 999

99 9 2015 cifre 2014 cifre 2013 cifrea) Demonstraţi că numărul B nu este pătrat perfect.b) Câte cifre de 8 are numărul B?Problema 4. Un sportiv urcă un şir de trepte după regula: urcă 4 trepte şi coboară o treaptă, urcă iar 5 trepte şi coboară 3 trepte, după care reia procesul.a) Pe a căta treaptă se află sportivul după 2014 paşi?

47

Page 48: SOCIETATEA DE ŞTIINŢE MATEMATICE DIN - ISJ … matematica 2015... · Web viewpentru punctaj maxim la sectiunea liceu Zelina Mihai, clasa a X-a, C.N. „V. Lucaciu”, prof. Boroica

b) După câţi paşi ajunge pe treapta a 2014-a? (Se consideră pas urcarea sau coborârea unei trepte.)

CLASA a VI –a

Problema 1. Fie numărul , unde este scrierea în baza 10;

a) Aflaţi numerele astfel încât .b) Determinaţi cel mai mare număr de forma lui q.c) Care este cifra sutimilor din scrierea zecimală a lui q? Justificaţi.Problema 2. Fie k un număr natural şi S(k) fiind suma cifrelor numărului k.a) Arătaţi că 2014 – S(2014) este divizibil cu 9.b) Aflaţi două numere naturale a şi b astfel încât: S(a) = S(b) = S(a + b) = 9c) Să se arate că, dacă S(b) = S(a + b), cu a,b N*, atunci 9 îl divide pe a.Problema3. Unghiurile <AOB şi <BOC sunt adiacente iar [OX şi [OY sunt bisectoarele lor. a) Dacă OX OC şi OY OA, calculaţi măsurile unghurilor considerate;b) Dacă semidreptele [OX şi [OC , respective semidreptele [OY şi [OA sunt în prelungire, calculaţi măsurile unghiurilor considerate.

Problema 4. Punctele sunt coliniare în această ordine și A0A1=1cm,

A1A2 = 3 cm, …, AkAk+1= (2∙k +1) cm, k∈N. Punctele interioare segmentelor ,

, … , se colorează în roșu, iar punctele interioare segmentelor

, , … , se colorează în albastru, p∈N.

a) Stabiliți dacă există un segment de forma de lungime 17 cm.

b) Stabiliți culoarea mijlocului segmentului și lungimea segmentului

.

Clasa a VII-aProblema 1. a) Să se determine x ştiind că:

b) Să se determine n ştiind că sunt îndeplinite simultan condiţiile:

i)

ii)

48

Page 49: SOCIETATEA DE ŞTIINŢE MATEMATICE DIN - ISJ … matematica 2015... · Web viewpentru punctaj maxim la sectiunea liceu Zelina Mihai, clasa a X-a, C.N. „V. Lucaciu”, prof. Boroica

iii)

Problema 2. Ştiind că

şi

a) Calculaţi: b) Demonstraţi că şi Problema 3. ABCD trapez dreptunghic în care m (∡ A )=m (∢D )=900 ,

DC ∥AB în care şi . Ştiind că [CM bisectoarea(∡ ACD )

arătaţi că:a)

b)Problema 4. În paralelogramul ABCD se consideră un punct M pe CD, diferit de C şi D. Dreptele AM şi BC se intersectează în punctul N. Să se arate că triunghiurile DMN şi BCM au aceeaşi arie.

Clasa a VIII-aProblema 1. Fie a=√7−4√3−√7+4√3a) Calculaţi a2;b) Calculaţi b=(1+a+2√3 )2014;

c) Arătaţi că (√7−4 √3 )x+(√7+4√3)x ≥ 2, (∀ ) x∈Z .Problema 2. a) Demonstraţi că ∀ xϵ Q ,x ≥1 , x∉N , are loc inegalitatea 1[ x ]

+ 1{x }

> 4x ,

unde [x] este partea întreagă a lui x şi {x} este partea fracţionară a lui x. b) Demonstraţi că dacă a ,b , c∈ (2 ;+∞ ), atunci ab + bc + ca > 4 ( a + b + c) – 12.Problema 3. Se numeşte mediană a unui tetraedru un segment care uneşte un vârf al tetraedrului cu centrul de greutate al feţei opuse. Demonstraţi că cele patru mediane ale unui tetraedru sunt concurente. Punctul lor comun se numeşte centru de greutate al tetraedrului.

Problema 4. Fie ABCD un tetraedru oarecare şi punctele coplanare L (AB), M∈ (BC ) , N∈ (CD) şi P (AD). Să se arate că :

ALAB

∙ BMBC

∙ CNCD

∙ PDAD

≤ 116

49

Page 50: SOCIETATEA DE ŞTIINŢE MATEMATICE DIN - ISJ … matematica 2015... · Web viewpentru punctaj maxim la sectiunea liceu Zelina Mihai, clasa a X-a, C.N. „V. Lucaciu”, prof. Boroica

TABĂRA JUDEȚEANĂ DE EXCELENȚĂ LA MATEMATICĂ1-5 septembrie 2014, Vaser -Valea Babii

Clasa a VI-a

1. Să se determine numerele naturale x, y, z ştiind că

x3= y

4; y

6= z

5 şi 2 x+3 y+4 z=188 .2. Să se determine numerele naturale de două cifre care au proprietatea că raportul

dintre număr şi răsturnatul său este

74 .

3. Pe segmentul [OA] se consideră punctele A1 , A2 ,. .. , A2009 astfel încât OA1=

12

OA ,

OA 2=16

OA, OA 3=

112

OA ,…,

OA 2009=1

2009⋅2010OA

.

a) Să se demonstreze că A1 A2=4 A2 A3 .

b) Să se calculeze OA, dacă OA1+OA2+.. .+OA 2009=2009 cm

Clasa a VII-a1. Rezolvaţi, în mulţimea numerelor reale, ecuaţia:

√ x2+x+1+√2 x3+x2+2 x=√ 2x2+x+22

(Gazeta Matematică, nr. 6-7-8/2012)

2. Demonstraţi că

31⋅2

+ 42⋅3

+ 53⋅4

+ .. .+ n+2n⋅(n+1)

−( 12+ 1

3+. ..+ 1

n )<2.

(Gazeta Matematică, nr. 10/2008)3. Pe latura BC a triunghiului ABC se consideră punctele D şi E astfel încât BD=DE=EC . Mediana B B' (B'∈ AC ) , intersectează pe AD în M, iar mediana C C ' (C '∈ AB ) , intersectează pe AE în N. Arătaţi că:a) BMNC este trapez.

b) MN=1

4BC

. (Gazeta Matematică, nr. 6-7-8/2012)

Clasa a VIII-a

50

Page 51: SOCIETATEA DE ŞTIINŢE MATEMATICE DIN - ISJ … matematica 2015... · Web viewpentru punctaj maxim la sectiunea liceu Zelina Mihai, clasa a X-a, C.N. „V. Lucaciu”, prof. Boroica

1. Rezolvaţi, în mulţimea numerelor întregi, ecuaţia

√ x2−20+√45−x2=|2 x+5| .

2. Fie numerele reale x1 , x2 ,. .. , xn , unde n∈N , n impar. Dacă |x1+x2−2x3|=|x2+x3−2 x4|=. ..=|xn+x1−2 x2|, demonstraţi că x1=x2=. ..=xn .

3. Fie G1 şi G2 centrele de greutate ale triunghiurilor ACD, respectiv BCD, triunghiurile fiind situate în plane diferite. Considerăm N mijlocul segmentului [CD],

M∈ ( AB )astfel încât

AMAB

=25 şi MN∩AG 2={ E } . Arătaţi că:

a) G1 G2‖AB .

b) EG1‖( BCD ) .

CLASA a IX–a

1. Se consideră expresia E ( x )=5x2−10 x+7

2 x2−4 x+3, x∈ R

.

a) Rezolvaţi ecuaţia E ( x )=2

b) Calculaţi E (x1)+E ( x2) , unde x1 , x2 sunt soluţiile ecuaţiei x2−x−1=0 .

c) Determinaţi cea mai mică valoare a expresiei E ( x ) .2. Fie a ,b∈R+

¿

cu a3+b3=2014 ab . Să se arate că a+b≤2014 .

3. Se consideră triunghiul ABC cu centrul de greutate G. Fie A' , B' , C'

simetricele punctelor A, B, C faţă de mijloacele segmentelor [BG], [CG], respectiv [AG]. Să se arate că triughiurile ABC şi A

' B' C'au acelaşi centru de greutate

Clasa a XI-a

1. Se consideră matricea

A=(a b 00 a 0c d a )∈M 3(C )

. Să se calculeze An , n∈ N ¿

.

2. Se consideră şirul (an)n≥1 dat prin relaţia de recurenţă:3 n (an+1−an)−4 (an+5 )=5 n (9 n+15 )−an+1 ,∀n∈N ¿ , a1=4 .

Să se calculeze limn→∞

an+3

1+8+15+. ..+(7 n−6 ) .51

Page 52: SOCIETATEA DE ŞTIINŢE MATEMATICE DIN - ISJ … matematica 2015... · Web viewpentru punctaj maxim la sectiunea liceu Zelina Mihai, clasa a X-a, C.N. „V. Lucaciu”, prof. Boroica

3. Se consideră matricea A=(1 2

4 8 )şi X (a )=I 2+aA , a∈R .

a) Să se arate că X (a )⋅X (b )=X (a+b+9ab ) , ∀ a ,b∈ R .

b) Să se calculeze (X (a ) )n , n∈N ¿.

Clasa a XII-a

1. a) Daţi un exemplu de funcţie f : (−π , π )→R bijectivă şi care are primitive.

b) Fie a ,b∈R, a<b şi f : [a,b ]→R o funcţie bijectivă. Demonstraţi că f nu are primitive.

2. a) Calculaţi ∫ x⋅e−2 x dx , x∈R şi ∫ xarccos x

√1−x2dx ,x∈ (−1,1 )

b) Fie a>0 , b<0 astfel încât a2+b≤0 şi f :R→R o funcţie astfel încât

( f ∘ f ) (x )=a⋅f ( x )+b⋅x ,∀ x∈R . Demonstraţi că f nu are primitive.

3. a) Fie n∈N ¿. Câte legi de compoziţie comutative şi cu element neutru se pot

defini pe o mulţime cu n elemente?b) Pe M=Z×Z se consideră operaţia „ * „ definită astfel: (x1 , y1)∗(x2 , y2)=(x1 x2 , y1+ y2 ) . Ştiind că ( M ,* ) este monoid, să se afle elementele simetrizabile ale acestui monoid.

TABĂRA JUDEȚEANĂ DE MATEMATICĂ 31 ianuarie 2015 – 4 februarie 2015

Secțiunea gimnaziuClasa a IV-a

Pentru problemele 1-3 pe foaia de concurs se scrie litera corespunzătoare rezultatului corect, iar pentru problemele 4–7 se trec doar rezultatele.1. Jumătate din jumătatea sfertului unui număr este 4. Aflaţi numărul.a) 16 b) 32 c) 24 d) 642. Un cangur sare de 20 de ori pe minut. Fiecare săritura are 3 metri. Care este distanţa parcursă de cangur într-un sfert de oră?a) 900 m b) 51 m c) 3060 m d) 1260 m

3. Dacă , iar , atunci : este:

a) b) c) d) 52

Page 53: SOCIETATEA DE ŞTIINŢE MATEMATICE DIN - ISJ … matematica 2015... · Web viewpentru punctaj maxim la sectiunea liceu Zelina Mihai, clasa a X-a, C.N. „V. Lucaciu”, prof. Boroica

4. Cel mai mic număr natural care are suma cifrelor este: .......................................5. Numerele cuprinse între şi care dau restul la împărţirea cu sunt: ...............6. Suma a numere pare consecutive este . Cel mai mare număr este: ......................7. Un număr de patru cifre are suma cifrelor 36. Suma cifrelor succesorului este egală cu .........................Pentru problemele 8-9 redactaţi, pe foaia de concurs, rezolvările complete.8. Patru prieteni au împreună 86 de lei. Dacă primul dintre ei ar primi de la fiecare dintre ceilalţi câte 5 lei, atunci sumele lor ar fi patru numere naturale consecutive. Aflaţi câţi lei are fiecare dintre cei patru prieteni. ( Gazeta Matematică, Nr.1/2011)9. O carte ciudată are toate paginile numerotate numai cu cifre impare.a) Calculaţi suma numerelor de pe primele pagini.b) Ce număr are a cincizecea pagină?c) Câte pagini are cartea dacă pe ultima pagină se află numărul ?

Problemele au fost selectate şi propuse de: prof. Ienuţaş Monica – Şc. Gim. „A. Iancu”, Baia Mare prof. Muşuroia Nicolae – C.N. „Gh. Şincai”, Baia Mare

Clasa a V-aPentru problemele 1-7 pe foaia de concurs, se scrie litera corespunzătoare rezultatului corect:1. Rezultatul calculului 1+2+3+4+. ..+2015 este:a) 2031120 b) 4062240 c) 2029105 d) 40312. Mama împarte copiilor săi, în mod egal, 24 de bomboane. Ştiind că are 5 fii şi fiecare dintre aceştia are o soră, atunci fiecare copil primeşte câte:a) 3 bomboane b) 2 bomboane c) 4 bomboaned) 6 bomboane

3. Rezultatul calculului {[ (32)3 ]5 :2710+12533}: [ (511)9+1620 :280 ]

este:a) 2 b) 0 c) 8 d) 14. Dacă a+b=17 , atunci 3a+3b+200 este egal cu:a) 251 b) 302 c) 51 d) 215

5. Se consideră şirul: 1 ;1;2 ;1;2 ;3 ;1;2 ;3 ;4 ;1 ;2 ;3; 4 ;5 ; .. .Al 50-lea termen al şirului este:a) 2 b) 3 c) 5 d) 7

6. Restul împărţirii lui x=1⋅2⋅3⋅. ..⋅41+1 la numărul y=36⋅2015 este egal cu:

53

Page 54: SOCIETATEA DE ŞTIINŢE MATEMATICE DIN - ISJ … matematica 2015... · Web viewpentru punctaj maxim la sectiunea liceu Zelina Mihai, clasa a X-a, C.N. „V. Lucaciu”, prof. Boroica

a) 2 b) 0 c) 3 d) 17. Numărul de moduri în care putem aşeza 5 cărţi diferite pe raftul unei biblioteci este:a) 125 b) 120 c) 5 d) 10Pentru problemele 8-9 redactați, pe foaia de concurs, rezolvările complete.8. Un număr natural A îl numim „super3” dacă suma cifrelor sale este de 3 ori mai mare decât suma cifrelor numărului A+1. Aflaţi toate numerele „super3” cu cel mult 4 cifre. (Gazeta Matematică, nr. 5/2013)9. În tabăra de matematică, elevii participanţi de clasa a V-a sunt repartizaţi într-un amfiteatru. Dacă se aşează câte 2 elevi într-o bancă, rămân 5 bănci libere şi o bancă ocupată doar cu un elev. Dacă mai vin 39 de elevi şi se aşează câte 3 elevi intr-o bancă, rămâne 1 elev în picioare. Câţi elevi au fost iniţial şi câte bănci sunt în amfiteatru?

Problemele au fost selectate şi propuse de: prof. Neaga Nadina–Şc.Gim. „Dr.V. Babeș” Baia Mare prof. Nagy Anamaria-Şc.Gim. „L. Blaga” Baia Mare prof. Caltea Amalia-Șc.Gim. „Al.I.Cuza” Baia Mare

Clasa a VI-aPentru problemele 1-6 pe foaia de concurs, se scrie litera corespunzătoare rezultatului corect

1. Rezultatele calculului ( 0 , (14 )

0 , (21 )+ 0 ,14

0 , 21+141414

212121 )4030

: (22015+22015) este:

a) 1 b) 22014

c) 22015

d) 2

2. Dacă a, b, c reprezintă măsurile unghiurilor unui triunghi 3 a+2 b+c=280∘,

atunci2 a+b este :

a)180∘b)80∘

c) 90∘d) 100∘

3. Cel mai mic număr natural n pentru care

1⋅2⋅3⋅. . .⋅n66

∈ N este

a) 6 b) 12 c) 15 d) 66

4. Numărul divizorilor lui A=1+2+3+.. .+2016 este a) 72 b) 60 c) 36 d) 105. Fie punctele coliniare A, B, C, D în această ordine. Dacă M, N şi P sunt mijloacele segmentelor [AB], [BC], respectiv [CD] şi MN= 9 cm, NP= 7 cm, iar AB+CD= 16 cm, atunci suma lungimilor segmentelor [AB], [BC] şi [CD] este a) 32 cm b) 23 cm c) 24 cm d) 12 cm

54

Page 55: SOCIETATEA DE ŞTIINŢE MATEMATICE DIN - ISJ … matematica 2015... · Web viewpentru punctaj maxim la sectiunea liceu Zelina Mihai, clasa a X-a, C.N. „V. Lucaciu”, prof. Boroica

6. Fie ∠ AOB , ∠BOC , ∠COD şi ∠DOA unghiuri în jurul unui punct. Se ştie că m (∠BOC )=130∘

, unghiurile ∠ AOB şi ∠COD sunt complementare, iar unghiurile ∠ AOB şi ∠DOA sunt suplementare.A) Măsura unghiului ∠ AOB este:

a)40∘b) 30∘

c) 60∘d) 90∘

B) Măsura unghiului dintre bisectoarea [OE a unghiului ∠COD şi semidreapta [OF perpendiculară pe dreapta OA, unde E şi B sunt în semiplane opuse faţă de dreapta OA, este:

a) 60∘b) 65∘

c) 70∘d) 75∘

Pentru problemele 7-8 redactați, pe foaia de concurs, rezolvările complete.7. Fie triunghiul ABC cu AB= 8 cm, M este mijlocul segmentului [BC] şi [BE este

bisectoarea unghiului ∠ ABC , unde E∈ AC şi BE⊥AM . Determinaţi lungimea segmentului [BC].8. A) Rezolvaţi în N ecuaţia:x+1

2+ x+5

3+ x+11

4+ x+19

5+ x+29

6+ x+41

7+ x+55

8+ x+71

9+ x+89

10=45

B) Determinaţi numerele naturale abc cu proprietatea că aa⋅(b2+c2)=2013Problemele au fost selectate şi propuse de:

prof. Horvat-Marc Andrei- U. T. Cluj-Napoca, Centrul Univ. Nord Baia Mare prof. Ionescu Sorin-Șc.Gim. „I.L. Caragiale” Baia Mare

Clasa a VII-aPentru problemele 1-6 pe foaia de concurs, se scrie litera corespunzătoare rezultatului corect:

1. Rezultatul calculului ( 7

4−√2)

2

+2⋅(74+√2)( 7

4−√2)+(√2+ 7

4 )2

este :a) 8 b) 12,1 c) 12,25 d) 42. Fie x = 20,15. Dintre următoarele, cel mai mare este numărul :

a)

x+32 b)

6 xx+3 c) √3 x d) √ x2+9

2

3. Ecuaţia

x−2√34+√3

= 4−√3x+2√3 are suma soluţiilor egală cu:

a)

265 b) 0 c)

245 d) 5

55

Page 56: SOCIETATEA DE ŞTIINŢE MATEMATICE DIN - ISJ … matematica 2015... · Web viewpentru punctaj maxim la sectiunea liceu Zelina Mihai, clasa a X-a, C.N. „V. Lucaciu”, prof. Boroica

4. Dacă partea întreagă a numărului real x se notează cu [x], atunci [√2⋅4 ]+ [√4⋅6 ]+. ..+ [√20⋅22 ]=:a) 55 b) 110 c) 120 d) 121

5. În Δ ABC , m (∠ A )=90 ° , BC = 18 cm, (BD este bisectoarea ∠ ABC , D∈ ( AC ) . Paralela prin D la AB intersectează BC în E. Dacă m (∠BED )=120 ° , EC=…………a) 6 cm b) 9 cm c) 13,5 cm d) 12 cm6. În trapezul ABCD, cu bazele [AB] şi [CD], E şi F sunt mijloacele laturilor [AD] şi [BC], iar G şi H sunt mijloacele segmentelor [ED] , respectiv [FC]. A) Dacă CD = 6 cm şi GH = 9,5 cm, AB=……….a) 13 cm b) 20 cm c) 19 cm d) 16,5 cm

B) Dacă AD∩BC= {M } , raportul dintre AABCD şi AΔ ABM se scrie ca procent:a) 70% b) 30% c) 9% d) 91%Pentru problemele 7-8 redactați, pe foaia de concurs, rezolvările complete.

7. a) Arătaţi că 3+5+9+11+. ..+2 n−1=(n−1 ) (n+1 ) .b) Se consideră numerele raţionale

An=23+ 2

3+5+ 2

3+5+7+ 2

3+5+7+9+. ..+ 2

3+5+7+.. .+(2 n−1 ) , cu n∈N , n≥3Calculaţi primele 4 zecimale ale lui 5⋅A99 .8. Fie ABCD un dreptunghi cu centrul O şi M mijlocul laturii [AB], AB>BC , iar E∈(DA astfel ca A∈ ( DE ) , DA=2 EA . Fie N şi P intersecţiile dreptei CM cu BD, respectiv BE. Se dă AC = 12 cm.a) Calculaţi AP.b) Arătaţi că E, O şi mijlocul Q al segmentului [AN] sunt puncte coliniare.c) Ştiind, în plus, că CP⊥BD calculaţi AX, unde X∈BD astfel încât AY+YP>AX+XP , pentru orice punct Y ∈BD .

Subiectele au fost propuse şi selectate de : prof. Bunu Iulian-Liceul de Arte Baia Mare prof. Pop Sever-Șc.Gim. „V. Alecsandri” Baia Mare

Clasa a VIII-aPentru problemele 1-5 pe foaia de concurs se trec doar rezultatele.

1. Inversul numărului este....56

Page 57: SOCIETATEA DE ŞTIINŢE MATEMATICE DIN - ISJ … matematica 2015... · Web viewpentru punctaj maxim la sectiunea liceu Zelina Mihai, clasa a X-a, C.N. „V. Lucaciu”, prof. Boroica

2. O echipă de 10 muncitori termina o lucrare in 6 zile. Dacă numărul muncitorilor din echipă se dublează atunci aceeaşi lucrare poate fi terminată în ... zile3. Un cerc cu lungimea de cm are aria egală cu...4. Cel mai mic număr natural care împărţit pe rând la 7 şi 11 dă de fiecare dată restul 3 şi câtul diferit de zero este...

5. Partea întreagă a numărului este ....Pentru problemele 6-7, pe foaia de concurs, scrieţi varianta corectă.

6. Soluţia ecuaţiei este

a. b. c. d.

7. Pe fiecare din cele 6 muchii ale unui tetraedru se consideră câte trei puncte diferite de vârfuri. Câte triunghiuri se pot forma având vârfurile printre punctele considerate, diferite de vârfurile tetraedrului? a. 90 b. 270 c. 540 d. 810Pentru problemele 8-9 redactați, pe foaia de concurs, rezolvările complete.

8. Fie expresia: .

a) Să se arate că: b) Să se rezolve în muţimea numerelor întregi inecuaţia:

c) Numerele reale pozitive au suma egală cu 4. Să se demonstreze că:

9. a) Desenaţi un cub .b) Să se arate că există tetraedre astfel încât: m(<VBA) = m(<ABC) = m(<ACV) = 900 . c) Fie un tetraedru astfel încât m(<VBA) = m(<ABC) = m(<ACV) = 900 . Aflaţi aria totală a tetraedrului în funcţie de .d) Care este numărul maxim de unghiuri drepte dintre muchiile unui tetraedru ?Justificaţi răspunsul.

Subiectele au fost propuse şi selectate de : prof. Ienuţaş Vasile, - Şc. Gim. „G. Coşbuc”, Baia Mare prof. Zah Ştefan - Şc. Gim. „G. Coşbuc”, Baia Mare

Secţiunea liceuCLASA a IX – a

57

Page 58: SOCIETATEA DE ŞTIINŢE MATEMATICE DIN - ISJ … matematica 2015... · Web viewpentru punctaj maxim la sectiunea liceu Zelina Mihai, clasa a X-a, C.N. „V. Lucaciu”, prof. Boroica

1. a) Să se determine partea întreagă a numărului , unde . b) Să se arate că dacă şi , atunci are loc inegalitatea

.

2. Pentru fiecare n∈N ¿considerăm numărul .

a) Să se arate că este divizibil cu 13, ∀ n∈N ¿.

b) Să se determine valorile naturale ale lui n pentru care este divizibil cu 13.

3. Pe laturile şi ale patrulaterului convex ABCD se consideră respectiv

punctele M şi N astfel încât şi Fie .Se ştie că şi .

a) Exprimaţi vectorul în funcţie de şi .b) Arătaţi că ABCD este paralelogram.

Subiectele au fost propuse şi selectate de: prof. Boroica Gabriela – C.N. „V Lucaciu”, Baia Mare

Clasa a X-a1. Fie f ∶ Z →Zastfel încât

∀nϵ Z , .

Dacă , să se demonstreze că ∀n∈Z , .2. Fie a ,b∈R , a<b.

a) Dacă x∈C, astfel încât , să se arate că x∈ R și .

b) Să se rezolve ecuația . Leonard Giugiuc

3. Fie triunghiul echilateral ABC și punctul D, diferit de A, B, C, pe cercul circumscris

triunghiului. Fie .

a) Dacă , să se arate că .

b) Dacăa ,d∈C ,d ≠ 0, să se arate că funcția f : [0,1]→ R, este convexă.

58

Page 59: SOCIETATEA DE ŞTIINŢE MATEMATICE DIN - ISJ … matematica 2015... · Web viewpentru punctaj maxim la sectiunea liceu Zelina Mihai, clasa a X-a, C.N. „V. Lucaciu”, prof. Boroica

c) Să se arate că , unde R este raza cercului circumscris triunghiului ABC. Subiectele au fost propuse şi selectate de:

prof. Heuberger Dana – C.N. „Gh. Şincai”, Baia Mare prof. Heuberger Cristian – C.N. „Gh. Şincai”, Baia Mare

Clasa a XI-a

1. Pentru o matrice A=(a bc d)∈M 2 ( R ) , , notăm f A ( x )=det ( A−x ∙ I 2 ) , x∈R şi

a) Să se arate că .

b) Dacă şi , atunci arătaţi că .

c) Daţi un exemplu de matrice A∈M 2 (R ) , astfel încât .

2. a) Să se arate că şirul este convergent.

b) Dacăf ∶ N ¿→ N ¿

este o funcţie injectivă, să se calculeze . ( Argument 2011)

3. FieA∈M n ( R ) , şi o rădăcină de ordinul n a unităţii,n∈N , n ≥ 2. Să se arate că

este un număr real sau un număr complex pur imaginar.Subiectele au fost propuse şi selectate de:

prof. Boroica Gheorghe – C.N. „Gh. Şincai”, Baia Mare

Clasa a XII-a M1

1. Fie

M={X∈M 3´( R ) X3−X2−I 3=O3 }

și .

a) Să se demonstreze că și că orice matrice din e inversabilă.b) DacăB∈M 3(R) și iar , să se demonstreze că:

. 59

Page 60: SOCIETATEA DE ŞTIINŢE MATEMATICE DIN - ISJ … matematica 2015... · Web viewpentru punctaj maxim la sectiunea liceu Zelina Mihai, clasa a X-a, C.N. „V. Lucaciu”, prof. Boroica

2. Fie și . Admitem că este grup.

a) Să se calculeze , unde .

b) Să se determine elementele de ordin finit ale grupului .

3. Fie funcțiaF : R → R , F ( x )={x2sin 1x2 , x ≠ 0

0 , x=0.

a) Să se demonstreze că funcția este o primitivă a unei funcții ce se cere determinată.b) Să se dea un exemplu de funcţieg :[0,1]→ R care admite primitive, dar care nu este integrabilă. Subiectele au fost propuse şi selectate de

prof. Bojor Meda – C. N. „Gh. Şincai”, Baia Mare prof. Bojor Florin – C. N. „Gh. Şincai”, Baia Mare

Clasa a XII-a M2SUBIECTUL I

1. Să se arate că numărul (1−√3 )3+(1+√3 )3 este număr natural.

2. Să se calculeze aria triunghiului format de graficul funcţiei f : R→R ,

f ( x )=−125

x+12 cu axele de coordonate.

3. Să se rezolve ecuaţia .4. Prețul unui obiect se majorează cu 15%. După o perioadă de timp obiectul se ieftineşte cu 10% , produsul ajungând astfel să coste 3105 lei. Să se calculeze preţul iniţial al produsului.

5. Să se determine parametrul real α astfel încât vectorii u=(α+12)⋅i+5 j şi

v=5 i+(α−12) j să fie coliniari.

6. În triunghiul ABC se dau AB=7 , cosC=24

25 . Să se calculeze lungimea razei cercului circumscris triunghiului ABC .SUBIECTUL II

60

Page 61: SOCIETATEA DE ŞTIINŢE MATEMATICE DIN - ISJ … matematica 2015... · Web viewpentru punctaj maxim la sectiunea liceu Zelina Mihai, clasa a X-a, C.N. „V. Lucaciu”, prof. Boroica

1. Se consideră ecuaţia x2−3 x−2=0 cu soluţiile x1 , x2∈ R . Fie matricele

A=(a b cb c ac a b )∈M 3(R )

şi

V=(1 x12 x2

2

1 x1 x2

1 1 1 )∈M3 (R )

.

a) Să se arate că det (V )=4 ( x2−x1 ).

b) Să se arate că dacă det ( A )=0 atunci a+b+c=0 sau a=b=c .2. Pe R se defineşte legea de compoziţie asociativă x∗y=−2 (x−1 ) ( y−1 )+1,∀ x , y∈R .a) Să se determine numerele reale care sunt egale cu simetricele lor faţă de legea de compoziţie * .b) Să se rezolve ecuaţia x∗x∗x=x , x∈ R .SUBIECTUL III 1. Se consideră funcţiaf : R→R , f ( x )=arctgx-arctg ( x- 2) .

a) Să se determine intervalele de monotonie şi punctele de extrem ale funcţiei f .

b) Să se demonstreze că funcţia g : R→R ,g (x )= f ( x )+arctg ( x−1 )2

2 este constantă.

2. Se consideră funcţiilef ,F : R→R , date prin f ( x )=2 e2 x (x2+3 x+4 ) şiF (x )=e2 x (ax 2+bx+c ) , unde a ,b , c∈R .

a) Să se determine numerele reale a , b , c astfel încât funcţia F să fie o primitivă a

funcţiei f .

b) Pentru a=1 , b=2 , c=3 să se calculeze ∫0

1 f '( x )⋅F ( x )− (f ( x ))2

( F ( x ))2dx

.Subiectele au fost propuse şi selectate de:

prof. Pop Adrian – C. N. „Gh. Şincai”, Baia Mare

OLIMPIADA NAȚIONALĂ DE MATEMATICĂ – etapa locală28 februarie 2015

Clasa a IV - a1.a) Să se calculeze: 10−2+9 ∙ 4 :2b) În exercițiul 9 ∙ 4 :2+10−2 , folosiți paranteze pentru a obține, pe rând, rezultatele 1, apoi 90.2. a) Să se calculeze suma numerelor naturale de două cifre din care cel puțin una este

egală cu 2.

61

Page 62: SOCIETATEA DE ŞTIINŢE MATEMATICE DIN - ISJ … matematica 2015... · Web viewpentru punctaj maxim la sectiunea liceu Zelina Mihai, clasa a X-a, C.N. „V. Lucaciu”, prof. Boroica

b) Pentru a confecționa 12 uniforme pentru fete și 20 de uniforme pentru băieți s-au folosit 84 m de stofă. Pentru 6 uniforme pentru fete se folosește tot atât material cât pentru 4 uniforme pentru băieți. Câți m de stofă s-au folosit pentru fiecare tip de uniformă?3. Pe o tablă sunt scrise numerele de la 1 la 20. Celor 19 elevi din clasă li se propune următorul joc: primul elev șterge două numere și pune în locul lor suma acestora mărită cu 1; al doilea elev șterge două numere și pune în locul lor suma acestora mărită cu 2; al treilea elev șterge două numere și pune în locul lor suma acestora mărită cu 3 și tot așa mai departe până la ultimul elev. Ce număr rămâne pe tablă la terminarea jocului? (Fiecare elev joacă o singură dată)

Subiectele au fost selectate şi propuse de: prof. Bretan Andrei – Şc. Gim. „N. Iorga”, Baia Mare Fornvald Natalia - Inspector şcolar pentru învățământ primar

Clasa a V – aVarianta 1

1.a) Fie n=1+3+5+…+2015. Arătaţi că n este pătrat perfect.b) Se consideră suma S=2+2∙ 6+2 ∙62+2∙63+…+2∙ 699. Arătaţi că 2015 ∙ S este divizibil cu 10.

(prof. Marinela Mihali)2.Determinaţi numerele naturale abcd ştiind că 2 ∙ abcd=(acd−2 ) ∙ 19.

(Gazeta Matematică nr. 3/2014)3. Diferenţa a două numere naturale este 3. Aflaţi numerele ştiind că unul dintre ele este cu 11 mai mic decât triplul celuilalt număr.

Subiectele au fost selectate şi propuse de: prof. Popovič Ioana –Şc.Gim. „O. Goga” Baia Mare prof. Szerasz Maria - Şc.Gim. „D. Cantemir” Baia Mare prof. Sabău Ştefan - C.N. „V. Lucaciu” Baia Mare prof. Zah Ştefan - Şc.Gim. „G. Coşbuc” Baia Mare

Clasa a V-aVarianta II

1.Se dau numerele x=[2302

∙ (26 )100∙2+(324 )100

:2500 ]2+23004 şi

y=5 ∙ (32002−32001−91000).Se cere:a) Să se arate că x și y sunt pătrate perfecte.b) Să se arate că x este divizibil cu 10.c) Să se compare numerele x și y.2 . Aflați numărul natural a a pentru care 2 ∙ aa2+2∙ a3+2 ∙ a2−a=2 ∙2014

62

Page 63: SOCIETATEA DE ŞTIINŢE MATEMATICE DIN - ISJ … matematica 2015... · Web viewpentru punctaj maxim la sectiunea liceu Zelina Mihai, clasa a X-a, C.N. „V. Lucaciu”, prof. Boroica

(Gazeta Matematică nr. 10/2014)3. Un elev trebuie să rezolve 24 de probleme în patru zile. În fiecare zi el rezolvă mai multe probleme decât în ziua precedentă. În ziua a patra rezolvă de cinci ori mai multe probleme decât în prima zi. Care este numărul maxim de probleme pe care le poate rezolva în a treia zi?

Subiectele au fost selectate şi propuse de: prof. Mic Vasile–Şc.Gim. „G. Coşbuc” Sighetu Marmației prof.Vișovan Magdalena–Lic.Ped. „Regele Ferdinand” Sighetu Marmației prof. Zetea Teodora–C.N. „D. Vodă” Sighetu Marmației

Clasa a VI-a

1.Se consideră numerele şi .a) Să se arate că este număr natural.

b) Să se arate că .( Supliment Gazeta Matematică nr.12/2014)

2. Spunem că un număr natural are proprietatea (P) dacă este prim, cel puţin egal cu cinci şi se poate scrie ca suma de două pătrate perfecte .a) Daţi patru exemple de numere naturale care au proprietatea (P) .

b) Dacă numerele au proprietatea (P), arătaţi că numărul

, unde este număr natural nenul, nu poate fi pătrat perfect . 3. Fie unghiurile adiacente suplementare ∢ AOB şi ∢BOC astfel încât raportul

măsurilor lor să fie egal cu . Fie semidreapta opusă bisectoarei unghiului ∢BOC . În interiorul unghiului ∢COD se consideră punctele M şi N astfel încât

m (∢CON )=m (∢DOM )=2 m (∢MON )>45°.a) Aflaţi măsura unghiului ∢COD.b) Demonstraţi că punctele B,O,M sunt coliniare .

Subiectele au fost selectate şi propuse de: prof. Zlampareţ Horia–C.N. „V.Lucaciu” Baia Mare prof. Nagy Anamari –Şc.Gim. „L. Blaga” Baia Mare prof. Caltea Amalia–Şc.Gim. „Al .I.Cuza” Baia Mare

63

Page 64: SOCIETATEA DE ŞTIINŢE MATEMATICE DIN - ISJ … matematica 2015... · Web viewpentru punctaj maxim la sectiunea liceu Zelina Mihai, clasa a X-a, C.N. „V. Lucaciu”, prof. Boroica

prof. Pop Sever–Şc.Gim. „V.Alecsandri” Baia Mare

Clasa a VII - aVarianta I

1.a) Să se calculeze [S ] , unde S= 2

5+ 1

5+ 1

10+ 1

20+.. .+ 1

5⋅2n−2, n∈N ¿

,unde [ x ] reprezintă partea întreagă a numărului x .

(Supliment Gazeta Matematică nr. 9/2014)

b) Fie mulţimea {x1 ; x2 ; x3 ; x4 ; . .. ; x2015}={1;2 ;3 ;4 ; . . .;2015 }. Arătaţi că printre

numerele |x1−1|;|x2−2|; |x3−3|; .. . ;|x2015−2015| există cel puţin două numere egale.

2 . Să se rezolve în mulţimea numerelor întregi ecuaţia x2 y−2 x=3 y−1 .

3. Se consideră triunghiul ABC , m (∠ A )=90 ° , D simetricul punctului C faţă de A, CE⊥BD ,E∈BD şi AB∩CE= {F } . Ştiind că AE // BC , iar paralela prin F la

BD intersectează [BC ] în P şi CD în Q să se arate că:

a) m (∠C )=2⋅m (∠B ) ;

b)

23⋅BC<PE+EQ<BC .

(Gazeta Matematică nr. 12/2014)

4. Pe laturile [ AB ] şi [ AC ] ale triunghiului oarecare ABC se construiesc în exteriorul triunghiului ABC triunghiurile echilaterale ABD şi ACE . Dacă M, N şi P

sunt mijloacele segmentelor ( AD ) , ( BC ) , respectiv ( AE ) , arătaţi că triunghiul MNP este echilateral.

Subiectele au fost selectate şi propuse de: prof. Ienuţaş Vasile-Şc.Gim. „G. Coşbuc” Baia Mare prof. Rotaru Dumitru-Şc.Gim. „A. Iancu” Baia Mare prof. Erdei Mariana-Şc.Gim. „D. Cantemir” Baia Mare prof. Bunu Iulian-Liceul de Arte, Baia Mare

Clasa a VII-aVarianta II

1.a) Ştiind că

1a+ 1

b= 1

2015, a , b∈Q¿

, arătaţi că √( a

31−65)( b

31−65)

este număr rațional.

64

Page 65: SOCIETATEA DE ŞTIINŢE MATEMATICE DIN - ISJ … matematica 2015... · Web viewpentru punctaj maxim la sectiunea liceu Zelina Mihai, clasa a X-a, C.N. „V. Lucaciu”, prof. Boroica

b) Aflați numerele întregi x şi y care verifică relația 169 x2−13 xy+12 y=119 .(Supliment Gazeta Matematică nr.10/2014)

2. Demonstrați că: 1

√1⋅3 (√1+√3 )+ 1√3⋅5 (√3+√5 )

+. ..+ 1√ (2n−1 )⋅(2n+1 ) (√2n−1+√2 n+1 )

≤ 14

, n∈N ¿

3. În triunghiul ABC cu m ( A )=90∘şi m (C )=30∘

, considerăm bisectoarea [BE, E∈ [ AC ] şi un punct D pe [BC] astfel încât BC=3⋅BD . Dacă {O }=BE∩AD şi F este mijlocul lui [AB], arătați că F, O, C sunt coliniare.

(Gazeta Matematică nr.11/2014)

4. Prin vârful B al Δ ABC se duce o dreaptă care intersectează mediana [ A A' ] în M. Paralela prin M la AB intersectează BC în N , iar AN intersectează BM în P. a) Să se arate A

' P că este paralelă cu AC .

b) Dacă 3 N A '=B A ' şi SABC=108 cm2

, aflați SA ' MN .

c) Arătați că S

AC { A ' P'=34

S ABC¿, unde {P' }=A ' P∩AB .

Subiectele au fost selectate şi propuse de: prof. Mihali Marinela –Liceul Borşa prof. Urda Maria – Şc. Gim. Nr. 1, Moisei

Clasa a VIII–a1. Pe planul triunghiului ABC, cu m (∢C )=30° şi m (∢ A )=90° şi perimetrul egal cu 24+24√3cm se ridică perpendiculara AM=12cm. a) Arătaţi că AB=8√3cm.b) Aflaţi distanţa de la punctul M la dreapta BC.c) Determinaţi măsura unghiului dintre planele (MBC) şi (ABC).

(Prelucrare Supliment Gazeta Matematică nr. 1/2015)2. Se consideră expresia E(x; y) = (x + y)2 − 2(x + y + xy) + 18a) Pentru ce valori ale lui x și y valoarea numerică a expresiei este minimă?b) Aflați valoarea numerică a expresiei E(x; y) știind că numerele x și y verifică egalitatea

√ x2−4 x+4+√4−4 y+ y2= 2xy − x2 – y2

3. Pikachu se antrenează pentru luptă făcând salturi în sensul pozitiv al axei Ox, pornind iniţial din O. Primul salt are lungimea egală cu 1u.m. și fiecare salt are o

65

Page 66: SOCIETATEA DE ŞTIINŢE MATEMATICE DIN - ISJ … matematica 2015... · Web viewpentru punctaj maxim la sectiunea liceu Zelina Mihai, clasa a X-a, C.N. „V. Lucaciu”, prof. Boroica

lungime dublă faţă de precedentul. Dacă Sn reprezintă intervalul de pe axă corespunzător celui de-al n−lea salt, se cere:a) Precizaţi: S1 , S2 , S3

b) Găsiţi S10∩ Nc) Justificaţi dacă este posibil ca Pikachu să se întoarcă, la un moment dat, în locul de plecare, dintr-un singur salt efectuat în sens contrar.4. Fie piramida patrulateră regulată VABCD, AC¿ BD = {O} și P, Q ∈ (VO). Dacă AP¿ CV = {E}, CP¿ AV = {F}, BQ¿ DV = {S} și DQ¿ BV = {T}, aflați măsura unghiului determinat de dreptele EF și ST.

(Gazeta Matematică nr. 11 / 2014)

Subiectele au fost selectate şi propuse de: prof. Neaga Nadina-Șc.Gim.,,Dr. V. Babeș” Baia Mare prof. Știru Aurica-Șc.Gim. ,,N. Stănescu, Baia Mare prof. Mihalca Cristina-Șc.Gim. ,,N. Stănescu” Baia Mare prof. Breitkopf Marieta-Șc.Gim. ,,N. Stănescu” Baia Mare

Clasa a IX-a M1Varianta I

1. Rezolvaţi în R ecuaţia [ x2−4 x+4 ]= [−2 x2+8 x−6 ] .2. Fie x , y , z∈ (0 ,∞ ) cu proprietatea

a) Determinaţi m∈ (0 ,∞ ) maxim pentru care: (1+x2) (1+ y2)

2+x2+ y2 ≥m (1+xy ) , ∀ x , y∈ (0 ,∞ ) cu xy≤1 .

b) Demonstraţi inegalitatea:(1+x2) (1+ y2)

2+x2+ y2 +(1+ x2 ) (1+ z2)

2+ x2+z2 +(1+ y2 ) (1+ z2)

2+ y2+z2 ≥3+xy+xz+ yz2

, ∀ x , yz∈ (0 ,∞ )

3. Fie şirul (xn )n≥0 de numere reale cu proprietatea că xn+m+ xn−m=x3 n pentru

orice numere naturale n şi m cu n≥m . Să se determine x2015 .( Gazeta Matematică nr. 10/2014)

4. Pe laturile AB, BC, CD, DA ale paralelogramului ABCD se consideră punctele M,

N, P, Q astfel încât

AMMB

=l , CNNB

=k , CPPD

=m , AQQD

=p, unde l , k , m, p>0 şi

AP+ AN+ CQ+CM=0 . Arătaţi că dreptele QN, PM şi AC sunt concurente.

66

Page 67: SOCIETATEA DE ŞTIINŢE MATEMATICE DIN - ISJ … matematica 2015... · Web viewpentru punctaj maxim la sectiunea liceu Zelina Mihai, clasa a X-a, C.N. „V. Lucaciu”, prof. Boroica

Subiectele au fost selectate şi propuse de: prof. Fărcaş Natalia-C.N. „ V. Lucaciu” Baia Mare prof. Pop Radu-Liceul Teoretic Sanitar Baia Mare.

Clasa a IX-a M1VariantaII

1. a) Arătaţi că |x−3|+|x−2|+|x−1|+|x+1|+|x+2|+|x+3|≥ 12 ,∀ x∈R .

b) Se consideră a ,b∈R astfel încât şi

.2. a) Să se arate că {n ∙ a }= {n∙b }, ∀n∈N ¿.

b) Să se determine a , b∈R

, dacă, în plus are loc .

3. Se consideră o mulţime G⊂R care satisface simultan proprietăţile:a)

b)

c) .

Arătaţi că (Gazeta Matematică nr.12/2014)

4. Se consideră triunghiul şi punctele şi - mijlocul

lui . Să se arate că dacă , atunci şi sunt mediane.

Subiectele au fost selectate şi propuse de: prof. Boroica Gabriela-C.N.”V. Lucaciu” Baia Mare prof. Muşuroia Nicolae-C.N. „Gh. Şincai” Baia Mare

Clasa a X-a M1Varianta I

1. Fie A⊂R cu cel puțin două elemente și funcția astfel încât ,

.

a) Să se demonstreze că nu poate fi strict descrescătoare.b) Dacă A=R , să se arate că există o infinitate de funcții bijective care verifică relația din enunţ.

67

Page 68: SOCIETATEA DE ŞTIINŢE MATEMATICE DIN - ISJ … matematica 2015... · Web viewpentru punctaj maxim la sectiunea liceu Zelina Mihai, clasa a X-a, C.N. „V. Lucaciu”, prof. Boroica

c) Să se determine , dacă este finită.

2. Să se arate că (prof. Covaciu Traian)

3. Să se demonstreze că dacă , atunci există o infinitate de perechi de

numere reale strict pozitive astfel încât:

a) .

b) . (prof. Heuberger Dana )

4. Fie și centrul cercului circumscris, respectiv ortocentrul triunghiului . Demonstrați că dacă și , atunci .

( Gazeta Matematică nr. 12 / 2014)

Subiectele au fost selectate şi propuse de: prof. Heuberger Dana-C.N. „Gh. Șincai” Baia Mare prof. Covaciu Traian-C.N. „V. Lucaciu” Baia Mare

Clasa a X-a M1Varianta a II-a

1. a) Dacă , să se arate că:

.b) Să se rezolve ecuația următoare, în mulțimea numerelor reale:

. (prof. Longaver Ludovic)

2. Să se rezolve ecuația .

3. Arătați că dacă z∈C" left lbrace - ital i , `i right rbrace } {¿ este de modul 1, atunci numărul este real. Care numere reale sunt de această formă?

(Gazeta Matematică nr. 11 / 2014)

4. Fie triunghiul și punctele astfel încât e mijlocul lui ,

e mijlocul lui , , ,

68

Page 69: SOCIETATEA DE ŞTIINŢE MATEMATICE DIN - ISJ … matematica 2015... · Web viewpentru punctaj maxim la sectiunea liceu Zelina Mihai, clasa a X-a, C.N. „V. Lucaciu”, prof. Boroica

și ∠DAB≡∠EBC . Să se demonstreze că ∠ ADM ≡∠BEN dacă și numai dacă Δ ABC este echilateral.

( prof. Heuberger Dana)Subiectele au fost selectate şi propuse de:

prof. Heuberger Cristian-C.N. „Gh. Șincai” Baia Mare prof. Longaver Ludovic-Lic.Teor. „N. Làszló” Baia Mare prof. Cioclu Costel-Lic.Teor. „E. Racoviță” Baia Mare

Clasa a XI–a M1

1.Se consideră matricea

A=(1 0 01 1 00 0 1 )∈M3 (R )

.

a) Să se determine a,b∈R astfel încât .

b) Să se arate că det ( An−nA )≠0 ,∀n∈N , n≥2 .

2. Șirurile și de numere reale sunt definite prin și

,∀ n∈N .

a) Să se arate că și yn+2−3 yn+1+ yn=0 ,∀n∈N .

b) Să se demonstreze că ecuația are o infinitate de soluții în R×R .

3. Fie A , B∈M3 (R ) astfel încât .

a)Să se demonstreze că ∃ b , c ,d∈R astfel încât det (B+xA )=x3+b x2+cx+d ,∀ xϵ R .

b) Dacă , demonstrați că∃ x0∈ R astfel încât .

4. Se consideră șirul definit prin și .

a) Să se demonstreze că șirul este divergent.

69

Page 70: SOCIETATEA DE ŞTIINŢE MATEMATICE DIN - ISJ … matematica 2015... · Web viewpentru punctaj maxim la sectiunea liceu Zelina Mihai, clasa a X-a, C.N. „V. Lucaciu”, prof. Boroica

b) Să se calculeze .

Subiectele au fost selectate şi propuse de: prof. Petruțiu Crina-C. N. „ Gh. Şincai” Baia Mare prof. Boroica Gheorghe-C. N. „ Gh. Şincai” Baia Mare prof. Bojor Florin-C.N. „Gh. Şincai” Baia Mare

Clasa a XII–a M1Varianta I

1. Pentru x , y∈G=(1 , 3) definim x∗y=2 xy−3 x−3 y+6xy−2x−2 y+5 .

a) Arătaţi G este parte stabilă în raport cu „¿”.b) Presupunând că ¿ este grup abelian arătaţi că ¿.

c) Dacă n∈N , n ≥ 2,calculaţi En=

137∗33

17∗…∗4 n2−3

2 n2−1

.

(R.M.T. nr.1/2015)

2. Fie ¿¿ un grup şi mulţimea Z(G )= {x∈G| xy= yx ,∀ y∈G } . Dacă x2=e ,

pentru orice x∈G−Z (G) , atunci G este comutativ. (Gazeta Matematică nr. 12 / 2014)

3. Să se determine primitiva F a funcţiei f :(−π4

, 3 π4 )→ R , f ( x )= sinx

1+sin 2x cu

proprietatea F (0 )=1+√2 ln (√2−1)2

.

( Szollosy Gheorghe)4. Să se determine funcţiile f : R → R derivabile, cu f (0 )≠ 0 , care verifică

∫0

x

f ( t ) dt= x2

(f ( x )+f (0)) ,∀ x∈ R.

Subiectele au fost selectate şi propuse de: prof. Pop Vesel Floare-Lic.Teor. „B. Vodă” Vişeu de Sus prof. Gherasin Gheorghe-Lic. Ped. „Regele Ferdinand” Sighetu Marmaţiei prof. Giurgi Vasile-C.N. „D. Vodă” Sighetu Marmaţiei

70

Page 71: SOCIETATEA DE ŞTIINŢE MATEMATICE DIN - ISJ … matematica 2015... · Web viewpentru punctaj maxim la sectiunea liceu Zelina Mihai, clasa a X-a, C.N. „V. Lucaciu”, prof. Boroica

Clasa a XII-aVarianta II

1.a) Calculaţi ∫ sinx+cosxex+acosx

dx , x∈(0 , π2 ) , a>0.

b) Dacă f : [0,1]→ R este o funcţie continuă şi n∈N , n ≥ 2, arătaţi că

2∫0

1

f ( x ) dx−n∫0

1

f 2 (xn )dx≤ n2n−1

.

2. Fie funcţia f : ( 0 , ∞) → R , f ( x )=(1+ 1x ) ln x4+2 x3−2 x+1

x2 . Să se determine

primitiva F a funcţiei, ştiind că F (1+√52 )=2 ln 5+ π

2.

3. Fie G un grup cu n elemente şi p cel mai mic divizor prim al lui n. Să se arate că grupul G este abelian dacă şi numai dacă pentru orice a ,b∈G există k∈ {1,2 , …, p−1 }, astfel încât ab=bk a.

(Gazeta Matematică nr. 6-7-8/2014)4. Fie (G ,∙ )un grup şi mulţimea H n= {x∈G /xn=e} , n∈N . a) Să se arate că H 2 este subgrup al lui G dacă şi numai dacă xy= yx , ∀ x , y∈H 2 .b) Dacă p este un număr prim cu proprietatea că H p are cel mult p elemente atunci H n={e } sau H p este un subgrup al lui G izomorf cu ¿.

Subiectele au fost selectate şi propuse de: prof. Pop Vesel Floare-Lic.Teor. „B. Vodă” Vişeu de Sus prof. Gherasin Gheorghe-Lic.Ped. „Regele Ferdinand” Sighetu Marmaţiei prof. Giurgi Vasile-C.N. „D. Vodă” Sighetu Marmaţiei

OLIMPIADA NAŢIONALĂ DE MATEMATICĂ - etapa judeţeană14 martie 2015Clasa a IV – a

1. a) Aflaţi valoarea lui a din egalitatea[(280 : 4 – a ) x 2 + 100 : 5 ] : 3 = [(60 : 6 + 5a) x 2 + 4 x 5 ] : 3

b) Suma a patru numere naturale consecutive este egală cu 54. Aflaţi cele patru numere.2. 2 jachete şi 3 cămăşi de acelaşi fel costă 161 de lei, iar 2 cămăşi şi 4 jachete costă 222 lei. Ce rest vei primi de la 3 bancnote a câte 100 lei, dacă doreşti să cumperi 3 jachete şi 4 cămăşi?3. Tabla de şah din figura de mai jos este împărţită în 8 x 8 pătrate egale. În pătratul din stânga jos (A1) se află o monedă. Doi copii joacă următorul joc : pe rând, fiecare dintre ei mută moneda din locul în care se afla pe un pătrat oarecare situat la dreapta,

71

Page 72: SOCIETATEA DE ŞTIINŢE MATEMATICE DIN - ISJ … matematica 2015... · Web viewpentru punctaj maxim la sectiunea liceu Zelina Mihai, clasa a X-a, C.N. „V. Lucaciu”, prof. Boroica

pe aceeaşi linie sau pe un pătrat oarecare situat deasupra, pe aceeaşi coloană. Câştigă jucătorul care plasează moneda în pătratul din dreapta sus (H8). Arătaţi că al doilea jucător poate câştiga, oricum ar juca primul jucător.

87654321

A B C D E F G H

4. Elevii unei clase merg pe o potecă de munte, unul în spatele celuilalt. Când Andrei a ajuns la cabană, în cabană se aflau deja jumătate din numărul elevilor aflaţi încă pe traseu. Bianca a sosit a zecea după Andrei, iar după ea au rămas de două ori mai puţini elevi decât cei ajunşi înaintea ei la cabană. Câţi elevi sunt în acea clasă ?

Subiectele au fost selectate şi propuse de: prof. Bretan Andrei–Şc.Gim. „N. Iorga” Baia Mare Fornvald Natalia-Inspector şcolar pentru învățământ primar prof. Şteţco Maria-Lic.Ped. „Regele Ferdinand, Sighetu Marmaţiei

CONCURSUL NAȚIONAL DE MATEMATICĂ APLICATĂ „ADOLF HAIMOVICI”–etapa locală

28 februarie 2015

Clasa a IX–aFiliera teoretică - Profil real - Specializarea Ştiinţe ale naturii

1. a) Fie numere reale pozitive. Demonstrați că .b) Demonstrați că pentru orice număr natural , numărul se divide cu 64.2. Se consideră propozițiile:

„Ecuația are o infinitate de soluții numere raționale”.

„Ecuația are trei soluții numere naturale”.

72

Page 73: SOCIETATEA DE ŞTIINŢE MATEMATICE DIN - ISJ … matematica 2015... · Web viewpentru punctaj maxim la sectiunea liceu Zelina Mihai, clasa a X-a, C.N. „V. Lucaciu”, prof. Boroica

a) Stabiliți valoarea de adevăr a propoziției .b) Stabiliți valoarea de adevăr a propoziției .3. Doi călători au de parcurs un traseu lung de . Primul călător parcurge în prima zi , iar apoi parcurge în fiecare zi cu un kilometru mai mult decât în ziua precedentă. Al doilea călător parcurge în prima zi , iar apoi în fiecare zi parcurge dublul distanței parcurse în ziua precedentă. a) Determinați numărul de zile necesare fiecărui călător pentru a parcurge traseul.b) Dacă cei doi călători pornesc în aceeași zi, la sfârșitul cărei zile va fi depășit primul călător de către cel de-al doilea?4. În patrulaterul convex se notează cu mijlocul lui și cu mijlocul lui .

a) Demonstrați că .

b) Demonstrați că egalitatea este adevărată pentru orice punct din planul patrulaterului .

Subiectele au fost propuse şi selectate de: prof. Friedrich Gabriela–C.Ec.”N. Titulescu” Baia Mare prof. Hotea Vasile–C.T. „A. Vlaicu, Baia Mare

Clasa a IX – aFiliera tehnologică –Profil tehnic – toate specializările

1. a) Dintr-o clasă de 30 de elevi de la un liceu, 16 elevi au participat la Concursul Naţional „Adolf Haimovici”, 23 au participat la un concurs de limba şi literatura română şi 5 nu au participat la nici un concurs. Câţi elevi au participat la ambele concursuri? Dar numai la concursul Haimovici? Dar numai la concursul de limba şi literatura română?b) Numerelex , y , z sunt în progresie geometrică şi au suma 114. Numerele x, y + 36, z

sunt în progresie aritmetică. Să se calculeze1x+ 1

y+ 1

z .c) Rezolvaţi ecuaţia

2. a) Demonstraţi că ,

b) Fie a, b, c, d numere reale strict pozitive astfel încât ,

sunt în progresie aritmetică. Să se arate că a, b, c, d sunt în progresie aritmetică.

73

Page 74: SOCIETATEA DE ŞTIINŢE MATEMATICE DIN - ISJ … matematica 2015... · Web viewpentru punctaj maxim la sectiunea liceu Zelina Mihai, clasa a X-a, C.N. „V. Lucaciu”, prof. Boroica

3. Dacă O este centrul cercului circumscris hexagonului regulat ABCDEF , să se

determine numerele realex şi y ştiind că: AB+ AC+ AD+ AE+ AF=x⋅AO şi

BD+ BF= y⋅BO

4. Se consideră numărul a= 3

12⋅22+ 5

22⋅32+…+ 2n+1

n2⋅( n+1)2, n∈N∗¿ ¿

a) Să se determine partea întreagă şi partea fracţionară a numărului a .

b) Să se determine n∈N∗¿ ¿ştiind că {a }=0 , 9999 .

Subiectele au fost propuse şi selectate de:

prof. Borşa Raul–C.Ec. „N. Titulescu” Baia Mare prof. Pop Adria –C.N. „Gh. Şincai” Baia Mare

Clasa a IX–aFiliera tehnologică –Profil servicii,resurse naturale si protectia mediului – toate

specializările profesionale

1. Fie predicatele unare: p ( x ): x−√162+122+√162−122−√16−12

18+2√28=0

,x∈ R

q ( y ) : y−[ 148

:√25256

+(− 1√3 )

2⋅√20 , 25−1]:√0 , 32 (1 )=0

, y∈R .

a) Aflaţi universul de adevăr al predicatelor p ( x ) şi q ( y ) .b) Notăm cu r propoziţia particulară p (1 ) şi cu s propoziţia particulară q (3 ) . Aflaţi valorile de adevăr ale propoziţiilor r, s, r¿ s,r¿ s,r→ s,r↔ s.

2.a) Demonstraţi că

21⋅3

+ 53⋅5

+. ..+ n2+1(2 n−1 ) (2 n+1 )

=n (n+3 )2 (2 n+1 )

,∀ n≥1.

b) Calculaţi [1+√5 ]−{ 1

2−√5 }−4⋅{ 13−√5 }.

3. Fie ABCD un paralelogram şi punctele M∈ [ AB ] ,N∈ [ AC ] astfel încât AM→

=14

AB→

şi AN→

=15⋅AC

.Notăm vectorii AB→=a

şi AD→=b

.

a) Exprimaţi vectorii DN→

şi MN→

în funcţie de vectorii a→

şi b→

.

b) Demonstraţi că punctele M ,N , D sunt coliniare.74

Page 75: SOCIETATEA DE ŞTIINŢE MATEMATICE DIN - ISJ … matematica 2015... · Web viewpentru punctaj maxim la sectiunea liceu Zelina Mihai, clasa a X-a, C.N. „V. Lucaciu”, prof. Boroica

4. O tribună a unui stadion se compune din 41 de rânduri de scaune şi pe fiecare rând următor se află cu 10 locuri mai multe decât pe rândul precedent.În ultimul rând sunt 500 de locuri.a) Câţi spectatori pot intra în această tribună?b) Câţi spectatori vor fi pe rândul 20?

Subiectele au fost propuse şi selectate de către: prof.Podina Camelia–Lic.Teor. „E. Racoviţă” Baia Mare prof Ocean Cristina–Lic.Teor. „E. Racoviţă” Baia Mare

Clasa a IX–aFiliera teoretică - Profil uman - Specializarea Filologie, Ştiinţe Sociale

1. Să se rezolve ecuația: + = .

2. Suma primilor n termeni a unui șir este =2 . a) Determinați b4.b) Arătați că acest șir formează o progresie geometrică și calculați S5.

3. Fie , iar M, N, P mijlocul laturilor și respectiv . Arătați că:

= .

+ + = 0.

4. Fie dreptunghic cu m = , m = , iar m = .Împărțiți triungiul ABC în trei triunghiuri mai mici de arii egale. Dacă ON este perpendiculara ridicată în O pe latura BC, O- mijlocul segmentului

[BC], iar N AB, demonstrați că: ON= .

Subiectele au fost propuse şi selectate de: prof. Grigor Mihai – L. T.” Marmația”, Sighetu Marmaţiei prof. Tivadar Cornel – C. N. „ D.Vodă”, Sighetu Marmaţiei

Clasa a X–aFiliera teoretică - Profil real - Specializarea Ştiinţe ale naturii

1.a) Să se calculeze suma S= ;

b) Să se calculeze produsul … pentru  ;

75

Page 76: SOCIETATEA DE ŞTIINŢE MATEMATICE DIN - ISJ … matematica 2015... · Web viewpentru punctaj maxim la sectiunea liceu Zelina Mihai, clasa a X-a, C.N. „V. Lucaciu”, prof. Boroica

2. a) Dacă și să se verifice egalitatea

b) Să se aducă la o formă mai simplă expresia:

3. a) Fie C astfel încât = =1. Să se arate că

;

b) Dacă = , să se calculeze , unde n N.

4. a) Să se rezolve în C ecuația unde C ;

b) Dacă și demonstrați că soluțiile ecuației de mai sus sunt afixele vârfurilor unui dreptunghi.

Subiectele au fost propuse şi selectate de către: prof. Temian Gavril – C.Ec. „ N. Titulescu”, Baia Mare prof. Birta Adriana- C. T.” A. Saligny”, Baia Mare

Clasa a X–aFiliera tehnologică –Profil tehnic – toate specializările profesionale

1.a) Fie x, y, z astfel încât 2x+2y=2z. Arătați că:i) x, y, z nu pot fi toate egale;ii) există o infinitate de numere întregi astfel încât 2x+2y=2z.b) Dacă x, y, z asfel încât x, z, y nu sunt în progresie aritmetică (în ordinea dată), atunci: 2x +2y > 2z+1 sau 2x+z + 2y+z > 2x+y+1.2.a) Dacă atunci are loc egalitatea:

b) Să se arate că , unde [x] - partea întreagă a numărului real x.3. a) Dacă este o soluţie a ecuaţiei x2 + x + 1 = 0, atunci calculați b) Fie ecuația a2z2 + abz + c2 = 0 a, b, c și z1, z2 soluțiile sale.

Demonstrați că, dacă sau 4.a) Calculați

76

Page 77: SOCIETATEA DE ŞTIINŢE MATEMATICE DIN - ISJ … matematica 2015... · Web viewpentru punctaj maxim la sectiunea liceu Zelina Mihai, clasa a X-a, C.N. „V. Lucaciu”, prof. Boroica

b) Determinați numerele complexe z cu proprietățile .

Subiectele au fost propuse şi selectate de: prof. Pop Mariana–Lic.Teor. „P. Rareş” Târgu Lăpuş prof. Ionaş Mirela–Lic.Teor. „P. Rareş” Târgu Lăpuş prof. Leşe Teodor–Lic.Teor. „P. Rareş” Târgu Lăpuş

Clasa a X–aFiliera tehnologică–Profil servicii, resurse naturale și protecția mediului – toate

specializările profesionale

1.a) Să se arate că .

b) Fie . Arătaţi că, dacă , atunci .

2. a) Demonstraţi că .

b) Rezolvaţi ecuaţia: .3. Arătaţi că expresia

nu depinde de .4. a) Pe o sârmă, distanţate la câte 5 cm, se agaţă beculeţe colorate după regula: roşu, roşu, verde, verde, verde, roşu, roşu, verde, verde, verde şi aşa mai departe, mereu după două beculeţe roşii urmând trei beculeţe verzi. Care este distanţa dintre al treilea beculeţ roşu şi al 21-lea beculeţ roşu?b) Aveţi la dispoziţie un vas de 4 şi un vas de 9 . Cum puteţi măsura de la robinet 6 de apă cu cele 2 vase?

Subiectele au fost propuse şi selectate de către: prof. Bedeoan Loredana–C.N.”D. Vodă” Sighetu Marmaţiei prof.Bilaşco Ioan–L.T. Forestier Sighetu Marmaţiei

Clasa a X– aFiliera teoretică - Profil uman - Specializarea Filologie, Ştiinţe Sociale

1.a) Efectuaţi: (a18−1)⋅( a

18+1)⋅(a

14 +1 )⋅(a

12+1)⋅(a+1 ) , a∈R .

b) Comparaţi numerele 999910 şi 1040

.

77

Page 78: SOCIETATEA DE ŞTIINŢE MATEMATICE DIN - ISJ … matematica 2015... · Web viewpentru punctaj maxim la sectiunea liceu Zelina Mihai, clasa a X-a, C.N. „V. Lucaciu”, prof. Boroica

2.a) Determinaţi a∈Q dacă √ 3√33√9

=3a

.

b) Arătaţi că numărul √3−√29−12√5−√5 este întreg.3. Calculaţi:

a) E= x ln y

y ln x ,unde x,y∈ (0;+∞ )− {1 }

b) M=aln

bc⋅b

lnca⋅c

lnab ,a , b , c∈ (0 ;+∞ )−{1}

4. Un patrulater convex are 2 diagonale, un pentagon convex are cinci diagonale. Câte diagonale are un poligon convex cu 6, 7 respectiv 8 vârfuri? Este posibil ca un poligon convex să aibă 50 de diagonale? Explicaţi.

Subiectele au fost propuse şi selectate de: prof. Balogh Erika–C.T. „C.D.Neniţescu” Baia Mare prof. Dale Camelia–C.T. „Transilvania” Baia Mare

Clasa a XI–aFiliera teoretică - Profil real - Specializarea Ştiinţe ale naturii

1. Se consideră matricea A ( x )=( 1 0 0x 1 0

x2+2 2 x 1) , x∈R

a) Să se calculeze A ( x ) ∙ A ( y ) , x , yϵ R

b) Să se arate că matricea este inversabilă (∀)x∈R și să se verifice că ❑❑

A−1 ( x )=A ( x ) ,(∀) x∈ R

c) Să se calculeze

2. Fie A∈M 3(R) o matrice pătratică simetrică având elementele diagonalei principale egale între ele şi suma elementelor de pe fiecare coloană egală cu k∈N ¿.a) Să se dea două exemple de astfel de matrici .

b) Să se arate că este un pătrat perfect.

3. a) Să se determinea , b , c∈R

astfel încât

78

Page 79: SOCIETATEA DE ŞTIINŢE MATEMATICE DIN - ISJ … matematica 2015... · Web viewpentru punctaj maxim la sectiunea liceu Zelina Mihai, clasa a X-a, C.N. „V. Lucaciu”, prof. Boroica

b) Să se calculeze

4. a) Se consideră funcţia

f : (−∞ , 4)→ R

, . Să se

determinea∈R astfel încât funcția să fie continuă pe .

b) Să se determine funcția f :R→R continuă în ,cu şi

f ( x )−f ( x2 )=x ,∀ x∈R

Subiectele au fost propuse şi selectate de către: prof. Brisc Viorica–C.T. „A. Saligny” Baia Mare prof. Pop Anca–C.T. „G. Bariţiu” Baia Mare

Clasa a XI–aFiliera tehnologică–Profil tehnic–toate specializările profesionale

1. Într-un sistem de axe xOy, considerăm punctele O(0,0) şi număr natural nenul.a) Determinați ecuația dreptei .b) Determinați aria triunghiului .c) Arătați că punctele sunt coliniare.

2. Se considera matricea A= şi matricea X(a)= +aA, a număr real.a) Demonstrați că X(a)X(b)=X(a+b+5ab).b) Aflaţi c număr real astfel încât X(a)X(c)=X(c), oricare ar fi a şi c numere reale.

c) Determinaţi t număr real astfel încât X( )X( )…X( )X( )=X(t).

3. a) Calculatilimx→3

√x2−x−√6x−3

b) Studiaţi dacă funcţiaf : R−{−1 }→R , f ( x )= 3 x2+1

|x+1| are limită în x0=−1 .

c) Calculaţilim

x→−∞

2 x+1√ x2+x+2

79

Page 80: SOCIETATEA DE ŞTIINŢE MATEMATICE DIN - ISJ … matematica 2015... · Web viewpentru punctaj maxim la sectiunea liceu Zelina Mihai, clasa a X-a, C.N. „V. Lucaciu”, prof. Boroica

4.a) Determinaţivalorile reale ale numerelora şib astfel încât funcţia

f : R−{−2 }→R , f ( x )=ax2+bx+1x+2

să aibă asimptota oblică y=x+2 spre +∞ .

b) Determinaţi valorile parametrului real a pentru care funcţia

f : R→R , f ( x )=¿ {√x2+a2 , x≤0 ¿ ¿¿¿

are limita în x0=0 .

c) Calculaţilimx→0

ln (1+sin 2 x )3 x

.Subiectele au fost propuse şi selectate de .:

prof. Borcut Marin – C. Ec. „ P. Vitezu”, Cavnic prof Ocean Cristina –Lic. Teor. „ E. Racoviţă”, Baia Mare

Clasa a XI–aFiliera tehnologică –Profil servicii, resurse naturale și protecția mediului – toate

specializările profesionale

1. Fie matricea A=( 1 −√3

√3 1 )a) Calculaţi A2 și A3. b) Determinaţi A2016.

2. În reperul cartezian se dau punctele , unde .

a) Să se determine n pentru care punctele au coordonatele egale . b) Să se calculeze aria triunghiului A1A2A3. 3. a) Să se determine mulțimea A R astfel încât pentru orice a A și pentru orice x

R, să avem ax2 + x + 3 0. Pentru fiecare a A să se calculeze

.

b) Se consideră funcția f:( ;1] [2; ) R, f(x) = . Să se determine ecuația asimptotei spre - ∞ la graficul lui f.

4. Fie matricea , x,y R.

a) Calculați .

80

Page 81: SOCIETATEA DE ŞTIINŢE MATEMATICE DIN - ISJ … matematica 2015... · Web viewpentru punctaj maxim la sectiunea liceu Zelina Mihai, clasa a X-a, C.N. „V. Lucaciu”, prof. Boroica

b) Rezolvaţi ecuaţia det A(x;1) = 0.

Subiectele au fost selectate şi prelucrate de: prof. Cristescu Ştefan–C.T. „G. Bariţiu” Baia Mare prof. Mic Leon–C.T. „A. Vlaicu” Baia Mare

Clasa a XI–aFiliera teoretică - Profil uman - Specializarea Ştiinţe Sociale

1. Un profesor îşi ia din catalogul unei clase mediile la matematică pe semestrul trecut în vederea unor prelucrări statistice. Acestea sunt: 6,7,7,5,9,8,4,10,7,5,6,6,7,8,4,4,6,5,8,6,7,5,6,9,7.a) Să se completeze un tabel care conţine rubricile: Nota, Frecvenţa absolută, Frecvenţa relativă, Frecvenţa cumulată.b) Câţi elevi au notele între 7 şi 10? Indicaţi procentul lor.2. a) Ce preţ are un sortiment de cafea dacă un kilogram de amestec format din 20 kg din acest sortiment şi 5 kg de 20 lei/kg costă 16 lei?b) O editură pune în vânzare prin nouă librări loturi egale de cărţi. Timpul de epuizare a acestor loturi este redat în tabel:Librărie 1 2 3 4 5 6 7 8 9Nr. zile 49 50 48 50 51 54 49 50 49Să se calculeze dispersia şi abaterea medie pătratică.3.a) Un melc urcă în timpul zilei pe un copac 3m şi alunecă noaptea 2m. După câte zile ajunge în vârful copacului înalt de 10m.b) Aceeaşi problemă dacă ziua urcă 3m, iar noaptea coboară 1m, trebuind să „escaladeze” un copac de n metri, n N, , n impar.4. O curea de transmisie leagă două roţi cu lungimile cercurilor de 35 cm şi respective 60 cm.a) Dacă prima roată face 24 rotaţii, câte face cea de a doua?b) După câte rotaţii vor reveni în poziţia iniţială?

Subiectele au fost propuse şi selectate de: prof. Gavrilaş Emilia–Lic.Teor. „P. Rareş” Târgu Lăpuş prof. Miholca Gavril–Lic.Teor. „P. Rareş” Târgu Lăpuş prof. Boga Ovidiu–L.T. „Gr. Moisil” Târgu Lǎpuş

Clasa a XII–aFiliera teoretică - Profil real - Specializarea Ştiinţe ale naturii

1. Definim pe R legea de compoziţie „*” prin x∗y=ln (e x+e y ) ,

x , y∈R

81

Page 82: SOCIETATEA DE ŞTIINŢE MATEMATICE DIN - ISJ … matematica 2015... · Web viewpentru punctaj maxim la sectiunea liceu Zelina Mihai, clasa a X-a, C.N. „V. Lucaciu”, prof. Boroica

a) Arătaţi că legea de compoziţie „*” este comutativă, asociativă, dar nu admite element neutru.

b) Demonstraţi că x+( y∗z )= (x+ y )∗( x+z ) , oricare ar fi x , y , z∈R .

2. Se consideră matricea A=( 2 1

−2 −1 )şi mulţimea G= {Xa=I 2+aA , a>−1}.

a) Arătaţi că ¿¿ este grup abelian.

b) Demonstraţi că funcţia f :G→R , f (Xa )= ln (a+1 ) este izomorfism de la grupul

¿¿ la grupul ( R ,+ ) .3. Determinaţi a ,b∈R , astfel încât funcţia F :R→R ,

F (x )={ x2+ax+1, x≤1x3+x2−4 x+b , x>1

să fie o primitivă pentru o funcţie

f :R→R . Aflaţi, apoi funcţia f.

4. Fie şirul ( I n )n≥1 , I n=∫

0

1

xn sin x dx.

a) Calculaţi I 1 .

b) Arătaţi că I n+n (n−1 ) I n−2=nsin 1−cos1 , ∀n∈N , n≥3 .

Subiectele au fost propuse şi selectate de: prof. Zlampareţ Mihaela–C.Ec. „N. Titulescu” Baia Mare prof. Polgar Corina–C. T. „C.D. Nenitescu” Baia Mare

Clasa a XII-aFiliera tehnologică – Profil tehnic – toate specializările

1. Pe R se consideră legea de compoziţie x,y .a) Să se calculeze ,b) Să se arate că ,c) Să se găsească două numere a,b cu proprietatea că a .

2. Se consideră mulţimeaa) Arătaţi că .b) Demonstraţi că G este grup în raport cu înmulţirea matricelor pătratice de ordinul doi.

82

Page 83: SOCIETATEA DE ŞTIINŢE MATEMATICE DIN - ISJ … matematica 2015... · Web viewpentru punctaj maxim la sectiunea liceu Zelina Mihai, clasa a X-a, C.N. „V. Lucaciu”, prof. Boroica

3. Se consideră , .a) Calculaţi şi .

b) Arătaţi că = .

4. Se consideră , .a) Să se determine o primitivă F a lui f cu proprietatea că F( .b) Să se arate că F este crescătoare şi concavă, pentru F determinată la punctul a).c) Să se calculeze suma: , unde F este primitiva determinată la punctul a).

Subiectele au fost propuse şi selectate de: prof. Huminiuc Monica–Lic.Teor. „E. Racoviță” Baia Mare prof. Tărău Rodica–Lic.Teol. Penticostal Baia Mare

CLASA a XII–AFiliera tehnologică–Profil servicii, resurse naturale şi protecţia mediului–toate

specializările profesionale

1. Fie mulţimea Z8= {0 , 1, 2 , 3 , 4 , 5 , 6 , 7 }

a) Se consideră S suma soluţiilor ecuaţiei 4 x+1=1 şi P produsul elementelor

inversabile din Z8 în raport cu înmulţirea. Să se calculeze S + P.

b) Determinaţi mulţimea valorilor funcţiei f : Z8→Z8 , f ( x )=x3−x .

c) Determinaţi numărul elementelor mulţimii H= {x10+1 |x∈Z8 } .2. Pe mulţimea numerelor raţionale se defineşte legea de compoziţie

x∗y=12

(xy+x+ y−1 ) , ∀ x , y∈R. Notăm cu mulţimea numerelor întregi

impare.

a) Să se arate că x∗y=12¿ .

b) Să se arate că

c) Să se calculeze x∗x∗x…∗x ,⏟ x∈ R, unde n∈R.

3. Se consideră funcţiile f , g : (0 ,∞ )→R , f ( x )=ln x

√x şi g ( x )=2√x (ln x−2 ) .a) Demonstraţi că funcţia g este o primitivă a funcţiei f .

83

Page 84: SOCIETATEA DE ŞTIINŢE MATEMATICE DIN - ISJ … matematica 2015... · Web viewpentru punctaj maxim la sectiunea liceu Zelina Mihai, clasa a X-a, C.N. „V. Lucaciu”, prof. Boroica

b) Calculaţi∫1

4 f ( x )ln x

dx

c) Calculaţi ∫ f ( x )dx

4. Se consideră funcţia f :R→R , f ( x )=ex−x−1 . Admitem că f ( x ≥ 0 ) ,∀ x∈R .

a) Calculaţi .

b) Demonstraţi că .

Subiectele au fost propuse şi selectate de: prof. Bojor Meda-C.N. „Gh. Şincai” Baia Mare prof. Podina Camelia-Lic.Teor. „E. Racoviţă” Baia Mare

Clasa a XII-aFiliera teoretică – Profil uman – Specializarea Ştiinţe Sociale

1.

Se consideră mulţimea S=¿¿

ca suma elementelor pe fiecare linie , respectiv coloana este 1¿¿a) Verificaţi dacă I2 S.

b) Demonstraţi că produsul elementelor oricărei matrice din S este cel mult

116 .

2. Fie matricea

A( x )=¿ ( 2 0 0 ¿ ) ( 0 1 0 ¿ ) ¿¿

¿¿ M3(R).

a) Să se arate că A3(2)=3 A2(2)−2 A (2) .b) Să se determine x∈ R , ştiind că A

3 ( x )−3 A2 ( x )+2 A ( x )=O3 .

3. Fie matricea

A=¿ ( 1 1 1 ¿ ) ( 0 1 1 ¿ ) ¿¿

¿¿ M3(R),

84

Page 85: SOCIETATEA DE ŞTIINŢE MATEMATICE DIN - ISJ … matematica 2015... · Web viewpentru punctaj maxim la sectiunea liceu Zelina Mihai, clasa a X-a, C.N. „V. Lucaciu”, prof. Boroica

a) Să se calculeze A2

şi A3

.

b) Să se studieze dacă există numerele reale astfel încât A3=xA2+ yA+ I3 .

4. Fie matricea A =

( a b ¿ )¿¿

¿¿ M2(R), a ,b , c ,d∈R cu a+d≠0 .

a) Demonstraţi că A2−(a+d )A+(ad−bc ) I 2=O2 .

b) Să se demonstreze că matricea B∈M 2(R ) comută cu matricea A , dacă şi numai

dacă matricea B comută cu matricea A2

.

Subiectele au fost propuse şi selectate de: prof. Pop Adela–C. T. ,,A. Vlaicu” Baia Mare prof. Berciu Ioan–L.T.A. ,,A. Berinde” Seini

CONCURSUL DE MATEMATICĂ „SIGMA”EDIŢIA A XX-A, 09.05.2014, SIGHETU-MARMAŢIEI

Clasa a IV-a1.a) Dacă a+b=300 ;b+c=250 şi c+d=5 calculaţi A=5× a+7× b+5 × c+3 × d (Gazeta Matematică nr. 1/2015)b) Aflaţi a din egalitatea:6+26× 10 : [5+4 × (18+4002 :a×6 ):24 ]−22=10 c) Pe două rafturi sunt 88 de cărţi. După ce de pe un raft s-au luat 16 cărţi, iar de pe celălalt s-au luat 24, pe unul din rafturi au rămas de 3 ori mai multe cărţi decât pe celălalt. Câte cărţi au fost iniţial pe fiecare raft?

2. Fie şirul de numere naturale: 2,5,9,14,20 ,…a) scrieţi următorii trei termeni ai şirului.b) calculaţi diferenţa dintre termenul de pe poziţia 20 şi cel de pe poziţia 10.c) stabiliți daca 2015 este termen al șirului și dacă da precizați pe ce poziție.

(R.M.T. nr. 1/2015 – enunţ modificat)3.a) Suma a trei numere naturale este 60. Aflaţi numerele ştiind că dacă adăugăm 3 la jumătatea primului număr, adunăm 6 la jumătatea celui de-al doilea număr şi adunăm 9 la jumătatea celui de-al 3-lea număr obţinem numere consecutive. b) O bunică are 2 nepoţi. Vârsta bunicii este un număr de 2 cifre. Cifrele reprezintă vârsta celor 2 nepoţi. Ce vârstă are bunica şi fiecare nepot dacă peste 2 ani suma vârstelor lor este de 75 de ani.

Subiectele au fost selectate şi propuse de prof. Bedeoan Loredana-C.N. „D. Vodă” Sighetu Marmaţiei

85

Page 86: SOCIETATEA DE ŞTIINŢE MATEMATICE DIN - ISJ … matematica 2015... · Web viewpentru punctaj maxim la sectiunea liceu Zelina Mihai, clasa a X-a, C.N. „V. Lucaciu”, prof. Boroica

Clasa a V-a

1. Fie numerele naturale nenule a , b , c , d şi M=1a+ 2

b+ 3

c+ 4

d .

a) Există valori naturale nenule ale numerelor a ,b , c , d şi pentru care M este număr natural?b) Să se demonstreze că M poate lua valorile 1,2,3,4,5.c) Câte valori numere naturale poate lua M?

(Gazeta Matematică nr. 3/2015)2. Să se determine toate numerele naturale de forma abc , ştiind că: abc=a+b2+c3 . (prof. Gherasin Gheorghe)3. Considerăm numerele A=5n2+n+9n2+n şi B=m8+2 (15 p2+1 ) unde n , m , p sunt numere naturale nenule. Arătaţi că oricare ar fi n , m , p∈N , produsul A ∙ B nu este pătrat perfect.

Subiectele au fost selectate şi propuse de prof. Tomoiagă Ioan-C.N. „D. Vodă” Sighetu Marmaţiei

Clasa a VI-a1. a) Numerele naturale x , y , z sunt direct proporţionale cu numerele prime p , q , r.

Stiind că p<q<r, p+q+r=10 şi x+ y+z=100, calculaţi suma ultimelor 2015 cifre ale numărului a=x2014+ y2014+z2014.b) Determinaţi numerele naturale x , y , z ştiind că xy+ yz+2 zx=3 xyz.2. a) Fie a ,b , c numere raţionale astfel încât a ∙ b ∙ c=1. Calculaţi valoarea expresiei

E=20 a+2015a+ab+1

+ 20 b+2015b+bc+1

+ 20 c+2015c+ac+1

.

(Gazeta Matematică nr. 3/2015)

b) Fie a , b numere naturale nenule astfel încât ab=1+ 1

2+ 1

3+…+ 1

101 . Arătaţi că

a−b ⋮ 103.3. FieABC un triunghi isoscel cu [ AB ]≡ [AC ] şi m ( A )=72° . Pe latura AB se iau puncteleD şi E astfel încât ACD ≡ DCE ≡ E CB , iar F∈ (BC ) astfel încât ¿ este bisectoarea BEC.

a) Arătaţi că∆ AFB≡ ∆ CFE.b) Demonstraţi că AF⊥CE .

Subiectele au fost selectate şi propuse de prof. Giurgi Vasile-C.N. „D. Vodă” Sighetu Marmaţiei

Clasa a VII-a86

Page 87: SOCIETATEA DE ŞTIINŢE MATEMATICE DIN - ISJ … matematica 2015... · Web viewpentru punctaj maxim la sectiunea liceu Zelina Mihai, clasa a X-a, C.N. „V. Lucaciu”, prof. Boroica

1.a) Fie n∈N ¿. Arătaţi că: 1

3√2+ 1

5√6+ 1

7 √12+…+ 1

(2n+1 )√n (n+1 )< n

2 n+2 .

(Gazeta Matematică nr. 3/2015)

b) Rezolvaţi ecuaţia: x−62008

+ x−22012

= x−20086

+ x−20122 .

2. Aflaţi numărul soluţiilor ecuaţiei x3−3 y=7, ştiind că x , y∈Z şi |x|≤100

3.a) Fie ∆ ABC cu D proiecţia lui A pe BC. Atunci arătaţi că:i) dacă m (∡C )<90°avem relaţia: AB2=CA2+CB2−2 CB∙ CDii) dacă m (∡C )>90°avem relaţia: AB2=CA2+CB2+2CB ∙CD

b) Fie M un punct pe latura [BC ] a unui triunghi ABC. Arătaţi că:AM2 ∙BC=AB2 ∙MC+AC2 ∙ MB−BC ∙BM ∙ CM

c) Într-un ∆ ABC avem L mijlocul lui [BC ]. Arătaţi că: AL2=2 ( AC2+AB2 )−BC 2

4d) Fie ∆ ABC , AB=c , BC=a ,CA=b şi ma ,mb ,mc lungimea medianelor corespunzătoare laturilor [BC ] , [ AC ] , respectiv [ AB ]. Arătaţi că ∆ ABC este

dreptunghic în A dacă şi numai dacă:mb

2+mc2=5 ∙ ma

2

Subiectele au fost selectate şi propuse de prof. Bedeoan Loredana-C.N. „D. Vodă” Sighetu Marmaţiei

Clasa a VIII-a1. Fie ABCDEFGH un cub cu AB=a şi M∈ ( AC )astfel încât ∆ EMC să fie

isoscel.

a) Aflaţi raportul AMMC

.

b) Calculaţi distanţa de la punctul C la planul (BMO), unde O este centrul cubului.

2. Într-un sistem de axe xOy se ia punctul A(1 ,−1) şi prin A se duce o dreaptă care intersectează axa Ox în B şi axa Oy în C. Determinaţi coordonatele punctelor B şi C ştiind că aria triunghiului OBC este egală cu 2.

(Gazeta Matematică nr. 2/2015)3. a) Să se descompună într-un produs de factori ireductibiliE=bc (b2−c2 ) (a3 x+b3+c3−3 abc )+ca (c2−a2) (a3+b3 x+c3−3abc )+ab (a2−b2) (a3+b3+c3 x−3abc ) . ( Szollosy Gheorghe)

87

Page 88: SOCIETATEA DE ŞTIINŢE MATEMATICE DIN - ISJ … matematica 2015... · Web viewpentru punctaj maxim la sectiunea liceu Zelina Mihai, clasa a X-a, C.N. „V. Lucaciu”, prof. Boroica

b) Dacă a , b , c>0 demonstraţi inegalitatea a

a+2b+2 c+ b

2a+b+2 c+ c

2a+2b+c≥ 3

5 .

Subiectele au fost selectate şi propuse de prof. Berci Ioan - C. N. „D. Vodă”, Sighetu Marmaţiei

Clasa a IX-a

1.a) Să se arate că a+2ba+2c

+ b+2 cb+2 a

+ c+2 ac+2 b

≥3 , ∀a , b , c>0.

(Gazeta Matematică nr. 2/2015)b) Demonstraţi că şirul (an)n ≥1 este progresie aritmetică dacă şi numai dacă:

a1+a2+…an=(a1+an )n

2,∀n ≥ 2.

2. Să se calculeze Sn=∑k=1

n

[√k2−k+3+√k2+k+3 ], n∈N ¿ .

( Szollosy Gheorghe)3. Fie punctele M , N , P situate pe laturile ∆ ABC astfel încât AM=BN=CP. Dacă are loc relaţia AB∙ AT+BC ∙ BT+CA ∙ CT= 0 , unde T este centrul de greutate al triunghiului MNP, arătaţi că ∆ ABC este echilateral.Subiectele au fost selectate şi propuse de prof. Giurgi Vasile-C.N. „D. Vodă” Sighetu Marmaţiei

Clasa a X-a

1. Fie n∈N . Să se arate că în dezvoltarea (x2−2x+2 )n coeficientul lui x

neste

egal cu (−√2 )n∑

k=0

n

(Cnk )2cos (2 k−n ) π

4 .(Gazeta Matematică nr. 1/2015)

2. Să se rezolve în R ecuaţia

1008x−10081007−x = 1

1008(|x|+|x−1|+|x−2|+. ..+|x−2015|)

(prof. Tivadar Cornel)

3. Dacă x≥4

3 , să se calculeze [√ x2−x+√ x2−1+√x2+x−3 x ] , unde [ x ]

reprezintă partea întreagă a numărului x.

88

Page 89: SOCIETATEA DE ŞTIINŢE MATEMATICE DIN - ISJ … matematica 2015... · Web viewpentru punctaj maxim la sectiunea liceu Zelina Mihai, clasa a X-a, C.N. „V. Lucaciu”, prof. Boroica

(Szollosy Gheorghe)

Subiectele au fost selectate şi propuse de prof. Tivadar Cornel-C.N. „D. Vodă” Sighetu Marmaţiei

Clasa a XI-a, a XII-a

1.Se consideră şirul (xn )n≥0 , definit prin x0 , x1∈ (1 ,∞ ) şi xn+1= log3 (1+xn+xn−1) ,∀n∈N ¿

.

a) Demonstraţi că şirul (xn+

12

xn−1)n≥1 este convergent.

b) Demonstraţi că limn→∞

xn=1. (Gazeta Matematică nr. 12/2014)

2. Fie a , b>1 numere date. Câte soluţii are ecuaţia ax+ 1

bx=a+b

?( Szollosy Gheorghe)

3. Determinaţi funcţiile continue f : ( 0 ,∞ )→R care verifică relaţia f ( x )+ f (x2 )+f (x4 )=x8−x+7 ln x ,∀ x∈ (0 ,∞ )

Subiectele au fost selectate şi propuse de prof. Tivadar Cornel-C.N. „D. Vodă” Sighetu Marmaţiei

CONCURSUL „ARGUMENT”Ediția a VI-a

Clasa a V-aLa problemele 1-8, scrieţi pe foaia de concurs doar litera corespunzătoare răspunsului corect.

1. Numărul este egal cu: a) 10 b) 7 c) 9 d) 8

2. Numărul este egal cu: a) 25 b) 41 c) 521 d) 93. Câte numere de trei cifre au exact două cifre identice ? a) 900 b) 891 c) 270 d) 2434. Diferenţa dintre cel mai mic număr natural de 4 cifre diferite două câte două şi cel mai mare număr natural de 3 cifre diferite două câte două este egal cu: a) 216 b) 24 c) 34 d) 36

89

Page 90: SOCIETATEA DE ŞTIINŢE MATEMATICE DIN - ISJ … matematica 2015... · Web viewpentru punctaj maxim la sectiunea liceu Zelina Mihai, clasa a X-a, C.N. „V. Lucaciu”, prof. Boroica

5. Suma a două numere naturale este 23. Cea mai mare valoare a produsului acestora este : a) 126 b) 130 c) 132 d) 1446. Aflaţi suma tuturor numerelor naturale de două cifre nenule care au proprietatea că sunt mai mari cu 54 decât răsturnatele lor. a) 153 b) 175 c) 236 d) 246

7. Ultima cifră a numarului este : a) 0 b) 2 c) 4 d) 6

8. Suma cifrelor numărului este: a) 18128 b) 18137 c) 18146 d) 18164

La problemele 9 şi 10 redactaţi rezolvările complete.9. Fiecare vârf al unui cub se etichetează cu câte un număr de la 1 la 8 şi fiecare faţase etichetează cu câte un număr de la 9 la 14.Există etichetări în care suma etichetelor celor trei vârfuri vecine cu orice vârf să fie aceeaşi ?Există etichetari în care suma etichetelor de pe cele trei feţe care au câte un vârf comun sa fie aceeaşi ? (Vasile Pop)

10. a) Câte cifre de 7 are numărul ?.b) Aflaţi numerele naturale n ştiind că între n şi 2n se află exact 5 numere naturale pare.

Subiectele au fost selectate şi propuse de prof. Meda Bojor–C.N. „Gh. Şincai” Baia Mare

Clasa a VI-aLa problemele 1-8, scrieţi pe foaia de concurs doar litera corespunzătoare răspunsului corect.

1. Soluţia ecuaţiei este: a) 7 b) 8 c) 9 d) 102. Numărul de divizori naturali ai numărului 10584 este: a) 24 b) 32 c) 18 d) 48

3. Dacă şi atunci cea mai mare valoare a numărului este: a) 45 b) 105 c) 200 d) 604. Cel mai mic număr de patru cifre care împărţit pe rând la 8, 9 şi 10 dă restul 1 este: a) 1001 b) 1441 c) 1009 d) 1081

5. Media aritmetică a numerelor şi este:90

Page 91: SOCIETATEA DE ŞTIINŢE MATEMATICE DIN - ISJ … matematica 2015... · Web viewpentru punctaj maxim la sectiunea liceu Zelina Mihai, clasa a X-a, C.N. „V. Lucaciu”, prof. Boroica

a) 502 b) 1006 c) 503 d) 1007.6. Fie punctele coliniare şi în această ordine astfel încât

.

Dacă sunt mijloacele şi atunci este: a) 3 b) 2,5 c) 4 d) 3,5

7. Se consideră segmentul având lungimea 24 cm şi astfel încât BM = 18 cm.

Mijlocul segmentului determinat de mijloacele segmentelor şi se află la distanţa de punctul . Atunci este: a) 18 cm b) 8cm c) 12cm d) 9cm8. Punctele sunt coliniare şi

şi . Lungimea segmentului este:

a) 40 cm b) 30 cm c) 10 cm d) 20 cmLa problemele 9 şi 10 redactaţi rezolvările complete.9. Pe o foaie de matematică cu pătrăţele sunt scrise numerele de la 1 la 121, fiecare într-un pătrăţel. Să se arate că:a) Există un pătrat de dimensiune astfel ca suma celor patru numere aflată în căsuţele lui să fie cel mult 286.b) Există un pătrat de dimensiune astfel ca suma celor patru numere aflată în căsuţele lui să fie cel puţin 210.10. Se dau punctele A1 , A2 ,…, An , An+ 1n∈N ¿ ,n ≥ 1000coliniare şi în această

ordine astfel încât .

a) Să se calculeze lungimea segmentului .b) Aflaţi numărul punctelor, dintre cele date, care se află la o distanţă mai mare de

90cm de dar mai mică de 90 cm faţă de .

Subiectele au fost selectate şi propuse de prof. Gheorghe Boroica–C.N. „Gh. Şincai” Baia Mare

Clasa a VII-aLa problemele 1-8, scrieţi pe foaia de concurs doar litera corespunzătoare răspunsului corect.

1. Numărul este egal cu:

91

Page 92: SOCIETATEA DE ŞTIINŢE MATEMATICE DIN - ISJ … matematica 2015... · Web viewpentru punctaj maxim la sectiunea liceu Zelina Mihai, clasa a X-a, C.N. „V. Lucaciu”, prof. Boroica

a) 48 b) 23 c) 128 d) 125

2. Numărul soluţiilor x∈Z ale inecuaţiei este:a) 7 b) 0 c) 9 d) 83. Suma soluţiilor ecuaţiei

este: a) 0 b) c) d) -

4. Dacă , unde x , yϵ Q , x ≠−1 , y≠−2

, atunci are valoarea: a) 4 b) 0 c) 6 d) 7

5. Rezultatul calculului este:

a) 2 b) c) d) 6. Dacă trapezul isoscel are diagonalele perpendiculare, iar linia mijlocie este

cm atunci aria sa este: a) 10 cm2 b) 16 cm2 c) 100 cm2 d) 20 cm2 7. Fie punctul de intersecţie al diagonalelor patrulaterului convex . Dacă

triunghiurile au ariile respectiv cm2, 200 cm2, 300 cm2, atunci aria triunghiului este: a) 150 cm2 b) 200 cm2 c) 300 cm2 d) 400 cm2

8. Fie un paralelogram cu perimetrul cm, , iar

centrele de greutate ale triunghiurilor .

Atunci este:

a) b) 16 cm c) 25 cm d) 20 cmLa problemele 9 şi 10 redactaţi rezolvările complete.

9. Fie , astfel încât .

a) Calculaţi .

b) Determinaţi câte valori diferite poate lua suma .10.Se consideră pătratele şi şi în exteriorul lor, triunghiurile echilaterale şi .

92

Page 93: SOCIETATEA DE ŞTIINŢE MATEMATICE DIN - ISJ … matematica 2015... · Web viewpentru punctaj maxim la sectiunea liceu Zelina Mihai, clasa a X-a, C.N. „V. Lucaciu”, prof. Boroica

a) Să se arate că ;

b) Să se arate că punctele sunt coliniare;

c) Dacă cm, să se calculeze suma ariilor: , unde

. Subiectele au fost selectate şi propuse de prof. Nicolae Muşuroia–C.N. „Gh. Şincai” Baia Mare

Clasa a VIII-aLa problemele 1-8, scrieţi pe foaie doar litera corespunzătoare răspunsului corect.

1. Partea întreagă a numărului este egală cu: a) 2013 b) 42 c) 43 d) 2014

2. Dacă şi atunci media aritmetică a numerelor este: a) 72 b) 6 c) 48 d) nu se poate determina

3. Valoarea minimă a expresiei unde a , b∈R este:

a) b) c) d)

4. Dacă a , b∈Q şi atunci este: a) 13 b) 1 c) 2 d) 10

5. Dacă x∈ R¿

şi , atunci este: a) 2 b) -3 c) 27 d) 18.6. Numărul maxim de puncte din spaţiu cu proprietatea că distanţa dintre oricare două este de 5cm, este: a) 3 b) 2 c) 4 d) o infinitate7. Un paralelipiped dreptunghic cu dimensiunile de 18 cm, 24 cm şi 36 cm se împarte în cuburi prin plane paralele cu feţele paralelipipedului. Dacă muchia fiecărui cub astfel obţinut este , undel∈N , atunci numărul de valori pe care le poate lua este: a) 1 b) 4 c) 6 d) 128. Piramida patrulateră regulată are toate muchiile egale, atunci m (∡ AVC )

93

Page 94: SOCIETATEA DE ŞTIINŢE MATEMATICE DIN - ISJ … matematica 2015... · Web viewpentru punctaj maxim la sectiunea liceu Zelina Mihai, clasa a X-a, C.N. „V. Lucaciu”, prof. Boroica

este:

a) b) c) d) La problemele 9 şi 10 redactaţi rezolvările complete.

9. a) Să se arate că nu există trei numere prime astfel ca ,

şi .

b) Să se determine numerele prime prime astfel ca ,

şi .(Vasile Pop)

10. Un bloc în formă de cub este format din 27 camere identice. Camera centrală este inaccesibilă.Decideţi dacă următoarele afirmaţii sunt adevărate sau false (justificaţi).Pot fi împărţite cele 26 de camere accesibile în 13 perechi de câte două camere vecine (cu un perete comun pe verticală sau orizontală)?Poate cineva să viziteze toate camerele accesibile trecând o singură dată prin fiecare? (se poate trece din orice cameră într-o cameră vecină)

(Vasile Pop)

Subiectele au fost selectate şi propuse de prof. Florin Bojor–C.N. „Gh. Şincai” Baia Mare

Clasa a IX-aProblema 1. Vârfurile unui cub trebuie etichetate folosind 8 valori distincte din mulţimea{1,2,3,…,n}(n∈N ) astfel încât să se respecte următoarele reguli: (1) suma etichetelor oricăror două vârfuri vecine trebuie să fie multiplu de 3; (2) suma etichetelor vecinilor oricărui vârf să fie multiplu de 3.[două vârfuri se consideră vecine dacă sunt extremităţile aceleaşi laturi] Să se determine cel mai mic număr natural pentru care o astfel de etichetare este posibilă, iar pentru valoarea minimă determinată să se dea un exemplu de o astfel de etichetare.

Problema 2. Se consideră şirurile de numere naturale definite prin relaţiile de recurenţă:

:a) Să se arate că 2 xn

2− yn2=1 ,∀ n∈ N

b) Să se arate că yn=[ xn√2 ] ,∀n∈N .

94

Page 95: SOCIETATEA DE ŞTIINŢE MATEMATICE DIN - ISJ … matematica 2015... · Web viewpentru punctaj maxim la sectiunea liceu Zelina Mihai, clasa a X-a, C.N. „V. Lucaciu”, prof. Boroica

Problema 3. Se consideră ecuaţia { 1{x }}=1

x(x∈R), unde prin se notează partea

fracţionară a numărului real .a) Să se arate că ecuaţia nu are soluţii în mulţimea numerelor raţionale.b) Să se determine o soluţie a ecuaţiei.Problema 4. Fie un semicerc obţinut dintr-un cerc de centru şi rază .a) Să se arate că pentru orice număr par există pe punctele distincte

astfel ca lungimea vectorului să fie mai mică decât 1.b) Să se arate că pentru orice număr impar şi orice puncte distincte

pe S, lungimea vectorului este mai mare decât 1.

Clasa a X-a

Problema 1. Se consideră funcţia f : R → R , f ( x )={ x }−[ x ] ,∀ x∈R, unde şi

notează partea fracţionară, respectiv partea întreagă a numărului real .a) Să se arate că funcţia f este bijectivă şi să se determine funcţia inversă.b) Să se determine valorile lui a∈R pentru care funcţia ga : R → R , ga ( x )=a { x }−[ x ] , ∀ x∈R este bijectivă.Problema 2. Să se determine toate funcţiilef : R → R care verifică relaţia:

;pentru oricex , y∈R, unde este un număr natural fixat.

Problema 3. Pentru orice numere pozitive considerăm suma

:

a) Să se arate că .

b) Să se determine intervalul de lungime minimă cu proprietatea

pentru orice numere pozitive .Problema 4. Să se determine numărul de regiuni în care împart planul cele şase drepte ce unesc câte două vârfuri ale unui patrulater convex.

Clasa a XI-a

95

Page 96: SOCIETATEA DE ŞTIINŢE MATEMATICE DIN - ISJ … matematica 2015... · Web viewpentru punctaj maxim la sectiunea liceu Zelina Mihai, clasa a X-a, C.N. „V. Lucaciu”, prof. Boroica

Problema 1. Fie numere naturale astfel încât şi fie mulţimea

. Să se determine câte dintre submulţimile lui cu elemente conţin cel puţin două numere consecutive.Problema 2. Să se arate că pentru orice număr a∈C există două matrice

B ,C ∈M 2(C) ambele având valorile proprii şi , astfel încât matricea

să aibă valorile proprii şi .

Problema 3. Fie mulţimea tuturor şirurilor de numere complexe ce verifică proprietatea de recurenţă:

unde

:a) Să se arate că mulţimea termenilor tuturor şirurilor din S formează o reţea plană de hexagoane regulate de latură în planul complex.

b) Să se determine toate numerele naturale pentru care există un şir din

astfel încât .

Problema 4. Se consideră şirul de numere reale definit recurent prin

xn+1=1

{xn },∀n∈N

, unde prin se notează partea fracţionară a numărului real .

a) Să se arate că şirul este corect definit dacă şi numai dacă este număr iraţional.

b) Să se arate că, pentru orice valoare a lui , şirul nu poate fi strict descrescător.

c) Să se arate că dacă şirul este strict crescător, atunci este nemărginit.

Clasa a XII-aProblema 1. Fie f : [0,1]→ R

o funcţie care admite o primitivă cu proprietăţile

şi . Să se arate că există , astfel ca

.

96

Page 97: SOCIETATEA DE ŞTIINŢE MATEMATICE DIN - ISJ … matematica 2015... · Web viewpentru punctaj maxim la sectiunea liceu Zelina Mihai, clasa a X-a, C.N. „V. Lucaciu”, prof. Boroica

Problema 2. Fie o mulţime nevidă şi o lege de compoziţie cu

proprietatea că dacă şi , atunci .

a) Să se arate că pentru orice , funcţia este bijectivă.b) Dacă există element neutru, atunci să se arate că orice element din este simetrizabil.

Problema 3. Fie o mulţime finită, nevidă şi o lege de compoziţie

asociativă: .

Să se arate că pentru orice număr natural există un element astfel ca

. (S-a notat ).Problema 4.a) Să se determine toate funcţiile continue f : R → R care verifică relaţia:

f (2 x+1 )=f ( x )+3 x ,∀ x∈Rb) Daţi un exemplu de funcţie discontinuă f : R → R care verifică relaţia de mai sus.

CONCURSUL DE MATEMATICĂ „AS” AL ELEVILOR DIN CENTRUL JUDEȚEAN PENTRU TINERII CAPABILI DE

PERFORMANȚĂEdiţia a VI-a, Baia Mare, 24 ianuarie 2015

Clasa a IX-a1. a) Fie a ,b∈R , două numere reale astfel încât a≥1 , b≥1 . Să se arate că:

(a+ 1a )(b+ 1

b )≥2( ab+ b

a ) .b) Să se demonstreze că:2 (ab+ac+ad+bc+bd+cd )≤3 (a2+b2+c2+d2 ) ∀a ,b , c ,d∈R .c) Dacă x este partea întreagă a numărului -24,71 şi x+8 y=7 , să se calculeze valoarea expresiei:

E=√ (x−7 )2−15 y2+√ ( x+25 )2−15 ( y3−64 ) . 2. a) Să se rezolve ecuaţia:3 x+ (3 x+2 )+(3 x+4 )+.. .+(3 x+100 )=2652 .

97

Page 98: SOCIETATEA DE ŞTIINŢE MATEMATICE DIN - ISJ … matematica 2015... · Web viewpentru punctaj maxim la sectiunea liceu Zelina Mihai, clasa a X-a, C.N. „V. Lucaciu”, prof. Boroica

a) Numerele: x+3 y+1, 3 x+2 y , 2 ( x+1 )+ y , 6 x− y−3 , sunt în progresie

aritmetică. Să se determine x , y∈R şi cele patru numere.

3. În reperul cartezian se consideră punctele A (2,3 ) , B(−3,2) , C (4 ,−1) .Să se determine:

a) Coordonatele vectorilor

b) Coordonatele vectorului

c) Coordonatele punctului D∈Ox , astfel ales încât vectorul să fie

coliniar cu vectorul .

Clasa a X-aSUBIECTUL I

a) Arătaţi că numărul A=log6 1+ log6 3+ log6 12−1+(27

125 )13

este raţional. b) Determinaţi valorile reale ale numărului x , pentru care este definit logaritmul

lg 2−3 x

4 x−1 .

c) Comparaţi numerele x=1+2+3+…+89

89 , y=√2005 şi

z=√18+√50−√242+√32 . SUBIECTUL II Se consideră expresia:

a) Să se calculeze E (2 ) b) Să se arate că

c) Să se rezolve ecuaţia SUBIECTUL III

a) Rezolvaţi în C ecuaţia

b) Se consideră funcţia , , unde este conjugatul

numărului complex z. Să se verifice că ,

c) Pentru funcţia de la b), rezolvaţi ecuaţia

98

Page 99: SOCIETATEA DE ŞTIINŢE MATEMATICE DIN - ISJ … matematica 2015... · Web viewpentru punctaj maxim la sectiunea liceu Zelina Mihai, clasa a X-a, C.N. „V. Lucaciu”, prof. Boroica

Clasa a XI-aSUBIECTUL I

Se consideră matricele ,

a) Demonstraţi că .

b) Calculaţi .

c) Determinaţi x∈ R ştiind că .SUBIECTUL II

Fie Δ (a ,b , c ) determinantul matricei ,a ,b . c∈ R .

a) Calculaţi Δ (0,1,0 ) .b) Determinaţi x∈ R ştiind că Δ (x ,1,−1 )=0.

c) Demonstraţi că: Δ (a , b , c )=(a+b+c )⋅(a2+b2+c2−ab−bc−ac ) ,∀ a ,b , c∈R .SUBIECTUL III

a) Calculaţi limx→3

√x2−2x+6−√ x2+2 x−6x2−4 x+3 .

b) Aflaţi valorile reale ale numerelor a şi b astfel încât funcţia

f : R−{−2 }→R , f ( x )=ax2+bx+1x+2

sa aibă asimptota oblică y=x+2 spre +

∞ .

c) Aflaţi valorile reale ale numerelor a şi b astfel încât limx→∞

(√ x2+x−ax−b )=0.

Clasa a XII-a1.Pe mulţimea numerelor întregi se defineşte legea de compoziţie

, oricare ar fi x , y∈Z , unde sunt numere întregi şi .a) Pentru arătaţi că legea „ „ este asociativă.

b) Arătaţi că legea „ „ este asociativă dacă şi numai dacă .

99

Page 100: SOCIETATEA DE ŞTIINŢE MATEMATICE DIN - ISJ … matematica 2015... · Web viewpentru punctaj maxim la sectiunea liceu Zelina Mihai, clasa a X-a, C.N. „V. Lucaciu”, prof. Boroica

c) Dacă , demonstraţi că legea „ „ admite element neutru dacă şi numai dacă divide .

2. Se consideră funcţiile f , g : R→R , şi .a) Arătaţi că funcţia este o primitivă pentru funcţia .

b) Calculaţi .

c) Calculaţi .

3. Pentru orice număr natural se consideră integrala .

a) Calculaţi .

b) Arătaţi că , oricare ar fi n∈N .

c) Demonstraţi că .

CONCURSUL INTERJUDEŢEAN „PRIN LABIRINTUL MATEMATICII”ediţia a X-a, Baia Mare, 24 ianuarie 2015

CLASA a IV-aSubiectul 1a) În şirul 2015, a, 2007, b, c suma oricăror trei termeni consecutivi este aceeaşi. Să se calculeze a-c.b) Pe nişte bilete sunt scrise numere naturale (câte un număr pe fiecare bilet), astfel încât suma şi produsul lor sunt egale cu 12. Aflaţi numărul biletelor.( Găsiţi toate soluţiile posibile). Subiectul 2Un elev a cumpărat 8 caiete şi 12 creioane pentru care a plătit 144 lei. Un alt elev a cumpărat 10 caiete şi 15 creioane de acelaşi fel cu colegul său. a) Aflaţi câţi lei a încasat librăria de la cei doi elevi? b) Câţi lei costă un creion dacă un caiet costă de 3 ori mai mult?Subiectul 3O urna conţine bile albastre şi bile roşii. O persoană a inventat următorul joc:Extrage succesiv bile, una câte una, până când constată că pentru prima dată numărul bilelor albastre extrase este egal cu numărul bilelor roşii extrase. La unul

100

Page 101: SOCIETATEA DE ŞTIINŢE MATEMATICE DIN - ISJ … matematica 2015... · Web viewpentru punctaj maxim la sectiunea liceu Zelina Mihai, clasa a X-a, C.N. „V. Lucaciu”, prof. Boroica

dintre jocuri constată că în final au fost extrase 10 bile şi că nu există trei bile de aceeaşi culoare extrase consecutiv. Să se arate că în această situaţie a cincea şi a şasea bilă extrase au culori diferite.

Subiectele au fost selectate şi propuse de prof. Bretan Andrei-C.N. „V. Lucaciu” Baia Mare prof. Covaciu Traian-C.N. „V. Lucaciu” Baia Mare

CLASA a V-aSubiectul 1Arătaţi că numărul : a = 3+9+15+21+....+12081+8058+4031, se poate scrie ca sumă de trei pătrate perfecte consecutive.

Subiectul 2Determinaţi numerele prime p şi q pentru care p2 – q2 = 10 + 5p.

(Gazeta Matematică nr. 12/2014)Subiectul 3 În 16 cutii sunt în total 27 bile, fiecare cutie conţinând cel puţin una şi cel mult trei bile. Numărul cutiilor ce conţin o bilă nu este mai mic decât 7, iar numărul bilelor din cutiile ce conţin două sau trei bile este mai mare decât 17. Aflaţi câte cutii conţin o bilă, două bile, respectiv trei bile.

Subiectele au fost selectate şi propuse de prof. Bob Robert-C.N. „V. Lucaciu”Baia Mare prof. Sabău Ştefan-C.N. „V. Lucaciu”Baia Mare

CLASA a VI-aSubiectul 1

Numerele au proprietatea .

a) Determinaţi numerele .

b) Determinaţi numerele .(R.M.T. nr. 4/2014-prelucrare)

Subiectul 2

a) Comparaţi numerele şi

101

abc 22221221 cccbbbaaa

bc9

abc

20152014

20122011

a20142013

20132012

b

Page 102: SOCIETATEA DE ŞTIINŢE MATEMATICE DIN - ISJ … matematica 2015... · Web viewpentru punctaj maxim la sectiunea liceu Zelina Mihai, clasa a X-a, C.N. „V. Lucaciu”, prof. Boroica

b) Generalizare: Comparaţi numerele şi

.Subiectul 3 Pe dreapta d alegem un punct O şi de aceeaşi parte a lui O, luăm punctele

( în această ordine) astfel încât

Pe semidreapta opusă lui , considerăm punctele ( de la

dreapta la stânga) astfel încât Fie

mijlocul segmentului .

a) Aflaţi lungimea segmentelor şi .

b) Aflaţi k şi p ştiind că .( toate segmentele sunt măsurate cu aceeaşi unitate de măsură)

Subiectele au fost selectate şi propuse de prof. Sfara Gheorghe-C.N. „V. Lucaciu” Baia Mare prof. Zlampareţ Horia-C.N. „V. Lucaciu” Baia Mare

CLASA a VII-aSubiectul 1a) Fie a,b,c,x,y,z numere reale nenule astfel încât:

Să se deducă o relaţie numai între a,b,c şi o relaţie numai între x, y şi z.

b) O foaie de dimensiune 9 9 este împărțită în 81 de pătrățele de dimensiuni 11, pe care sunt scrise numerele de la 1 la 81, fiecare într-un pătrățel. Să se arate că există un pătrat de dimensiune 2 2 astfel ca suma numerelor din cele patru pătrățele să fie cel mult 198.

Subiectul 2Se consideră trapezul ABCD cu AB CD. Fie O punctul de intersecţie al diagonalelor trapezului, M mijlocul segmentului BC şi N mijlocul segmentului AD. Să se arate că perimetrul triunghiului MON este jumătate din perimetrul trapezului ABCD dacă şi numai dacă diagonalele trapezului sunt perpendiculare.Subiectul 3

102

43

1

nn

nna

*N

nnn

nnb ,

32

21

kAAA ,...,, 21 .,...3,2,1 32211 etcAAAAOA

1OA pBBB ,...,, 21

.,...2,1, 32211 etcpBBpBBpOB

M pk BA

kOA pOB

31 MA

Page 103: SOCIETATEA DE ŞTIINŢE MATEMATICE DIN - ISJ … matematica 2015... · Web viewpentru punctaj maxim la sectiunea liceu Zelina Mihai, clasa a X-a, C.N. „V. Lucaciu”, prof. Boroica

Se consideră o mulţime A⊂Z care are proprietăţile: (1) 0∊A (2) dacă a,b∊Z și 2a-3b∊A atunci a∊A şi b∊A.

Să se arate că A = Z. (Gazeta Matematică nr. 12/2014) Subiectele au fost selectate şi propuse de

prof. Darolţi Erika-C.N. „V. Lucaciu” Baia Mare prof. Sabău Ştefan-C.N. „V. Lucaciu” Baia Mare

CLASA a VIII-aSubiectul 1

Fie suma .

a) Să se determine .

b) Pentru ce valori ale lui , avem ?Subiectul 2

a) Dacă şi , să se demonstreze că şi .

b) Determinaţi valorile lui pentru care .Subiectul 3Fie triunghiul şi notăm . În vârfurile triunghiului , de aceeaşi parte a planului se ridică perpendicularele

, . Notăm cu G centrul de greutate al triunghiului , iar cu centrul cercului circumscris triunghiului .

Să se arate că:

a)

b)

Subiectele au fost selectate şi propuse de prof. Bretan Andrei-C.N. „V. Lucaciu” Baia Mare prof. Fărcaş Natalia-C.N. „V. Lucaciu” Baia Mare

CLASA a IX-aSubiectul 1Să se determine partea întreagă a numărului

103

,1

1...83

1

32

12

nn

S n2, nn N

nS

2, nn N 122

nSn

N, ba N ba Na Nb

Nn N nn 20092009

ABC acbaABcACbBCa ,,,ABC

33,

33 bBBaAA

33cCC

ABC O CBA

CGBGAG

CBAOG

Page 104: SOCIETATEA DE ŞTIINŢE MATEMATICE DIN - ISJ … matematica 2015... · Web viewpentru punctaj maxim la sectiunea liceu Zelina Mihai, clasa a X-a, C.N. „V. Lucaciu”, prof. Boroica

.Subiectul 2a) Să se arate că triunghiurile ABC şi au acelaşi centru de greutate dacă şi

numai dacă .

b) Să se arate că dacă triunghiul este triunghiul median al triunghiului ABC (triunghiul format de mijloacele laturilor triunghiului ABC), atunci ele au acelaşi centru de greutate.

c) Pe laturile unui triunghi ABC considerăm

Notăm cu respectiv ortocentrele, respectiv centrele cercurilor

circumscrise triunghiurilor BMN, CNP, AMP. Arătaţi că dacă

, atunci triunghiurile şi MNP au acelaşi centru de greutate.Subiectul 3

Să se arate că ecuaţia are:

a) cel puţin două soluţii în cu .b) o infinitate de soluţii în . (Gazeta Matematică nr. 11/2014)Subiectele au fost selectate şi propuse de

prof. Boroica Gabriela-C.N. „V. Lucaciu” Baia Mare prof. Covaciu Traian-C.N. „V. Lucaciu” Baia Mare

CLASA a X-aSubiectul 1

Fie f,g: [0,1] , şi . Să se

arate că:(Gotha Güntter, elev, Baia-Mare)

Subiectul 2a) Fie a şi b două numere reale astfel încât 147a = 3 şi 147b = 7. Să se

calculeze .b) Fie a,b,c > 0 astfel încât a+b+c = 1. Să se deducă

. Pentru ce valori ale numerelor a, b, c

104

, , , 0x y zE x y z

x y x z y z y x z x z y

1 1 1A B C

1 1 1 0AA BB CC

2 2 2A B C

, , .M AB N BC P CA

iH , 1,3iO i

1 2 3 0AH BH CH

1 2 3O O O

23 3 3x y z x y z *** NNN x y z

*** ZZZ

R xxxf )1(1)( xxxg 1)( xg

,xxf

,xxgxf

,xmax

]10[min

]10[max

]10[

aba

11

4

09log9log9log abacbc cba

Page 105: SOCIETATEA DE ŞTIINŢE MATEMATICE DIN - ISJ … matematica 2015... · Web viewpentru punctaj maxim la sectiunea liceu Zelina Mihai, clasa a X-a, C.N. „V. Lucaciu”, prof. Boroica

are loc egalitatea?Subiectul 3Se consideră patrulaterul convex ABCD cu BC=CD. Fie punctul M situat în acelaşi semiplan determinat de dreapta AB cu punctul D astfel încât AM=BM şi ∢ ∢ . Să se calculeze m ∢ ştiind că AD = .Subiectele au fost selectate şi propuse de

prof. Bob Robert-C.N. „V. Lucaciu” Baia Mare prof. Zlampareţ Horia-C.N. „V. Lucaciu” Baia Mare

CLASA a XI-aSubiectul 1

Se consideră matricea şi a) Să se arate că dacă , atunci există a,b,c numere reale astfel încât

.

b) Să se arate că dacă , atunci există şirurile de

numere reale , , astfel încât şi pentru orice număr natural nenul n.c) Să se rezolve în ecuaţia .Subiectul 2

Se consideră şirul definit prin şi .a) Să se demonstreze că şirul este convergent.

b) Să se calculeze .Subiectul 3a) Să se arate că pentru orice două matrice are loc relația:

b) Fie o matrice de ordin doi cu elemente reale și matricea transpusă. Ştiind că să se calculeze .

(Gazeta Matematică nr.11/2014)Subiectele au fost selectate şi propuse de

prof. Boroica Gabriela-C.N. „V. Lucaciu” Baia Mare105

AMB BCD ( )BCD MC3

Page 106: SOCIETATEA DE ŞTIINŢE MATEMATICE DIN - ISJ … matematica 2015... · Web viewpentru punctaj maxim la sectiunea liceu Zelina Mihai, clasa a X-a, C.N. „V. Lucaciu”, prof. Boroica

prof. Darolţi Erika-C.N. „V. Lucaciu” Baia Mare

CLASA a XII-aSubiectul 1

1. Să se calculeze

a)

b)Subiectul 2

Fie un grup şi mulţimea . Dacă ,

pentru orice , atunci G este comutativ. (Gazeta Matematică nr. 12 / 2014)Subiectul 3

Fie şi . Calculaţi şi apoi .

Subiectele au fost selectate şi propuse de prof. Fărcaş Natalia-C.N. „V. Lucaciu”Baia Mare prof. Sfara Gheorghe-C.N. „V. Lucaciu” Baia Mare

CONCURSUL „ GH. ŞINCAI PENTRU MICII MATEMATICIENI”- 29 aprilie 2015 –

I. Considerăm numerele a, b şi c determinate de:

, Iar c este egal cu diferența dintre cel mai mare număr natural de trei cifre distincte și cel mai mic număr natural impar de trei cifre.

1) Calculaţi cele trei numere;2) Arătați că restul împărțirii numărului c la b este egal cu suma cifrelor

numărului a.II. La împărțirea a două numere naturale nenule câtul este de 18 ori mai mic decât

diferența dintre deîmpărțit și rest, iar împărțitorul de 3 ori mai mare decât câtul. 1) Care este câtul împărțirii ?2) Dacă în plus restul este mai mare decât 15, să se determine numerele inițiale.

III. Se consideră următorul şir de numere: 0, 5, 10, 15, 20, 25, ... și șirul corespunzător sumelor cifrelor numerelor din primul șir: 0, 5, 1, 6, 2, 7,....

1) Calculați suma primilor 20 de termeni din primul șir.

106

R∫ xxdxxx ,9sin3sinsin

R

∫ xdxxx ,

11

4

2

,G GyyxxyGxGZ ,)( ex 2

)(GZGx

0,116

11

∫ xdxx

xI 0,116

3

∫ xdxx

xJ JIJI , I

Page 107: SOCIETATEA DE ŞTIINŢE MATEMATICE DIN - ISJ … matematica 2015... · Web viewpentru punctaj maxim la sectiunea liceu Zelina Mihai, clasa a X-a, C.N. „V. Lucaciu”, prof. Boroica

2) Arătați că numerele 4, 8, 9, 10 și 2015 se află în al doilea șir;3) Care este primul număr din primul şir cu suma cifrelor 27? (justificaţi

răspunsul)

Subiectele au fost propuse şi selectate de către : prof. Florin Bojor-C.N. „Gh. Şincai” Baia Mare prof. Gheorghe Boroica-C.N. „Gh. Şincai” Baia Mare

107


Recommended